FREE Practice MCAT (EXPLAINED)

Page 1

MCAT Practice Test 1 EXPLAINED.pages

The mechanism by which the heart reliably pumps blood to the body can be understood by looking solely at the mechanics of the right ventricle, the thickest chamber of the heart. From a fluid motion perspective, the right ventricle can be modelled as a single-chamber pump (Figure 1) attached in series to two pressure-activated, one-way valves. Using radiometric techniques, the volume and shape of the heart can be imaged in real time, as can the volume of blood that passes through the heart during each part of the cardiac cycle. This data can be combined to generate a pressurevolume diagram for the left-ventricle, corresponding to a single, full cardiac cycle (Figure 2). During the diastolic phase (segment AB in the Figures), the heart refills with blood from its contracted state at the end of the previous cycle. Thus, this phase is essentially a return stroke, in which blood slowly fills the heart, increasing its volume without changing the pressure due to the flexibility of the walls. The mitral valve closes at B, at which point the heart begins to contract around the trapped blood. The aortic valve opens at C, but the heart continues to contract---resulting in a rapid efflux of blood at a constant pressure. The aortic valve closes at D, and the heart tissue expands, relieving pressure, until the mitral valve re-opens at A, blood rushes back in, and the cycle repeats. Figure 1: A simple diagram of the four phases of the cardiac cycle

Figure 2: A pressure-volume diagram for the four phases of the cardiac cycle

1


MCAT Practice Test 1 EXPLAINED.pages

1. Which of these following reasons best describes why the volume remains constant despite pressure increase during section BC? Please choose from one of the following options. 1 *Liquid incompressibility 2 Blood efflux 3 Boyle’s law 4 The ideal gas law Hint #1 The ideal gas law, if it applied to blood, would imply a steady volume reduction in response to a pressure increase. Hint #2 The two closed valves prevent blood from exiting the heart during this phase Hint #3 The incompressibility of liquids (like blood) explains the constant volume during section BC

2


MCAT Practice Test 1 EXPLAINED.pages

2. During the pressurization phase BC, what is the pressure in the narrow valve entrance compared to the wider chamber? Please choose from one of the following options. 1 *Smaller 2 Larger 3 The same 4 Zero Hint #1 At point B, fluid is flowing out through the opening Hint #2 Hydrostatic pressure does not reflect the pressure of a flowing fluid Hint #3 The pressure in the opening is smaller due to the Venturi effect 3. During the efflux phase CD, what is the pressure in the narrower valve entrance compared to the wider chamber? Please choose from one of the following options. 1 *Equal 2 Larger 3 Smaller 4 Zero Hint #1 At point C, no fluid flow is occurring in the heart Hint #2 Pressure increases equilibrate very quickly when a fluid is stationary Hint #3 The hydrostatic pressure in the valve and the chamber are EQUAL via Pascal’s principle

3


MCAT Practice Test 1 EXPLAINED.pages

4. During which segment of the cycle does the mechanical potential energy of the heart increase? Please choose from one of the following options. 1 CD 2 AB 3 *BC 4 DA Hint #1 Mechanical potential energy is often stored as either gravitational potential energy or elastic (spring) potential. The former is not relevant to this system. Hint #2 During blood influx and outflux, the volume of the chamber is able to change in response to pressure, and so the tissue does not undergo heavy stretching or deformation. Hint #3 Pay attention to the source of the pressure increase in the heart; since gas laws don’t apply, it must be due to a stored mechanical force (like the flexible rubber membrane of a tire) Hint #4 Mechanical potential is stored during the pressurization phase BC

4


MCAT Practice Test 1 EXPLAINED.pages

5. Which of the following quantities is given by the area enclosed by the loop? Please choose from one of the following options. 1 The total number of moles of blood in the heart times the temperature 2 The net force exerted during one cardiac cycle 3 The total tissue displacement during one cardiac cycle 4 *The total work performed during one cardiac cycle Hint #1 The area within the loop has units of (pressure)Ă—(volume) Hint #2 Analyzing the units, (pressure)Ă—(volume) = (Force/ (Distance^2)*(Distance^3) = (Force)(distance) = Work Hint #3 Alternatively, remember that the area within a closed loop on a pV diagram always gives the work performed by a heat engine. Hint #4 The area of the loop gives the work performed during one cardiac cycle

5


MCAT Practice Test 1 EXPLAINED.pages

Muons are elementary, charged particles that undergo the same interactions as electrons, but which have much greater masses. For this reason, muons could be used as an alternative to electrons in transmission microscopy, in which the pattern formed by scattered, charged particles after passing through a sample may be used to infer the structure of biological tissues. Such methods are useful for probing structural details of biological materials that are smaller than the optical diffraction limit, which determines the smallest feature sizes that can be observed using a traditional, light-based microscope. A simple muon detector (Figure 1) consists of an array of diodes. Each diode emits an electric signal when a muon passes through it, and so the specific diode in the array that emits a signal after a muon strike indicates the two-dimensional location of the strike. A beam of muons with known energy passes through the sample, and at each beam location, the location on the detector plate at which muons arrive after passing through the sample is recorded. Locations at which the beam was most strongly deflected indicate the presence of internal structures, resulting in a twodimensional image of the tissue. The total deflection of the incident beam by the sample is, at most, a few degrees for the densest parts of the sample. This technique has recently been generalized to provide three-dimensional information about samples in a method known as muon tomography. In order to gain information about the three-dimensional scattering field with which the muon interacts before it reaches the detector, the direction that each muon is traveling after it exits the tissue sample must be known. This method requires at least two detector arrays, since the velocity vector of the muon can be calculated from the 2D location on each detector array where the muon struck, the distance between the two arrays, and the time between the two detection events. Assume that the muons are heavy enough to be treated as point masses subject to classical electromagnetism and Newton’s laws. Additionally, assume that the detectors themselves barely affect the muons’ trajectories.

6


MCAT Practice Test 1 EXPLAINED.pages

Figure 1: The elements of a muon measurement assembly. Muons exit the source and are deflected as they pass through the sample. The deflected muons pass through one of the diodes in the diode array, which indicates their position in the plane of the array.

7


MCAT Practice Test 1 EXPLAINED.pages

6. Which of the following best describes the cause of the small-angle deflections of the incident muon beam? Please choose from one of the following options. 1 The muons are captured by unfilled valence orbitals within the tissue, and then re-emitted. 2 The muons exchange their spin with the molecules in the tissue, causing their trajectories to become curved due to exchange of angular momentum. 3 The muons are repelled by the strong nuclear force arising in the nuclei of the tissue. 4 *The muons are electromagnetically repelled by the electrons in the tissue. Hint #1 A deflection (rather than reflection or emission) implies that the muons retain some of their original momentum as they pass through the material. Hint #2 Particles may not exchange spin, which is an intrinsic property distinct from angular momentum. Hint #3 The strong force holds the nucleus together, but it is not responsible for the deflection of a beam of charged particles. Moreover, the questions states that muons are like electrons, which do not interact via the strong force. Hint #4 Muons are charged particles and thus deflected electromagnetically predominantly by the electron cloud within the tissue.

8


MCAT Practice Test 1 EXPLAINED.pages

7. Which of the following would DECREASE the resolution of the three-dimensional structure of a tissue sample in the transmission microscope? Please choose from one of the following options. 1 *Covering the sample in a conductive coating or foil. 2 Applying the beam from multiple incident angles and recording changes in the scattering field. 3 Slicing the sample into thin slices, and imaging each slice separately. 4 Increasing the energy of the beam of muons directed at the sample. Hint #1 There is an inverse relationship between the energy of a particle and its characteristic wavelength. Hint #2 Thicker samples, or samples with significant structural variation along their vertical axis, may scatter muons multiple times, making it more difficult to distinguish single, large scattering events from multiple small scattering events. Two of the answer choices would actually help improve vertical resolution for this reason. Hint #3 A metal coating would absorb or reflect charged particles in a transmission microscope, reducing the resolution of the apparatus.

9


MCAT Practice Test 1 EXPLAINED.pages

8.In order to reconstruct the three-dimensional structure of a tissue during muon tomography, which of the following quantities must be measured during the experiment? I. The total charges of the atoms within the tissue sample II. The velocity of muons entering the sample III. The velocity of muons exiting the sample Please choose from one of the following options. 1 *II and III 2 I and III 3 I and II 4 I, II, and III The biological sample is essentially a “black box”---the purpose of the measurement is to determine its internal structures. Hint #2 The velocities (speeds and directions) of exiting muons are sufficient to create a two-dimensional image of the sample Hint #3 Three-dimensional information is encoded in the changes in all three coordinates of the momentum vector before and after the particle interacts with the sample. Hint #4 The total scattering of a muon is given by the difference in its velocity before and after interacting with the sample. Hint #5 In order to use Newton’s laws in three dimensions, the velocity (speed and direction) must be known before and after the particle interacts with the tissue.

10


MCAT Practice Test 1 EXPLAINED.pages

9. If muons are travelling towards the first detector along a direction normal to the plane of the diode array, where should a second detector array be placed so as to allow the most precise determination of the three-dimensional structure of the sample? Please choose from one of the following options.

1

2

*

3

11


MCAT Practice Test 1 EXPLAINED.pages

4 Hint #1 Three-dimensional information requires knowledge of the velocity of the exiting muons: both their speeds and directions. Hint #2 The position of the muons must be measured at least twice to find their velocity: The difference in the positions of the two detection locations, as well as the time interval between detections, allows the velocity to be determined. Hint #3 Larger intervals in both space and time can be measured with greater precision. Hint #4 Two of the options do not guarantee that the muon will strike both plates. Hint #5 Two parallel plates spaced distantly apart allow the three-dimensional coordinates of the muon to be measured at two separate times, and with greater precision because the muon travels farther between measurement events.

12


MCAT Practice Test 1 EXPLAINED.pages

10. In practice, muon-based measurements are rarely used on living tissues. Which of the following would NOT be a problem with using muons instead of electrons? Please choose from one of the following options. 1 Muons are more difficult to generate 2 *Muons are more likely to lose their charge as they pass through the sample 3 Muons require a greater beam energy to achieve a given incident momentum 4 Muons are heavier and thus decay faster Hint #1 Heavier particles tend to decay faster because they have more possible decay products Hint #2 Muons are not one of the dominant particles that comprise matter, suggesting that they are both scarce and that they decay quickly. Hint #3 The energy of a beam

E = p^2/2m, and so p=√2mE Hint #4 Muons are elementary particles, and their charge is an intrinsic property that they cannot shed. Hint #5 Remember to read the question closely! The question asks for which listed property would NOT be a problem with using muons instead of electrons. Since muons have an intrinsic charge that cannot be shed, the answer choice "Muons are more likely to lose their charge as they

13


MCAT Practice Test 1 EXPLAINED.pages

pass through the sample" is not true, and would thus be the correct answer.

During MRI, a magnetic field is used to align the spins of protons, which have an intrinsic magnetic moment coupled to the direction of their spins. As a result of this static field aligning the individual proton spins and magnetic moments, the sample acquires a net, measurable magnetization parallel to the applied field. Once the protons are aligned with the static field, radio-frequency (RF) photons are then applied to the protons, which cause them to enter a higher energy state with their spins aligned antiparallel to the field. The steps are outlined in Figure 1. During measurement, the applied RF field is then removed, and the protons gradually return to the lower-energy, aligned state, emitting photons with the same energy as the exciting pulse. This relaxation occurs gradually, with the number of spins aligned opposite the static field decreasing exponentially in time with a time constant known as T1, which depends on composition and structure of the tissue. In addition, neighboring spins in the sample tend to become aligned with one another due to pairwise magnetic interactions, which further improves the MRI signal. After the removal of the RF, this spin-spin alignment exponentially decays with a different time constant,T2, which also depends on material properties. The dependence of T1 and T2 on structural properties gives rise to contrast between different tissues in MRI. In many applications, “contrast agents� consisting of fluids with particularly short T1 times are used in order to differentiate otherwise similar tissues. Experimental results indicate a relationship between the observed relaxation timescale and the concentration of contrast agent used, as shown in Figure 2.

14


MCAT Practice Test 1 EXPLAINED.pages

Figure 1: The alignment of proton spins before the measurement begins, after application of a static magnetic field, and after application of a radiofrequency (RF) field that excites them to a higher energy state aligned opposite the field.

Figure 2: The strength of the MRI signal along two, perpendicular measurement axes allows T1and T2 to be separately observed for different concentrations of contrast agent. A: Measurement of the signal along one direction allows T1to be observed for two different concentrations of contrast agent X. B: Measurement along another, orthogonal direction allows T2 to be observed for different concentrations of contrast agent Y.

15


MCAT Practice Test 1 EXPLAINED.pages

Data adapted from: Yoo, B., & Pagel, M. D. (2007). An overview of responsive MRI contrast agents for molecular imaging. Frontiers in Bioscience: a Journal and Virtual Library, 13, 1733-1752. 11. Based on the description of MRI found in the passage, which of the following is most likely proportional to the “signal� graphed against time in Figure 2A? Please choose from one of the following options. 1 The number of RF photons emitted by different parts of the sample. 2 The net magnetization of the sample along the direction of the applied RF field. 3 The number of RF photons absorbed by the sample during excitation. 4 *The net magnetization of the sample along the direction of the static magnetic field. Hint #1 T1 affects the time over which spins return to their equilibrium state, aligned with the static field. Hint #2 As described in the text, aligned magnetic dipoles give rise to a non-zero net magnetization, representing the asymptote in the figure. Hint #3 The number of RF photons emitted by the sample should decrease after the cessation of applied RF Hint #4 The only macroscopic quantity readily measured in this simple MRI is the total magnetization of the material. Hint #5 T1 sets the timescale for alignment with the static field.

16


MCAT Practice Test 1 EXPLAINED.pages

Hint #6 The graph must show the net magnetization along the direction of the applied, static field. This is the only quantity described in the passage that would have a non-zero asymptote with time, and that makes sense physically.

12. Suppose that the applied RF field also heats the sample slightly. What effect would this have on the apparent T1 observed after the RF is removed? Please choose from one of the following options. 1 It would depend on the chemical composition of the contrast agent 2 T1 would remain the same 3 T1 decreases 4 *T1 increases Recall that T1 is a timescale for relaxation to a low-energy state. Hint #2 Thermal effects tend to drive systems towards more disordered states Hint #3 The applied RF field tends to cause spins to become misaligned with the static field. Heating would further contribute to this effect. Hint #4 An increasing temperature should thus result in a larger apparent realignment time, T1

17


MCAT Practice Test 1 EXPLAINED.pages

13. Which of the following explanations provides the best physical reasoning for why increased concentration of a contrast agent would change the effective T2 of the contrast agent? Please choose from one of the following options. 1 *A reduced mean distance between spins increases their electromagnetic coupling. 2 A higher density of emitted photons results in greater re-absorption of photons by neighboring spins. 3 A higher density of photons emitted during relaxation provokes stimulated emission of photons from nearby protons. 4 Higher concentrations of contrast agents shield particles from the applied magnetic field. While stimulated emission is involved in T1, it should not affect spin-spin coupling timescale T2. Stimulated emission would not strongly increase with density increases. Hint #2 It is apparent in both graphs that higher concentrations have smaller relaxation times, eliminating two answer choices. Hint #3 The force between two magnetic dipoles decreases with increasing distance. Hint #4 The most physically likely explanation for the effect is that higher concentrations of contrast agent encourage electromagnetic interactions by decreasing the mean distance between spins.

18


MCAT Practice Test 1 EXPLAINED.pages

14. In the Figure, which of the tested agents displays the largest and which displays the largestT2? 1 2 3 4

T1,

Largest T1: High X and Largest T2: Low Y *Largest T1: Low X and Largest T2: Low Y Largest T1: High X and Largest T2: High Y Largest T1: Low X and Largest T2: High Y

A sample with a large T1 timescale should take a long time to reach the asymptote, corresponding to all spins aligned with the external magnetic field. Hint #2 In a sample with a large spin-spin (T2) relaxation time, it should take a long time for the signal from transversely-magnetized spins to decay to zero. Hint #3 T2 is a signal decay timescale, and so a smaller time constant would indicate faster decay. Hint #4 T1 is a signal recovery timescale, and so a smaller time constant would indicate faster return to the asymptotic value. Hint #5 Low X and Low Y represent the largest T1 and T2, respectively

19


MCAT Practice Test 1 EXPLAINED.pages

15. The frequency of the applied RF signal used to excite spins is directly proportional to the magnitude of the static magnetic field used to align the spins, with proportionality constant 5 hz/T. If the strength of the applied field is known to be 20 T plus or minus 3 T, which of the following correctly describes the uncertainty in the INVERSE frequency (1/frequency)? Please choose from one of the following options. 1 2 3 4

4s 3/5s *3 / 2000 s 1 / 15 s

Error propagation for inverse proportionalities works differently than it does for direct proportionalities Hint #2 The error of a reciprocal relation,

B= k / (1/ω) ΔB=(k / (1/ω)^2) (Δ(1/ω)) Hint #3 Alternately, the error must be a smaller quantity than 1/15 s, which would be the error in the case where the inverse frequency and the magnetic field were directly proportional. Hint #4 The result of substituting the given values for

ΔB, B, and k is 3/2000 s

20


MCAT Practice Test 1 EXPLAINED.pages

Questions 16. 17. 18. 19. 20. are NOT based on passages. 16. Which of the following vector combinations will result in the least amount of displacement? (Note: Vectors a, b, d, and E have magnitudes double that of vectors c and f.)

Please choose from one of the following options. 1 2 3

*

21


MCAT Practice Test 1 EXPLAINED.pages

4 Let’s consider each case graphically to narrow down our choices. Recall that vectors c and f are half the magnitude of vector d. By drawing out c + f - d, we can see that the displacement is somewhat large:

Hint #2 All three vectors have the same magnitude here. By drawing out a + d + e, we can see that displacement is not large, but could be closer to zero:

Hint #3 Between the remaining two, it becomes hard to discern visually which has the least displacement:

22


MCAT Practice Test 1 EXPLAINED.pages

Hint #4 In analyzing a - b + e, let’s pick 10 for the magnitude of each vector, and resolve all the vectors into the x and y direction. In a 45-45-90 triangle, remember that the sides are in a proportion of 1:1:√2, so with the hypotenuse as 10 units, the x and y components are 5√2 units.In the xdirection, the x component vectors e and b overlap and add up to 10√2, which is opposed by vector a, with a magnitude of 10.

23


MCAT Practice Test 1 EXPLAINED.pages

Hint #5 In the y-direction, the y-component of vectors e and b cancel each other. The resultant vector points in the negative y-direction with an approximate magnitude of 4 since √2 is approximately equal to 1.4:

Hint #6 In analyzing e - c + d, let’s pick 10 for the magnitude of vectors d and e and 5 for the magnitude of vector and resolve all the vectors into the x and y components. Vector e resolves into an x and y components with magnitudes of 5√2 units: 24


MCAT Practice Test 1 EXPLAINED.pages

Hint #7 The resultant x-component is 2 in the negative-x direction, and the resultant y-component is 3 in the negative-y direction. To find our displacement, use the Pythagorean theorem to obtain √13, which is less than 4.

25


MCAT Practice Test 1 EXPLAINED.pages

17. A car travels from point A to B in 3 hours and returns back to point A in 5 hours. Points A and B are 150 miles apart along a straight highway. Which of the following statements most accurately describes the motion of the car? Please choose from one of the following options. 1 The average velocity is 40 miles per hour. 2 *The average speed is 37.5 miles per hour. 3 The car travels at 50 mph for the first half and 30 mph for the second half. 4 The total displacement of the trip is 300 miles. Hint #1 The displacement is the shortest distance from the initial to the final position. Since we start and end at the same place, displacement is zero. Hint #2 The average velocity is calculated with the displacement over time, which would also be zero. Hint #3

26


MCAT Practice Test 1 EXPLAINED.pages

The car could travel at any velocity or speed since we only know it traveled 150 miles in 3 hours. The velocity could have oscillated back and forth from 90 mph to 10 mph or remain constant at 50 mph. Hint #4 The average speed is calculated with the distance divided by time, which would be 300 miles divided by 8 hours. 300 / 8 = 37.5 mph.

18. On which of the following does the buoyant force on an object floating on the surface of a liquid most directly depend? Please choose from one of the following options. 1 viscosity of the liquid 2 *density of the liquid 3 mass of the liquid 4 volume of the object Hint #1 The buoyant force is the upward force by the fluid that opposes the downward force of gravity, and in the case of a floating object, the two forces are equal and opposite. Hint #2 The formula for buoyant force is F = pVg, where V is the volume of the object submerged. So the buoyant force does not depend on the volume of the object. Hint #3

27


MCAT Practice Test 1 EXPLAINED.pages

The buoyant force does not depend on the viscosity of the liquid since viscosity refers to fluids in motion and is the resistance to shearing flows. Hint #4 In the formula for buoyant force, the density in question is that of the liquid, so to reiterate: F = p(fluid) x V(displaced) x g

19. After MgCl2 dissolves in a neutrally charged solvent, what is the net charge of the solution? 1 2 3 4

The solution becomes negatively charged due to the majority Cl- ions The solution becomes positively charged due to the stronger Mg+2 ions *The solution remains neutrally charged. The solution becomes positive or negative, depending on the relative Mg+2 and Cl- concentrations

The law of conservation of charge states that the net charge of an isolated system remains constant. Hint #2 When the ions are added to a neutrally charged solvent like water, the overall solution remains the same. MgCl2 dissolves into one +2 cation and two -1 anions. Hint #4 28


MCAT Practice Test 1 EXPLAINED.pages

The only way to change the net charge of a system is to introduce a charge from elsewhere, or to remove a charge from the system. In this closed system, the resulting Mg+2 cation and two Cl- anions will yield an overall neutral charge.

20. Which statement below is true? Please choose from one of the following options. 1 *Sound waves are longitudinal waves and they propagate parallel to the transmitting medium. 2 Sound waves can propagate as longitudinal or transverse waves, depending on the temperature. 3 Sound waves can propagate as longitudinal or transverse waves, depending on the transmitting medium. 4 Sound waves are transverse waves and they propagate perpendicular to the transmitting medium. Hint #1 Longitudinal waves propagate parallel; transverse waves propagate perpendicularly. Hint #2 Transverse waves can only occur on a string, on the surface of a liquid, or through a solid.

29


MCAT Practice Test 1 EXPLAINED.pages

Hint #3 Sound waves are longitudinal waves and the propagate parallel to the transmitting medium. Although this seems like a simple definition question, it is important to realize that a sound wave must be a longitudinal wave because there is no way for a transverse wave to propagate inside a gas or a liquid in a way to maintain the perpendicular driving motion.

An electrocardiogram (ECG) measures the heart’s electrical activity using a series of probes placed on the skin. In addition to detecting rhythmic electrical activity due to the heart’s beating, an ECG can detect other processes, like spasms, that occur with different frequencies than the heartbeat. The ECG is able to convert data indicating the magnitude of the electrical activity as a function of time into information about specific processes that occur in the heart over different timescales using a mathematical operation known as a Fourier transform. In a Fourier transform, a time-varying signal is converted into a histogram showing the various frequencies that constitute the signal. As a result, an ECG voltage versus time graph can be replotted as an amplitude versus frequency graph, which indicates the relative presence of various frequencies in the original voltage reading. For example, The Fourier

30


MCAT Practice Test 1 EXPLAINED.pages

transform of a time series showing an undulating sine wave with frequency 1 hz would be a histogram with peaks at ±1 hz (Figure 1). For a more complex signal consisting of many different sine waves superimposed, the Fourier transform shows the relative amounts of various frequencies present in the signal. For example, in the Figure 2, the Fourier transform of the voltage signal

V(t)=(1 / 3 ) sin(2 π 3 t) + 1sin(2 π t) (Figure 2A) produces a bar chart with a higher peak at1 hz than at3 hz (Figure 2B). A mathematical theorem guarantees that a valid Fourier transform can be generated for any periodic signal, which equivalently suggest that any periodic ECG signal can be represented as a sum of sine waves with various frequencies and amplitudes. The raw output of an ECG is a periodic signal indicating the electrical activity arising from a patient’s heart as a function of time, but the shape of this signal is generally NOT purely sinusoidal-instead, it consists of a superposition of many different periodic processes in the body which generally ARE sinusoidal. As a result, the Fourier transform of an ECG signal can be used to indicate the presence or absence of various processes that occur with different characteristic frequencies based on their presence or absence in the Fourier histogram. Figure 1: A) A sinusoidal output from an ECG and B) its Fourier transform.

31


MCAT Practice Test 1 EXPLAINED.pages

Figure 2: A) An ECG signal consisting of two superimposed sinusoids and B) its Fourier transform.

32


MCAT Practice Test 1 EXPLAINED.pages

Figure 3: The output of an ECG test for a healthy patient contains many different frequencies and, as a result, is generally not a single sine wave.

33


MCAT Practice Test 1 EXPLAINED.pages

34


MCAT Practice Test 1 EXPLAINED.pages

21. Suppose a rare genetic mutation doubles the diameters of the electrically conducting cells that transmit electrical activity throughout the heart and body. Which of the following describes the most direct effect this would have on observed ECG voltage fluctuations if all other aspects of the patient (height, weight, composition) remain the same? Please choose from one of the following options. 1 A doubling of the amplitude. 2 A doubling of the frequency. 3 No direct effect. 4 *A quadrupling of the amplitude. The voltage produced by the heart is reduced linearly by the total resistance the current encounters as it passes through the body on the way to the ECG pads via Ohm’s law. Hint #2 Ohm’s law states that voltage(the signal) is proportional to the current and resistance. Hint #3 A reduced body resistance results in a stronger ECG signal. Hint #4 The resistance of a material goes as 1/(its cross-sectional area). Hint #5 If all conducting channels have twice the diameter, they will have four times the area, reducing resistance (and improving the signal) by a factor of four.

35


MCAT Practice Test 1 EXPLAINED.pages

22. Which of the following traits MUST be present in a Fourier transform of Figure 3? Please choose from one of the following options. 1 *A peak at 2 hz 2 A peak with amplitude 1 V 3 A peak with amplitude 2 V 4 A peak at .5 hz Hint #1 The signal appears to repeat itself every .5 seconds Hint #2 Since the signal has a well-defined frequency, and it must be a sum of sine waves, there must be a component with a frequency matching that of the signal. Hint #3 Two of the answer choices have the wrong units Hint #4 Because the signal repeats every .5 seconds, there must be a 2 hz peak in the Fourier transform.

36


MCAT Practice Test 1 EXPLAINED.pages

23. Which of the following graphs most likely represents the Fourier transform of an ECG reading that is well-fitted by the function

V(t) = 4 sin(2Ď€ 4 t )+ sin(2Ď€ 8 t ) Please choose from one of the following options.

1

37


MCAT Practice Test 1 EXPLAINED.pages

2

3

38


MCAT Practice Test 1 EXPLAINED.pages

4

In Figure 2, the presence of two different frequencies in the ECG output led to two pairs of peaks when the Fourier transform was applied. Hint #2 The x axis locations of peaks correspond to the frequencies (in Hz) of the sine waves from which they were derived with the Fourier transform. Hint #3 The amplitude of each frequency peak is proportional to the amplitude of that frequency component in the signal. Hint #4 The correct figure contains a peak at 8 Hz and 4 Hz, and the 4 Hz peak should be taller.

39


MCAT Practice Test 1 EXPLAINED.pages

24. Which of the following most directly determines the MINIMUM frequency that an ECG device can measure? Please choose from one of the following options. 1 The minimum change in voltage that the device can detect 2 The minimum total voltage that the device can detect. 3 *The total time for which the voltage is measured 4 The minimum time interval between two measurement points on the voltage versus time graph. Hint #1 The Fourier transform maps time to frequency, or inverse time. Hint #2 The longer the measurement period, the smaller the frequency accessible Hint #3 The limits on ECG amplitude sensitivity would not limit the low frequency resolution preferentially. Hint #4 If a frequency is so low that 1/frequency is longer than the total acquisition time, then the voltage will not undergo a full cycle at that frequency, making it difficult to observe Hint #5 The minimum observable frequency is set by the total measurement time.

40


MCAT Practice Test 1 EXPLAINED.pages

25. Which of the following most directly determines the MAXIMUM frequency that an ECG device can measure? Please choose from one of the following options. 1 The minimum change in voltage that the device can detect 2 *The minimum time interval between two measurement points on the voltage versus time graph. 3 The minimum total voltage that the device can detect. 4 The total time for which the voltage is measured Hint #1 The Fourier transform maps time to frequency, or inverse time. Hint #2 The shorter the time interval between successive measurements of the voltage, the larger the frequencies that can be accessed Hint #3 If the voltage varies faster than the time difference between the closest possible readings on the graph, then the ECG will not detect the fluctuation. Hint #4 The maximum observable frequency is set by the interval between two points on the electrical activity graph.

41


MCAT Practice Test 1 EXPLAINED.pages

A simple model of human standing is given by an inverted pendulum, a one-dimensional, classical model for the motion of a single, massive particle (Figure 1). A standing human is approximated as a single mass (located at the center of mass) separated from the ground by a massless rod of fixed length L. The feet are treated as attached to the ground by a fulcrum, such that the center of mass can only undergo motion along an arc of radius L around the feet---thus the mechanics of the system are described by the tilt angle θ, representing the angular displacement of the center of mass from directly above the feet, with the rod positioned normal to the ground. The pendulum is perfectly balanced when the mass is positioned directly above the pivot point; however, a very slight displacement of the mass from this position will cause the pendulum to tip over. Humans can overcome this difficulty and maintain standing balance, at which the system is at equilibrium, by maintaining active control of the position of their center of mass relative to the fulcrum formed by their feet. For small displacements, the ankles work to exert a torque that counteracts gravity and prevents the individual from falling over. The timescale that this restoring force must act to recover equilibrium is proportional to the period of a simple pendulum, demonstrated as T= 2Ď€ √L/g Precise measurements of active balancing can be made by filming an individual standing, and digitally tracking the location of the center of mass after the individual is tilted forward by a known angular displacement at t=0 . A sample figure showing the angular response is given in Figure 2. Surprisingly, these measurements show that the seemingly inert process of standing consists of many coordinated ankle motions that stabilize the body after it undergoes slight deflections. Figure 1: The analogy between a standing patient and an inverted pendulum.

42


MCAT Practice Test 1 EXPLAINED.pages

Figure 2: Simulated data showing the time-varying tilt angle of a standing individual who was tilted forward by 0.2 radians at t = 0, and who is actively returning to their original vertical standing position.

43


MCAT Practice Test 1 EXPLAINED.pages

Concept adapted from: Winter, D. A. (1995). Human balance and posture control during standing and walking. Gait & Posture, 3(4), 193-214.

44


MCAT Practice Test 1 EXPLAINED.pages

26. Which of the following correctly describe the difference between an inverted pendulum (as shown in in Figure 1) and a standard pendulum with identical length and mass? I. An inverted pendulum has minimal kinetic energy when it reaches its equilibrium point II. An inverted pendulum requires a non-gravitational restoring force to remain in equilibrium III. An inverted pendulum reaches a higher maximum gravitational torque Please choose from one of the following options. 1 *I and II 2 I, II, and III 3 I and III 4 II and III The maximum torque due to gravity is Lmg in both cases--for the inverted pendulum, this maximum is reached just before the mass collides with the ground. Hint #2 The kinetic energy is minimal when an individual is standing (equilibrium) Hint #3 Standing requires the influence of the ankles (an external force) to maintain the otherwise unstable equilibrium Hint #4 Only I and II are accurate statements.

45


MCAT Practice Test 1 EXPLAINED.pages

27. Suppose a standing person loses their balance by tripping forward by a fixed angle (theta). Which of the following quantities would be greater for a taller person than a short person of equal mass? Please choose from one of the following options. 1 *The torque that must be exerted by the ankles in order to restore balance. 2 The rate at which the ankles must exert a given torque in order to regain balance 3 The total gravitational force acting on the center of mass 4 The initial angular speed immediately after tripping forward Hint #1 The relevant timescale for the ankles to restore balance is given by the period of a simple pendulum, the rate is the inverse of this timescale. Hint #2 The period of a simple pendulum increases with increasing length Hint #3 The total gravitational force is the same for two objects of equal mass Hint #4 The torque, for a fixed angular displacement, is given by

mgLsin(θ) Hint #5 The taller person’s ankles must counteract a greater gravitational torque

46


MCAT Practice Test 1 EXPLAINED.pages

28. In Figure 2, which of the following sets the initial amplitude at t = 0? 1 2 3 4

The mass of the individual The magnitude of gravitational acceleration *The angular distance by which the person initially tilts forward The rate of the sudden tilt just before t=0

The graph depicts only information about the response of the standing individual after a sudden tilt at t = 0 Hint #2 The acceleration due to gravity sets the frequency of a simple pendulum, and so it determines the response time to equilibrate an inverted pendulum. Hint #3 The rate of a force application does not uniquely determine the amount by which the individual tilts forward Hint #4 The initial amplitude represents the initial tilt of the individual.

47


MCAT Practice Test 1 EXPLAINED.pages

29. In Figure 2, which of the following determines the frequency of the decaying oscillations? Please choose from one of the following options. 1 The mass of the individual 2 The rate of the sudden tilt just before t=0. 3 The angular distance by which the person initially tilts forward 4 *The magnitude of gravitational acceleration The rate of the initial tilt does not affect the response of the system after the cessation of the tilting force. Hint #2 The magnitude of the initial tilt does not affect the response of the system after the cessation of the tilting force. Hint #3 The period of a simple pendulum is independent of the mass. Hint #4 The acceleration due to gravity sets the frequency of a simple pendulum, and so it determines the response time to equilibrate an inverted pendulum.

48


MCAT Practice Test 1 EXPLAINED.pages

30.Which of the following provides the most likely explanation for the decrease in amplitude of successive peaks of the sinusoidal response in Figure 2? Please choose from one of the following options. 1 Fatigue in the ankles providing the restoring force. 2 As the tilt angle changes, the torque decreases. 3 Gravitational potential energy being dissipated as heat. 4 *Energy exiting the system due to damping effects. Hint #1 With a fatiguing restoring force, the amplitude would increase in successive cycles as the ankles increasingly fail to counteract gravity Hint #2 The potential energy is greatest when the angular displacement is zero, towards which the signal decays. Hint #3 The torque increases with the tilt angle, and the angle of the torque does not contribute to the amplitude of the signal in successive cycles. Hint #4 The decay in amplitude of oscillations is most likely the result of frictional or feedback damping dissipating the energy stored in the oscillations.

49


MCAT Practice Test 1 EXPLAINED.pages

Lyophilisation, or freeze-drying, is a technique of dehydration which utilizes low pressure, low temperature environments to induce the sublimation of water content from a material. Sublimation is the process converting a substance from a solid directly to a gas, as shown in Figure 1. Figure 1. Generalized diagram indicating the phase change of sublimation.

50


MCAT Practice Test 1 EXPLAINED.pages

Perhaps the most recognizable form of freeze-drying is freeze-dried ice cream, sometimes also called ‘astronaut ice cream’ due to its development and use in the Apollo space missions; however, lyophilisation also has many applications in the pharmaceutical agricultural industry. Freeze-drying is ideal for preservation because low water content prevents enzymes and microorganisms from spoiling or degrading the substance. Additionally a freeze-dried substance can be rehydrated easily because of the microscopic pores left behind by the ice crystals when they sublimate away. This makes lyophilisation ideal for the long term storage and relatively fast reconstitution of substances, e.g. pharmaceuticals from an inert to an active form. There is a three stage process to lyophilisation. First the substance must be frozen, usually at very low temperatures between -50°C and -80°C. Once frozen, the substance is placed in a vacuum on plates, and a small amount of heat is added to help sublime the water directly from solid ice to water vapor. This is known as the primary drying stage, which removes 95% of a material’s water content. It can last up to several days, and requires rigorously maintained conditions due to the sensitivity of the sublimating structure. Finally, there is a secondary drying stage, which involves the temperature being raised again slightly and pressure being lowered further to sublime as much of the last 5% of moisture as possible. Researchers have found in their own astronaut ice cream making experiments that varying the temperature conditions can influence the rate of sublimation, as well as the consistency of the dehydrated ‘cake’ product. Table 1 shows how various temperatures affect the overall structure of freeze-dried ice cream during the primary drying stage by increasing temperature.

51


MCAT Practice Test 1 EXPLAINED.pages

Table 1. Results of lyophilisation of ice cream under varying temperature conditions.

Condition # 1' 2' 3' 4' 5' 6' 7' 8' 9' 10' 11' 12' 13' 14'

' ' ' ' ' ' ' ' ' ' ' ' ' '

' ' ' ' ' ' ' ' ' ' ' ' ' '

Sublimation rate (mg/h) 142.8'' 30.3' ' 273.5'' 312.8'' 184' ' 160' ' 348' ' 49.6' ' 35.7' ' 23.8' ' 161.1'' 225.3'' 124.9'' 298.2''

' ' ' ' ' ' ' ' ' ' ' ' ' '

Temperature: Variable and Stable (°C) -49 to -37' -41 to -39' -31 to -23' -25 to -14' -26 to -23' -21 to -18' -20 to -17' -47' ' -44' ' -42' ' -38' ' -28' ' -22' ' -11' '

' ' ' ' ' ' ' ' ' ' ' ' ' '

Cake Structure ' ' ' ' ' ' '

Collapsed Solid Solid Loose Loose Solid Loose Collapsed Loose Solid Solid Solid Solid Solid

Citation: Tables Adapted from: Xiang, Jun, Hey, Jeffery M., Liedtke, Volker, D.Q. Wang, Investigation of freeze–drying sublimation rates using a freeze–drying microbalance technique. International Journal of Pharmaceutics Volume 279, Issues 1–2, 26 July 2004, Pages 95–105 Nail SL1, Jiang S, Chongprasert S, Knopp SA., Fundamentals of freezedrying. Pharm Biotechnol. 2002;14:281-360.

52


MCAT Practice Test 1 EXPLAINED.pages

31. Following the secondary drying stage, researchers found that the water vapor would consistently rehydrate the freeze-dried ice-cream, ruining their cake structure. Which of the following would best prevent the rehydration? Please choose from one of the following options. 1 *Increasing the temperature at which the ice cream is frozen to form less ice crystals 2 Utilizing a very slow sublimation rate in order to allow the water vapor to dissipate 3 Increasing the pressure during the drying stages to slow down the rate of sublimation 4 Decreasing the pressure during the drying stages to promote rapid sublimation Promoting sublimation simply sublimates ice into water vapor quickly, but it does not address the problem of rehydration. Hint #2 The water vapor would not dissipate over time because it is within in a sealed vacuum. Hint #3 Forming less ice crystals initially in the process would result in less microscopic pores to be formed, which would discourage rehydration from occurring.

53


MCAT Practice Test 1 EXPLAINED.pages

32. If the researchers were looking to maximize the speed of making ice cream while maintaining a solid consistency of their product, which condition should they provide for freeze-drying? Please choose from one of the following options. 1 2 3 4

Condition 3 Condition 7 Condition 12 *Condition 14

Condition 7 involves the fastest rate but its consistency is not solid. Condition 3 provides a solid consistency but it is not the fastest rate. Condition 14 provides the fastest rate of sublimation while maintaining a solid consistency.

33. During the sublimation of water from a solid to a gas, how is the temperature of water being affected? Please choose from one of the following options. 1 Temperature is decreasing because substances release heat during phase changes 2 *Temperature does not change during a phase change 3 Temperature is increasing because it requires heat to induce a phase change 4 The temperature varies during a phase change because it is dependent upon pressure Hint #1 While the temperature of a phase change is dependent on pressure, it will not vary during the phase change itself. Hint #2 Heat is required during a phase change, but the added heat will not increase the temperature. Hint #3 Temperature does not increase during a phase change due to the enthalpy of a phase transition

54


MCAT Practice Test 1 EXPLAINED.pages

34. Which of the following means of heat transfer would be least effective in warming a substance during the drying stages of lyophilisation? Please choose from one of the following options. 1 Conduction 2 Radiation 3 *Convection 4 All three means of transfer would be equally useful Hint #1 Conduction is the primary form of heating through the drying plates the substance rests upon. Hint #2 Radiation would also be relatively effective in heating the substance. Hint #3 Convection would not be effective because the substance is placed within a vacuum.

55


MCAT Practice Test 1 EXPLAINED.pages

35. Often, scientists will place a super-cooled (−50 °C) condenser plate in the freeze-drying vacuum to prevent water vapor from rehydrating their freeze-dried product. How would a condenser plate help to prevent rehydration of the product? Please choose from one of the following options. 1 The condenser plate could condense water into a liquid and have it drained away 2 The condenser plate could absorb heat radiated during the secondary drying stage to prevent over-heating of the product 3 *The condenser plate could act as a surface for water vapor to undergo deposition 4 The condenser plate could keep the environment cooler during the primary drying stage so that none of the initial ice crystals melt into the product The ice crystals are at no real risk of melting, the temperatures during freeze drying are never above the melting point of water. Hint #2 Although the name is misleading, the condenser plate is actually too cold to condense water into a liquid. Hint #3 A condenser plate could be used as a surface onto which the water vapour could deposit from water vapor directly to ice. This would effectively sequester any extra water vapor from rehydrating a freezedried product.

56


MCAT Practice Test 1 EXPLAINED.pages

Questions 36. 37. 38. 39. 40. are NOT based on passages. 36. Hypochlorous acid dissociates in water to create hydronium ions and hypochlorite ions HOCl + H2O H3O+ + OCl- Suppose that additional hypochlorite ions are added to the solution. Which of the following correctly describes the resultant effect on the concentration of HOCl? Please choose from one of the following options. 1 *It increases 2 It depends on the umber of hydronium ions 3 It increases 4 It decreases Le Châtelier's principle states that equilibrated solutions tend to resist changes in the relative concentrations of chemical species. Hint #2 The reaction thus seeks to minimize the effect of the added OClions, tilting the equilibrium to the left. Hint #3 The concentration of HOCl increases

57


MCAT Practice Test 1 EXPLAINED.pages

37. Carbon monoxide (CO) is a potent competitive inhibitor of hemoglobin, and it has an affinity for hemoglobin over 200 times greater than oxygen. What is the formal charge of the oxygen on the molecule? Please choose from one of the following options. 1 *+1 2 0 3 -1 4 -2 Hint #1 The HONC (pronounced hunk or honk) shortcut, which correspond to 1, 2, 3, or 4 bonds, will be employed for all questions in this set. When these four atoms have the corresponding bonds, then the atoms are neutral. One less bond to that atom makes its more negative by 1, and one more bond more positive by 1. Hint #2 Here is a chart that summarizes the shortcut:

Hint #3 Start by drawing a skeleton of C−O and consider the double bond. Looking at the formal charges, carbon would have no formal charge, as well as the oxygen. This would seem correct, but remember the octet rule. It is not much of a rule since C, N, O, and F are the only four elements that must adhere to the rule. Unless there is a carbene (:CH2), carbon usually needs to fill its octet even though C=O seems to be a feasible Lewis dot structure.

58


MCAT Practice Test 1 EXPLAINED.pages

Hint #4 n analyzing the triple bond C≥O, carbon has a formal charge of -1, and oxygen a formal charge of +1. Both have their octets filled, and so here is the Lewis dot structure:

Hint #5 To find the formal charge of the oxygen, start with the number of valence electrons, which is 6. Subtract any unbonded electrons, which is 2, and then subtract one-half of any bonded electrons, which is 3. Hint #6 Generally in determining structures, the more electronegative atom tends to have a negative formal charge, and the more electropositive a positive one. Despite being more electronegative, oxygen does not get assigned the automatic negative charge but has instead a formal charge of +1.

59


MCAT Practice Test 1 EXPLAINED.pages

38. 500 mL of water and 500 mL of dichloromethane (d = 1.3 g/mL) are added to a mixture containing benzoic acid, cresol, methoxyethane, and N-methylethanamine. Four solutions are available for extraction of the mixture: HCN, HCl, NaOH, and LiHCO3. After the initial wash, the top layer from each extraction was retained and one of the preceding solutions was added. Which of the following statements most accurately describes this procedure? Please choose from one of the following options. 1 Cresol must be extracted first with a strong mineral acid such as HCl.. 2 *Benzoic acid can be extracted and isolated with either LiHCO3 or NaOH. 3 After washing with LiHCO3 and then NaOH, only Nmethylethanamine and methoxyethane remain in the separatory funnel. 4 Methoxyethane will only be extracted upon protonation into a carbocation. Cresol is like phenol, but with an extra methyl group at the para position. Due to its alcohol functional group, it is considered a weak acid and would only be extracted with the stronger base. With base extraction, consider the strength of the acid before picking the base. Hint #2 Methoxyethane is an ether. While ethers can get protonated, the carbocation is only an intermediate and is of transient existence. It will remain in the organic layer once all the other 3 compounds are extracted. Hint #3 N-methylethanamine is a base due to its amine group, and amines are best extracted by a strong acid. Hint #4 Since we are retaining the top layer of each extraction, after the first wash, we have benzoic acid in the aqueous layer, which is the top layer. If we wash this layer with NaOH, we would have benzoate ion in the aqueous layer. No further separation is possible. Hint #5 60


MCAT Practice Test 1 EXPLAINED.pages

Benzoic acid can be extracted and isolated with either a weak or strong base, but if in a mixture with an alcohol, it should be extracted first with a weak base like LiHCO3 and the alcohol extracted a strong base like Na2S. 39. A new drug is developed which selectively cleaves covalent bonds between two sulfur atoms of non-adjacent amino acids in a polypeptide chain. Which level of protein structure in affected molecules would be most directly affected by the drug? Please choose from one of the following options. 1 Primary structure 2 Quaternary structure 3 *Tertiary structure 4 Secondary structure Primary structure is defined by the peptide bonds between amino acids. These bonds occur between the amino and carboxy termini of two respective amino acids. Hint #2 Secondary structure is largely defined by characteristic folding patterns such as the alpha helix and B-sheet. These are stabilized by hydrogen bonds between main chain carbonyl oxygens and amide hydrogens. Hint #3 Tertiary structure is the 3D shape of the amino acid, which is stabilized by various interactions and forces within the folded polypeptide chain. Disulfide bonds are unique to the tertiary structure, which provide a strong, stable bond between the sulfur atoms of two cysteines in a protein. Hint #4 This drug would mostly affect the tertiary structure of proteins.

61


MCAT Practice Test 1 EXPLAINED.pages

40. Chemical reactions can be classified according to free energy changes. A chemical reaction has a ΔG of -686/kcal/mol. Is this an endergonic or exergonic reaction? How would the addition of catalyst change the ΔG this reaction? Please choose from one of the following options. 1 *Exergonic, the catalyst would not reduce ΔG. 2 Endergonic, the catalyst would reduce the ΔG. 3 Exergonic, the catalyst would reduce the ΔG. 4 Endergonic, the catalyst would increase the ΔG. Hint #1 ΔG represents the different of free energy between products and reactants Hint #2 A negative ΔG means that the products have less energy than the reactants Hint #3 Because energy was released, negative ΔG, the reaction is exergonic. Hint #4 A catalyst, or enzyme, lowers the activation energy but it does not reduce ΔG.

62


MCAT Practice Test 1 EXPLAINED.pages

As a pump, the heart continually is increasing and decreasing the pressure it exerts on blood. During diastole, or the relaxation phase, blood flows into the various chambers (called the atria and ventricles) of the heart, and during systole blood is pumped out. The total mechanical energy involved in this process can be calculated simply by multiplying the pressure of the blood by the volume of blood ejected from a given chamber (stroke volume). Stroke volume is distinct from the total volume of blood in a heart chamber, because not all blood is ejected from a chamber during a contraction. This amount of energy transferred by the heart during a contraction is often referred to as ‘stroke work’. Much like work done by an internal combustion engine, we can treat the heart as a thermodynamic system, and visually represent stroke work with the use of a pressurevolume diagram, as seen in Figure 1. Figure 1. Generalized Cardiac Pressure-Volume Loop Diagram

63


MCAT Practice Test 1 EXPLAINED.pages

Let’s say that this diagram describes the relationship of volume and pressure specifically for the left ventricle of a heart. Position 1 of the graph would then mark the opening of the mitral valve, which allows blood to fill the chamber from the left atrium (diastole). Position 2 marks the closing of the mitral valve, and the beginning of systole. The chamber begins to contract, increasing blood pressure until it raises high enough to push open the aortic valve, leading out of the heart (Position 3). Once blood is ejected from the chamber (Position 4), pressure will decrease and the cycle is completed, which begins diastole again. There are various ways which we can influence how much work the heart does in a given stroke. One way we can increase stroke work is by increasing the volume of blood in the chamber during diastolic filling, known as preload. In other words, because the volume of preload is directly proportional to both stroke volume and blood pressure, an increase or decrease of preload will also increase or decrease stroke work respectively. 41. Which area of the graph would be equal to the amount of work done to the system after systole? Please choose from one of the following options. 64


MCAT Practice Test 1 EXPLAINED.pages

1 2

The area contained within the 4 points of the cycle The area underneath the curve from 2 - 3

3

*The area underneath line A from points 1

4

The area underneath line C entirely from points 3

-2 -4

Net stroke work is found by adding the work done by the system (curve3-4) to the work put into the system (curve1-2) since the work done by the system is negative, we can subtract the larger area from the smaller area to get the net work done by the system, which is the shaded area of the graph. Hint #2 The area underneath curve 3-4 is representative of the work done by the system. Hint #3 The area underneath curve 1-2 is representative of the work put into the system.

42. From step 4 to step 1, no work is being done either to or by the system. What type of thermodynamic process would this be? Please choose from one of the following options. 1 Adiabatic process 2 Isobaric process 3 Isothermal process 4 *Isochoric process An isobaric process involves a process at constant pressure. from4 to 1, pressure is changing. Hint #2 An isothermal process is one that occurs at a constant temperature. Hint #3 An adiabatic process is one that occurs so quickly that no energy is lost as heat, cardiac muscle cannot function that quickly. Hint #4 65


MCAT Practice Test 1 EXPLAINED.pages

Because volume is being held constant as pressure changes, the process from 4 to 1 is an isochoric process (also known as a constant-volume process, an isovolumetric process, or an isometric process)

43. How would a decrease in preload affect the shape of the Cardiac Pressure-Volume Loop? Please choose from one of the following options. 1 It would increase the area within the cycle, and shift it to the right 2 It would increase the area within the cycle, and shift it to the left 3 It would decrease the area within the cycle, and shift it to the right 4 *It would decrease the area within cycle, and shift it to the left Hint #1 If preload is decreased, then the net stroke work would decrease, and the area within the cycle would decrease. Hint #2 Increasing preload would result in an increase in pressure and stroke volume, shifting the cycle to the right. Hint #3 Decreasing preload would result in a decrease in pressure and stroke volume, shifting the cycle to the left.

66


MCAT Practice Test 1 EXPLAINED.pages

44. Aortic stenosis is a condition which stiffens the aortic valve. This results in the need for more pressure to cause ejection during systole. Which of the following would represent a cardiac cycle of someone with aortic stenosis? Please choose from one of the following options.

1

2

67


MCAT Practice Test 1 EXPLAINED.pages

2

3 The graph should indicate an increased area because the heart has to work harder to push the blood past the stenotic valve. Hint #2 Aortic stenosis would not result in an increased volume within the ventricle. Hint #3 The graph should indicate a large increase in pressure during systole, which is evidenced in the second graph.

68


MCAT Practice Test 1 EXPLAINED.pages

45. An adiabatic process is a thermodynamic process that involves maintaining a constant entropy, that is, no heat is lost from the system during the process. Would the cardiac cycle be considered an adiabatic process? Please choose from one of the following options. 1 Yes, only mechanical energy from the heart is transferred directly to the blood 2 Yes, mechanical energy and elastic potential energy are transferred without an increase in entropy 3 *No, some energy is lost as heat due to the chemical processes of cardiac muscle contraction 4 No, some energy is lost as heat due to the active process of opening heart valves Hint #1 Heart valves open due to the pressure of blood pushing on them, therefore they are opened by a passive process. Hint #2 An adiabatic process happens in an extremely short time, much faster than a heartbeat. Hint #3 Energy will be lost as heat during muscle contraction due to the metabolic processes involved in fueling cardiac tissue.

69


MCAT Practice Test 1 EXPLAINED.pages

A Carnot engine is a type of engine that allows for the maximum amount of thermodynamic energy to be transformed into work. Although it is physically impossible to create one, the Carnot model is useful for determining the maximum efficiency of an engine. Scientists interested in increasing the fuel efficiency of automobiles begin by developing a heat engine, the closest engineers have come to replicating a Carnot engine. The heat engine includes a piston can move up or down allowing work to be done on the gas or allow the gas to do work on the surroundings. Neon, a monatomic ideal gas, is placed in a sealed canister connected to the tightly fitted piston. The canister can be placed in a temperature controlled room which allows heat to be transferred to the gas or allows heat to flow out of the gas. At the start of each experimental trial the gas is reset to a temperature of 298K and atmospheric pressure (101,000Pa). Assume that + work refers to the case where the surroundings do work on the gas, and – work refers to the case where the gas does work on the surroundings. Figure 1. A diagram of the heat engine the scientists are using.

70


MCAT Practice Test 1 EXPLAINED.pages

46. During a certain trial, the canister is placed in a room which is at a temperature of 350K, and the piston is seen to move upwards. What can be said about the heat (Q) and the work done on the gas (W) during this trial? Please choose from one of the following options. 1 Q is -, and W is + 2 Q is -, and W is 3 *Q is +, and W is 4 Q is +, and W is + If heat flows into a gas, the heat Q will be positive. If heat flows out of a gas, the heat will be negative Hint #2 If the gas is compressed, the work done on the gas is positive. If the gas expands, the work done on the gas is negative Hint #3 Since heat flows in to the gas, and the gas expands, Q is + and W is -

71


MCAT Practice Test 1 EXPLAINED.pages

47. During a certain trial, the piston is prevented from moving and the canister is placed in a room which is at a temperature of 255K. During this trial what will happen to the average kinetic energy of the atoms of the gas in the canister and why? Please choose from one of the following options. 1 *The average kinetic energy of the gas atoms will decrease since the heat will be negative 2 The average kinetic energy of the gas atoms will decrease since the work done by the gas will be positive 3 The average kinetic energy of the gas atoms will increase since the heat will be positive 4 The average kinetic energy of the gas atoms will decrease since the work done by the gas will be negative Hint #1 If heat flows into a gas, the heat Q will be positive. If heat flows out of a gas, the heat will be negative Hint #2 The gas always starts at room temperature 298K

72


MCAT Practice Test 1 EXPLAINED.pages

Hint #3 Since the piston does not move, no work can be done Hint #4 Since the room temperature is lower than the initial temperature of the gas heat will flow out of the gas which means the heat flow will be negative and the kinetic energy of the gas will go down

48. During a certain trial a researcher does 3.2J of work by pushing the piston down. During the time while this work is done, the room in which the canister is placed absorbs 4.5J of heat from the gas. What can be concluded about the motion of the gas molecules after this trial compared to the motion before the trial? Please choose from one of the following options. 1 The average speeds of the gas molecules increase 2 The speeds of all gas molecules decrease 3 *The average speeds of the gas molecules decrease 4 The speeds of the gas molecules does not change Hint #1 If heat flows into a gas, the heat Q will be positive. If heat flows out of a gas, the heat will be negative Hint #2 If the gas is compressed, the work done on the gas is positive. If the gas expands, the work done on the gas is negative Hint #3

73


MCAT Practice Test 1 EXPLAINED.pages

The internal energy of the gas decreases here since the negative heat had more impact than the work done on the gas Hint #4 Since the internal energy of the gas decreased, the average speed of the molecules will decrease

49. During a certain trial, the piston is prevented from moving and the canister is placed in a room, which is at a temperature of 450K. What can be concluded about the pressure of the gas? Please choose from one of the following options. 1 The pressure of the gas will decrease since the heat will be 2 The pressure of the gas will not change since no work can be done 3 The pressure of the gas will increase since the work done will be + 4 *The pressure of the gas will increase since the heat will be + The room is at a higher temperature so heat will flow into the gas Hint #2 The piston cannot move so no work can be done Hint #3 The internal energy of the gas will increase but the volume does not change Hint #4

74


MCAT Practice Test 1 EXPLAINED.pages

Since the internal energy of the gas will increase but the volume does not change, the pressure must increase from heat flowing into the gas

50. During a certain trial, the pressure of the gas is tripled while the volume is cut in half. What can be said about the kinetic energy of the gas molecules? Please choose from one of the following options. 1 The average kinetic energy of the gas molecules decreases since the temperature of the gas decreases 2 *The average kinetic energy of the gas molecules increases since the temperature of the gas increases 3 The average kinetic energy of the gas molecules does not change since temperature will not change 4 The average kinetic energy of the gas molecules increases since the temperature of the gas decreases Hint #1 PV=nRT Hint #2

75


MCAT Practice Test 1 EXPLAINED.pages

Think about what happens to the temperature if pressure is tripled and volume is cut in half Hint #3 The temperature must increase, which means the average kinetic energy of the gas increases

Human skeletal muscle tissue is uniquely structured by specialized layers. Muscles are composed of a collection of muscle fibers. In turn, these fibers are made of closely packed myofibrils, or a subunit made of tubular muscle cells. These myofibrils are surrounded by sarcoplasm, a substance similar to cytoplasm in other cells. Finally, myofibrils are composed of repeated sections of sarcomeres, which contain actin and myosin filaments. Physiologists interested in testing muscle cell tension capacity and musculoskeletal dynamics removed a sarcomere from a primate and placed it in physiological laboratory conditions. They analyzed the sarcomere’s movements under a number of different scenarios, including movement. A simplified model of a section of this sarcomere is given in Figure 1.

76


MCAT Practice Test 1 EXPLAINED.pages

Figure 1. A section of the sarcomere used by the scientists.

The tension in the actin between each myosin section are summed up and referred to as a single tension. The tension following myosin section 1 (M1) is referred to as TA, and the tension following myosin section 2 (M2) is TB. This section of actin is directly connected to the sarcomere’s end. Scientists noted that M1 contained a abnormal mass, making its mass greater than the mass of M2. The mass of M1 = 60kDa, and the mass of M2 = 30kDa. 1.6x10^{-24} kg = 1kDa. Assume that frictional forces are negligible. 51. If the mass of M1 > the mass of M2, and the sarcomere is moving with constant velocity, what can be said about the tension in both actin sections? Please choose from one of the following options. 1 *Both tensions are the same and are zero 2 Both tensions TA and TB are equal and are not equal to zero 3 TA is smaller than TB since actin B has to pull the mass of both boxes 4 TA is larger than TB since the mass of M1 is larger than the mass of M2

77


MCAT Practice Test 1 EXPLAINED.pages

The sarcomere is moving with constant velocity Hint #2 Since the sarcomere is moving with constant velocity there is no acceleration Hint #3 : Since there is no acceleration the net force on the both myosin sections must be zero Hint #4 Since the net force must be zero, and there’s no friction, the tension in actin A must be zero, which means the tension in actin B must also be zero

52. If the mass of M1 > the mass of M2 and the sarcomere is accelerating to the right, how will the tensions in the actin sections compare? Please choose from one of the following options. 1 Both tensions TA and TB are equal and are not equal to zero 2 TA is larger than TB since the mass of M1 is larger than the mass of M2 3 *TA is smaller than TB since actin B has to pull the mass of both boxes 4 Both tensions are the same and are zero Hint #1 Use ÎŁF=ma for the horizontal forces on the masses Hint #2 The only horizontal force on the 60kDa mass is the tension from the actin A Hint #3

78


MCAT Practice Test 1 EXPLAINED.pages

Actin B has to pull the mass of both myosin sections so TB is greater than TA

53. During one trial, the acceleration is 2m/s^2 to the right. What calculation will give the tensions in the actin filaments during this trial? Please choose from one of the following options. 1 TA= 1.6x10^-24 x 60 x 2, TB= 1.6x10^-24 x (60+30) x 2 2

TA = 60 x 2, TB = 30 x 2

3

TA= 1.6x10^-24 x 60 x 2, TB= 1.6x10^-24

4

*TA = 60 x 2, TB = (30+60) x 2

x 30 x 2

Think about Newton’s second law ΣF=ma for each box and draw a force diagram for each box Hint #2 Newton’s second law requires the use of kg, not kDa. The conversion between the two is 1.6x10^-24kg = 1kDa 79


MCAT Practice Test 1 EXPLAINED.pages

Hint #3 The mass for TA is the mass of M1. The mass for TB is the mass of M1 plus the mass of M2. Hint #4 For M1, ΣF=ma gives TA=60kDa x 1.6x10^{-24} kg x 2m/s^2

54. If the maximum force an actin section can maintain before tearing is 9pN, what is the maximum acceleration the scientists could make the sarcomere go while maintaining its structural integrity? Please choose from one of the following options. 1 *9x10^−12N = (60kDa x 1.6 x 10^-24k) x a 2 9x10^−9N = (60kDa x 1.6 x 10^-24k) x a 3 9x10^−12N = (30kDa x 1.6 x 10^-24k) x a 4 9x10^−9N = (30kDa x 1.6 x 10^-24k) x a Use ΣF=ma and isolate acceleration algebraically. Hint #2 F = ma. We know the force is 9 picoNewtons, or 9 x 10^-12N Hint #3 You must use the mass of M1, as this will be able to handle less acceleration than M2. This conclusion can be drawn by the fact that mass and acceleration are inversely proportional in F = ma. Hint #4 Convert the mass of M1 from kDa to kg: 60kDa x 1.6 x 10^-24kg Hint #5 80


MCAT Practice Test 1 EXPLAINED.pages

Plug in the variables: 9x10^-12N = (60kDa x 1.6 x 10-24kg) x a

55. Assume tensions are distributed equally amongst actin filaments

in TA. What is the force F1 on one filament of the first actin section if acceleration is a? Please choose from one of the following options. 1 F1 = (60 x 1.6 x 10^-24)a x 4 2 F1 = 60a / 4 3 *F1 = (60 x 1.6 x 10^-24)a / 4 4 F1 = (60 x 1.6 x 10^-24)a Use F = ma to solve for the maximum force possible Hint #2 Note that there are 4 actin filaments in each actin section. So, the tension will be divided equally amongst these filaments. Hint #3 The calculation to convert kDa to kg is 60kDa * 1.6 * 10^-24 Plugging in our numbers and dividing by 4, we get: F1 = (60 x 1.6 x 10^-24)a / 4

81


MCAT Practice Test 1 EXPLAINED.pages

Questions 56. 57. 58. 59. are NOT based on passages. 56. Which of the following statements most accurately describes the comparison between the rate constant k and the equilibrium constant keq? Please choose from one of the following options. 1 A large k means that a reaction will go to equilibrium, and a large keq means that a reaction will go to completion quickly. 2 k is formed from the concentration of the reactants raised to their stoichiometric coefficients, while keq is formed from the concentration of the products over the reactants each raised to its stoichiometric coefficient. 3 *Both constants are dependent on temperature, but only Keq is independent of concentration. 4 The rate constant k indicates how long the reaction will take to complete, and the equilibrium constant keq indicates the ratio of the product concentrations to the reactant concentrations at equilibrium. 82


MCAT Practice Test 1 EXPLAINED.pages

Let’s evaluate each statement individually. A large rate constant k means that the reaction will go to equilibrium quickly, while a large equilibrium constant Keq means that the forward reaction will go to completion, and the concentration of products will exceed that of the reactants. keq is formed from the concentrations of the reactants each raised to their stoichiometric coefficients, while k is formed from the concentrations of the reactants raised to the stoichiometric coefficients only if dealing with elementary reactions. Otherwise, the order of the reaction must be determined by experimental data. Hint #3 The rate constant indicates how quickly it will take to reach equilibrium, which is similar to how long the reaction will take to reach completion. The keq does give an indication of the relative concentrations of the reactants versus the products at equilibrium. If we are talking about ratio, then its the concentrations set to their stoichiometric coefficients. Hint #4 The correct answer is that both constants are dependent on temperature. keq is independent of concentration. Once at equilibrium, if the concentrations of the reactants and products change, they will readjust such that the ratio equals keq again. Rate is not independent of concentration; the greater the concentration, the greater the initial speed of reaction.

83


MCAT Practice Test 1 EXPLAINED.pages

57. Which of the following statements can reasonably be deduced from the phase diagram of helium below?

84


MCAT Practice Test 1 EXPLAINED.pages

Please choose from one of the following options. 1 *Solid and gaseous helium never exist in equilibrium with each other at any temperature or pressure. 2 At atmospheric pressure, helium can exist in all three phases, as well as a supercritical fluid phase near absolute zero. 3 At room temperature, helium exists as a gas, and as pressure increases eventually becomes a solid. 4 Helium does not exist as a solid below 25 atmospheres due to its low mass. Helium has some peculiar features in its phase diagram. A typical phase diagram looks like the following:

85


MCAT Practice Test 1 EXPLAINED.pages

Hint #2 At room temperature, helium exists as a gas, and as pressure increases it eventually becomes a liquid. Looking at the line boundary between solid and the two liquid phases, the line curves up. Since the graph is using the logarithmic scale, the line curves more steeply than it would seem. Hint #3 At atmospheric pressure, helium can exist as a supercritical fluid (He-II), normal liquid (He-I), and gas, but not as a solid. Helium does not exist as a solid below 25 atmospheres due to the weak London dispersion forces that exist between noble gas atoms.

86


MCAT Practice Test 1 EXPLAINED.pages

58. Which of the following statements best characterizes the data represented in the liquid-vapor phase diagram of carbon dioxide where the dotted line represents phase change?

Please choose from one of the following options. 1 *Above the isotherm for 31degrees celcius, carbon dioxide exists only as a supercritical fluid. 2 Moving from point 1 to 2, carbon dioxide is undergoing condensation. 3 Moving left along the isotherm from point 1, the pressure increases drastically since carbon dioxide becomes an incompressible solid. 4 Point 5 is the critical point of carbon dioxide. This is a P-V graph, so the plotted lines represent isotherms, whereby each line represents a different temperature. Normally, the plot is hyperbolic in shape, but as the temperature decreases, the plot flattens into a liquidvapor equilibrium region or condensation region.

87


MCAT Practice Test 1 EXPLAINED.pages

Hint #2 Moving from point 1 to 2, carbon dioxide is undergoing evaporation not condensation since the liquid region is to the left and the vapor region is to the right. Hint #3 To the left of point 1, the pressure increases drastically because carbon dioxide has condensed into an incompressible liquid not solid. Hint #4 Point 3 is the critical point of carbon dioxide, and the isotherm through point 3 is the critical isotherm. Hint #5 Above the isotherm through point 3, carbon dioxide exists only as a supercritical fluid. 59. Which of the following carboxylic acids has the highest boiling point? Please choose from one of the following options. 1 heptanoic acid 2 octanoic acid 3 nonanoic acid 4 *decanoic acid The boiling point of a solution is affected by the intermolecular forces of the molecules Hint #2 Increased intermolecular forces increases the ability of molecules to stick together, causing the need for more energy to break them apart. Hint #3 Each answer choice has a carboxyl functional group and therefore hydrogen bonds. However, each answer choice differs in the amount of carbons in the chain. Increasing the amount of carbons increases the molecular weight. The larger the molecular weight, the stronger the Van Der Waals intermolecular forces, necessitating more heat energy to break them apart. Hint #4

88


MCAT Practice Test 1 EXPLAINED.pages

Decanoic acid, having the longest carbon chain will require the most heat energy for its molecules to separate and will therefore have the highest boiling point. The rationalizing of society can be conceptualized as the pursuit of efficiency, predictability, calculability, and control through technology. But rational systems inevitably spawn a series of irrationalities that result in the compromising and perhaps even the undermining of their rationality. Fast-food restaurants, which epitomize the rational model, proffer the fastest means of getting from a hungry state to a sated one, without surprises, at low cost, in a carnival-like setting suggesting that fun awaits the consumer at each visit. The wholesomeness of the food seems an insignificant consideration. Whereas in the past, working people were prepared to spend up to an hour preparing dinner, they now are impatient if a meal is not on the table within ten minutes. (For their part, some fast-food restaurants have developed chairs that become uncomfortable after about twenty minutes, to ensure that diners do not stay long.) Fast-food restaurants have preferentially recruited adolescent help, at least until recently, because this age group adjusts more easily than adults do to surrendering their autonomy to machines, rules, and procedures. Few skills are required on the job, so workers are asked to use only a minute portion of their abilities. This policy is irrational from the standpoint of the organization, since it could obtain much more from its employees for the money (however negligible) it pays them. These minimal skill demands are also irrational from the perspective of the employees, who are not allowed to think or to respond creatively to the demands of the work. These restrictions lead to high levels of resentment, job dissatisfaction, alienation, absenteeism, and turnover among workers in fast-food franchises. In fact, these businesses have the highest turnover rate of any industry in the U.S. The entire workforce of the fast-food industry turns over three times in a year. Although the simple, repetitive nature of the work makes it easy to replace those who leave, the organization would clearly benefit from keeping employees longer. The costs of hiring and training are magnified when the turnover rate is extraordinarily high.

89


MCAT Practice Test 1 EXPLAINED.pages

The application of the rational model to the house-building process in the 1950s and ’60s led to suburban communities consisting of nearly identical structures. Indeed, it was possible to wander into the residence of someone else and not to realize immediately that one was not at home. The more expensive developments were superficially more diversified, but their interior layouts assumed residents who were indistinguishable in their requirements. Furthermore, the planned communities themselves look very similar. Established trees are bulldozed to facilitate construction. In their place, a number of saplings, held up by posts and wire, are planted. Streets are laid out in symmetrical grid patterns. With such uniformity, suburbanites may well enter the wrong subdivision or become lost in their own. Many of Steven Spielberg’s films are set in such suburbs. Spielberg’s strategy is to lure the viewer into this highly repetitive world and then to have a completely unexpected event occur. For example, the film Poltergeist takes place in a conventional suburban household in which evil spirits ultimately disrupt the sameness. (The spirits first manifest themselves through another key element of the homogeneous society—the television set.) The great success of Spielberg’s films may be traceable to a longing for some unpredictability, even if it is bizarre and menacing, in increasingly routinized lives. Adapted from G. Ritzer, The McDonaldization of Society. ©1993 by Pine Forge Press. 1. The author’s argument suggests that the primary motive of employers who make humans work with machines is to: Please choose from one of the following options. 1 improve the quality of their products. 2 reduce the cost of wages and benefits. 3 avoid seeming to be behind the times. 4 *increase the uniformity of procedures.

90


MCAT Practice Test 1 EXPLAINED.pages

This is a “Foundations of Comprehension” question, which means that it wants you to understand the central theme or idea and be able to figure out what specific words or phrases mean within that context. Hint #2 In paragraph 2, the author uses the fast-food industry to exemplify an organization using the rational model "to proffer the fastest means of getting from a hungry state to a sated one, without surprises, at low cost... The wholesomeness of the food seems an insignificant consideration." This suggests that the author believes quality of the product is not an important motivation (option A). Although the author mentions the motivation of offering a low-cost product, there is no discussion about reducing wages and benefits as part of the model (option B). There is also no discussion of the need to stay with the times (option C). The opening sentence reads, “The rationalizing of society can be conceptualized as the pursuit of efficiency, predictability, calculability, and control through technology.” (Paragraph 1) In this sentence, the author mentions both the pursuit of "predictabilty" and "control through technology". In the second paragraph, the author cites the desire to avoid surprises for the consumer. In the third paragraph, the author cites employer preference to hire employees that are comfortable with "surrendering their autonomy to machines, rules and procedures". This is consistent with the claim that the employer has uniformity of procedures as a goal (option D). Hint #3 Thus, option D is correct.

91


MCAT Practice Test 1 EXPLAINED.pages

2. A common thread in the discussion of fast food and the discussion of suburban housing is that people today: Please choose from one of the following options. 1 are increasingly resistant to the regimentation of life. 2 expect their needs to be met at the lowest possible cost. 3 *allow themselves to be treated as interchangeable. 4 are unable to discriminate among products that differ in quality. Hint #1 This is a “Foundations of Comprehension” question, which means that it wants you to examine relationships among paragraphs to identify the central themes and ideas. Hint #2 Although the passage cites job dissatisfaction, resentment, and turnover in the fast food industry, and a “a longing for some unpredictability” with respect to suburban housing, there is no assertion that individuals are resisting regimentation of life, nor that they are increasingly resistant (option A), rather the passage describes a general acquiescence to the rational model. The only sentence about consumer preference in relation to fast food states that consumers are “impatient if a meal is not on the table within ten minutes”. There is no discussion of consumers expecting their needs to be met at the lowest possible cost either with respect to fast food or suburban housing (option B). There is no discussion of whether people can discriminate quality (option D). The passage describes adolescent employees in the fast food industry as being more willing to surrender “their autonomy to machines, rules, and procedures,” (paragraph 3) and to work in jobs where they are “not allowed to think or to respond creatively to the demands of the work.” (paragraph 3) From an employer’s standpoint, “the simple, repetitive nature of the work makes it easy to replace those who leave.” (paragraph 4) In terms of suburban housing, people live in “nearly identical structures,” (paragraph 5) get easily lost in one another’s homes,

92


MCAT Practice Test 1 EXPLAINED.pages

and long for movies that offer unpredictability in “increasingly routinized lives.” (paragraph 7) These descriptions detail a number of ways in which people allow themselves to be treated as interchangeable (option C). Hint #3 Thus, option C is correct.

3. Information in the passage suggests that a rationalized travel agency would emphasize: Please choose from one of the following options. 1 *planned tours to popular attractions with accommodations at large hotels. 2 computerized systems to provide low-cost customized itineraries. 3 personnel trained to make reservations but with little experience as travelers. 4 procedures that encourage problem-solving initiatives by managers. Hint #1 This is a “Reasoning Beyond the Text” question, which means that it wants you to either apply or extrapolate the ideas in the passage to new situations or to assess how new information would impact the ideas presented in the passage. It is important to understand the assumptions underlying the article, and how new information may or may not shift the central thesis. Hint #2 The passage describes that industries using a rational model attempt to provide a large number of consumers with a standard set of experiences. In the fast-food industry, the author mentions both the pursuit of "predictability" and the desire to avoid surprises for the consumer. consumers expect meals within ten minutes and tolerate chairs that accommodate them comfortably for 20 minutes. In the suburban housing industry, consumers buy homes that are indistinguishable from one another, and live in planned suburban communities that look similar to one another. Extending this to the rationalized travel agency, we would expect a similar degree of planning and standardization for large groups of consumers. Planned tours at popular destinations and accommodation in large hotels (which presumably offer a standard set of floor plans and amenities) characterizes this approach. Use of customized itineraries

93


MCAT Practice Test 1 EXPLAINED.pages

implies the opposite of the rational model because it offers individual variation (option B). There is no discussion of using employee background, including prior experiences or lack of prior experiences, as a part of personnel decisions according to the rational model (option C). Finally, the rational model implies minimizing creativity and thinking for employees, and does not discuss different practices for senior-level employees such as managers (option D). Hint #3 Thus, option A is correct. 4. Suppose that the employee responses to working conditions in fast-food franchises (paragraph 4) also apply to entry-level assembly line workers. In light of this information, the author’s main point in mentioning these responses is: Please choose from one of the following options. 1 weakened, since the fast-food industry is not unique in suppressing creativity. 2 weakened, since the monotony of work is not necessarily related to employee dissatisfaction. 3 *strengthened, since predictability and employee turnover are associated in another context. 4 strengthened, since low wages and job dissatisfaction are associated in another context. Hint #1 This is a “Reasoning Beyond the Text” question, which means that it wants you to either apply or extrapolate the ideas in the passage to new situations or to assess how new information would impact the ideas presented in the passage. It is important to understand the assumptions underlying the article, and how new information may or may not shift the central thesis. Hint #2 The passage uses the fast-food industry as an example of how employees are often unhappy with jobs that force them to follow strict routines and prevent them from thinking or responding creatively to the demands of their work. This job dissatisfaction, in turn, leads to high employee turnover. If this pattern is also seen in another context such as entry-level assembly line workers, then the author’s main point would be strengthened. An additional example to illustrate the author’s main point would strengthen, not weaken, the argument (options A and B). The author does not discuss

94


MCAT Practice Test 1 EXPLAINED.pages

low wages as a source of dissatisfaction (option D). The author’s main point is about employee dissatisfaction related to being forced to use only a standard set of routines and the resultant turnover problem for the employer (option C). Hint #3 Thus, option C is correct.

Americans are a “positive” people. This is their reputation as well as their self-image. In the well-worn stereotype, they are upbeat, cheerful, and optimistic. Who would be churlish enough to challenge these happy features of the American personality? Take the business of positive “affect,” which refers to the mood they display to others through their smiles, their greetings, their professions of confidence and optimism. Scientists have found that the mere act of smiling can generate positive feelings within us, at least if the smile is not forced. In addition, recent studies show that happy feelings flit easily through social networks, so that one person’s good fortune can brighten the day even for only distantly connected others. Furthermore, psychologists agree that positive feelings can actually lengthen our lives and improve our health. People who report having positive feelings are more likely to participate in a rich social life, and social connectedness turns out to be an important defense against depression, which is a known risk factor for many physical illnesses. It is a sign of progress, then, that economists have begun to show an interest in using happiness rather than just the gross national product as a measure of an economy’s success. Happiness is, of course, a slippery thing to measure or define. Philosophers have debated what it is for centuries, and even if they were to define it simply as a greater frequency of positive feelings than negative ones, when they ask people if they are happy, they are asking them to arrive at some sort of average over many moods and moments.

95


MCAT Practice Test 1 EXPLAINED.pages

Surprisingly, when psychologists measure the relative happiness of nations, they routinely find that Americans are not, even in prosperous times and despite their vaunted positivity, very happy at all. A recent metaanalysis of over a hundred studies of self-reported happiness worldwide found Americans ranking only twenty-third. Americans account for twothirds of the global market for antidepressants, which happen also to be the most commonly prescribed drugs in the United States. How can Americans be so surpassingly “positive” in self-image and stereotype without being the world’s happiest and best-off people? The answer is that positivity is not so much their condition as it is part of their ideology—the way they explain the world and think they ought to function within it. That ideology is “positive thinking,” by which they usually mean two things. One is the generic content of positive thinking—that is, the positive thought itself—which can be summarized as “Things are pretty good right now, at least if you are willing to see silver linings, make lemonade out of lemons, etc., and things are going to get a whole lot better.” The second thing they mean by “positive thinking” is this practice of trying to think in a positive way. There is, they are told, a practical reason for undertaking this effort: positive thinking supposedly not only makes us feel optimistic but actually makes happy outcomes more likely. How can the mere process of thinking do this? In the rational explanation that many psychologists would offer today, optimism improves health, personal efficacy, confidence, and resilience, making it easier for us to accomplish our goals. A far less rational theory also runs rampant in American ideology —the idea that our thoughts can, in some mysterious way, directly affect the physical world. Negative thoughts somehow produce negative outcomes, while positive thoughts realize themselves in the form of health, prosperity, and success. For both rational and mystical reasons, then, the effort of positive thinking is said to be well worth our time and attention. Adapted from B. Ehrenreich, Bright-sided. ©2009 by Metropolitan Books. 5. According to the passage, positive feelings are: Please choose from one of the following options.

96


MCAT Practice Test 1 EXPLAINED.pages

1 2 3 4

universal hereditary *contagious ephemeral

This is a “Foundations of Comprehension” question, which means that it wants you to understand the central theme or idea and be able to figure out what specific words or phrases mean within that context. Hint #2 The passage does not describe positive feelings as being universal (option A), in fact it discusses individual differences in the amount of positive feelings that people report which suggests that they are not ubiquitous. The passage does not describe positive feelings as being hereditary (option B). The author writes that positive “affect” which includes the way that a person’s mood is displayed to others, can affect members of a group. He writes that, “happy feelings flit easily through social networks, so that one person’s good fortune can brighten the day even for only distantly connected others” (paragraph 2). This movement of positive feelings from one individual to other individuals, means that positive feelings are contagious (options C). The passage does not describe positive feelings as being ephemeral (option D). Hint #3 Thus, option C is correct.

6. Suppose that economists do start using happiness instead of the gross national product as a measure of an economy’s success. Information presented in the passage would predict which of the following? I. The transition will be fraught with difficulty. II. The gross national product of the United States will appear to decrease. III. The economy of the United States will be seen as relatively less successful than today's. Please choose from one of the following options. 1 *I only 2 II only

97


MCAT Practice Test 1 EXPLAINED.pages

3 4

I and III only II and III only

This is a “Reasoning Beyond the Text” question, which means that it wants you to either apply or extrapolate the ideas in the passage to new situations or to assess how new information would impact the ideas presented in the passage. It is important to understand the assumptions underlying the article, and how new information may or may not shift the central thesis. Hint #2 The passage states that, “Happiness is, of course, a slippery thing to measure or define,” (paragraph 3) which implies that the transition to this metric will be fraught with difficulty (statement I). Nevertheless, when national happiness has been measured and compared to other countries, the United States has not ranked particularly well. The author notes that “A recent meta-analysis of over a hundred studies of self-reported happiness worldwide found Americans ranking only twenty-third” (paragraph 4). To corroborate this finding, the author brings up the widespread usage of antidepressants. However, there is no explicit mention of the US economy being successful when evaluating the GNP. Since this is not explicitly a part of the passage, there is no basis for the comparison made in statement III. The hypothetical suggestion to use happiness instead of GNP as a measure of the economy makes no prediction for what might happen to the GNP (statement II). Hint #3 Thus, the only correct statement is statement I, corresponding to option A. 7. What best represents the author’s explanation for why Americans can be “so surpassingly ‘positive’ in self-image and stereotype without being the world’s happiest and best-off people?” Please choose from one of the following options. 1 *Americans’ positivity is not a true reflection of their affect. 2 Being “well-off” is not the same as being “happy.” 3 Stereotypes tend to be unwarranted generalizations. 4 Americans tend to have high rates of depression.

98


MCAT Practice Test 1 EXPLAINED.pages

This is a “Foundations of Comprehension” question, which means that it wants you to understand the central theme or idea and be able to figure out what specific words or phrases mean within that context. Hint #2 A main theme of this passage is that positivity is part of an American ideology that governs how they think they should think and act, and not an expression of their condition or true affective state as stated in the second sentence of paragraph 5 “positivity is not so much their condition as it is part of their ideology” (option A). The passage doesn’t explore the concept of how being “well-off” relates to happiness (option B). Although stereotypes can sometimes be unwarranted generalizations , this is not a concept described in the passage (option C). Finally, in paragraph 4 the passage discusses high rates of the use of antidepressants in the United States, but this is offered as evidence for relative happiness of Americans, not as an explanation for how Americans can be “so surpassingly ‘positive’ in self-image and stereotype without being the world’s happiest and best-off people (option D). Hint #3 Thus, option A is correct.

99


MCAT Practice Test 1 EXPLAINED.pages

For someone used to contemporary academic writing, reading the chapter on color in William Gladstone’s Studies on Homer and the Homeric Age (1858) comes as rather a shock—the shock of meeting an extraordinary mind. It is therefore all the more startling that Gladstone’s nineteenth century tour de force comes to such a strange conclusion: Homer and his contemporaries perceived the world in something closer to black and white than to full Technicolor. No one would deny that there is a wide gulf between Homer’s world and ours: in the millennia that separate us, empires have risen and fallen, religions and ideologies have come and gone, and science and technology have transformed our intellectual horizons and almost every aspect of daily life beyond all recognition. Surely one aspect that must have remained exactly the same since Homer’s day, even since time immemorial, would be the rich colors of nature: the blue of sky and sea, the glowing red of dawn, the green of fresh leaves. Gladstone says things are not the same, for many reasons. One, Homer uses the same word to denote colors which, according to us, are essentially different. For example, he describes as “violet” the sea, sheep, and iron. Two, Homer’s similes are so rich with sensible imagery, we expect

100


MCAT Practice Test 1 EXPLAINED.pages

to find color a frequent and prominent ingredient, and yet his poppies have never so much as a hint of scarlet. Three, Gladstone notes, Homer uses “black” about 170 times, “white” 100 times, “red” thirteen, “yellow” ten, “violet” six times, and the other colors even less often. Four, Homer’s color vocabulary is astonishingly small. There doesn’t seem to be anything equivalent to our orange or pink in Homer’s color palette; most striking is the lack of any word that could be taken to mean “blue.” What is more, Gladstone proves that the oddities in Homer’s Iliad and Odyssey could not have stemmed from any problems peculiar to Homer. “Violet-colored hair” was used by Pindar in his poems. Gladstone is well aware of the utter weirdness of his thesis—nothing less than universal color blindness among the ancient Greeks—so he tries to make it more palatable by evoking an evolutionary explanation for how sensitivity to colors could have increased over the generations. The perception of color, he says, seems natural to us only because humankind as a whole has undergone a progressive “education of the eye” over the last millennia. The eye’s ability to perceive and appreciate differences in color, he suggests, can improve with practice, and these acquired improvements are then passed on to offspring. But why, one may well ask, should this progressive refinement of color vision not have started much earlier than the Homeric period? Gladstone’s theory is that the appreciation of color as a property independent of a particular material develops only with the capacity to manipulate colors artificially. And that capacity, he notes, barely existed in Homer’s day: the art of dyeing was in its infancy, cultivation of flowers was not practiced, and almost all of the brightly colored objects we take for granted were entirely absent. Other than the ocean, people in Homer’s day may have gone through life without ever setting their eyes on a single blue object. Blue eyes, Gladstone explains, were in short supply; blue dyes, which are very difficult to manufacture, were practically unknown; and natural flowers that are truly blue are rare. Gladstone’s analysis was brilliant, but completely off course. Indeed, philologists, anthropologists, and even natural scientists would need

101


MCAT Practice Test 1 EXPLAINED.pages

decades to free themselves from the error of underestimating the power of culture. Adapted from G. Deutscher, Through the language glass: why the world looks different in other languages. ©2010 Metropolitan Books. 8. It can be inferred from the passage that the author believes which of the following about contemporary academic writing? Please choose from one of the following options. 1 *Academic papers are typically not especially brilliant. 2 Academics seldom address color perception in their papers. 3 Academics often reach very strange conclusions in their papers. 4 Academic papers are usually outdated soon after they are written. This is a “Foundations of Comprehension” question, which means that it wants you to understand the author’s central theme or idea or point of view. Hint #2 The author begins the passage by saying that “For someone used to contemporary academic writing, reading the chapter on color in William Gladstone’s Studies on Homer and the Homeric Age (1858) comes as rather a shock—the shock of meeting an extraordinary mind” (paragraph 1). If someone used to reading contemporary academic writing is shocked at reading something extraordinary, then the implication is that academic papers are typically not especially brilliant (option A). The passage does not directly deal with the prevalence of color perception as content in academic writing (option B). The author refers to the strange conclusion that Gladstone reaches as being shocking, but does not assert that the conclusions of most academic writing are very strange (option C). The author does not discuss the idea that papers may quickly become outdated (option D). Hint #3 Thus, option A is correct. 9. It has been suggested that the Iliad and the Odyssey were a patchwork of a great number of popular ballads woven together from different poets, rather than a single work by a poet named Homer. If true, how would this affect the opinions expressed in the passage?

102


MCAT Practice Test 1 EXPLAINED.pages

Please choose from one of the following options. 1 *It would strengthen Gladstone’s basic thesis. 2 It would weaken Gladstone’s basic thesis. 3 It would require a modification of Gladstone’s basic thesis. 4 It would not affect Gladstone’s basic thesis. Hint #1 This is a “Reasoning Beyond the Text” question, which means that it wants you to either apply or extrapolate the ideas in the passage to new situations or to assess how new information would impact the ideas presented in the passage. It is important to understand the assumptions underlying the article, and how new information may or may not shift the central thesis. Hint #2 Gladstone’s basic thesis is that “Homer and his contemporaries perceived the world in something closer to black and white than to full Technicolor” and that there was “universal color blindness among the ancient Greeks”. If the Iliad and Odyssey were a patchwork composed by a great number of writers, then it would help to show that a group of individuals used a restricted range of colors in their writings rather than just a single individual (Homer) or two (Pindar). This would greatly strengthen Gladstone’s basic thesis because it would suggest that the “oddities in Homer’s Illiad and Odyssey could not have stemmed from any problems particular to Homer” (option A). If a larger group of authors show the same limitations in their reference to color, this would strengthen, not weaken the author’s argument (option B). Because it provides support for the author’s argument, the argument would not require modification, which excludes option C. It does however affect the thesis by strengthening it, which excludes option D. Hint #3 Thus, option A is correct. 10. Gladstone would predict which of the following about the children of an interior decorator who easily distinguishes among scarlet, burgundy, and fuchsia? Please choose from one of the following options. 1 *The children would be able to easily distinguish various versions of red.

103


MCAT Practice Test 1 EXPLAINED.pages

2 3 4

The children would be drawn more to objects in various versions of red than to those of any other color. The children would seldom bother mentioning what are to them obvious differences among various versions of red. The children would need to practice distinguishing among various versions of red for years before achieving proficiency.

This is a “Reasoning Beyond the Text” question, which means that it wants you to either apply or extrapolate the ideas in the passage to new situations or to assess how new information would impact the ideas presented in the passage. It is important to understand the assumptions underlying the article, and how new information may or may not shift the central thesis. Hint #2 Gladstone suggests that the eye can be trained and that using and manipulating colors helped humankind learn how to appreciate nuances in color. Furthermore, the thesis includes the idea that, “these acquired improvements are then passed on to offspring” (paragraph 5). This statement suggests that the children of an interior decorator would be similarly able to easily distinguish various versions of red (option A). Gladstone doesn’t suggest that this enhanced discrimination means that children would be attracted to objects of a certain color (option B). In addition, there is no suggestion that having a nuanced appreciation for color diminishes the need to mention differences in color (option C). In fact, the opposite is likely true based on Gladstone’s analysis which equates the limited use of color names with the limited perception of colors. Finally, Gladstone suggests improvements that are acquired through training are something that can be passed on to offspring. Children inherit these improvements, mitigating the need for years of practice (option D). Hint #3 Thus, option A is correct. 11. Homer’s sky is starry, or broad, or great, or iron, or violet; but it is never blue. How does this affect the opinions expressed in the passage? I. It supports Gladstone’s claim regarding Homer’s use of color. II. It extends Gladstone’s claim regarding Homer’s focus on nature.

104


MCAT Practice Test 1 EXPLAINED.pages

III. It challenges Gladstone’s claim regarding Homer’s penchant for strange imagery. Please choose from one of the following options. 1 *I only 2 II only 3 I and III only 4 II and III only This is a “Reasoning Beyond the Text” question, which means that it wants you to either apply or extrapolate the ideas in the passage to new situations or to assess how new information would impact the ideas presented in the passage. It is important to understand the assumptions underlying the article, and how new information may or may not shift the central thesis. Hint #2 Gladstone describes Homer’s use of descriptions of color as limited, and notes that it was unlikely that people in Homer’s generation experienced the color blue frequently. The example showing the kinds of words that Homer uses to describe the sky, supports Gladstone’s claim regarding Homer’s limited use of color (statement I). The author does not discuss any claims made by Gladstone about Homer’s focus on nature (statement II). Gladstone suggests that Homer’s similes are rich with “sensible imagery”, not strange imagery (statement III). Hint #3 Thus, only statement I is correct, corresponding to option A. In the United Kingdom, Physical Education (PE) is compulsory in state schools until students reach the age of 16. That is, sports are compulsory for as long as formal education is mandated by law. Because there are many children who don’t want to participate in PE classes, I believe that students should be allowed a choice. If their parents agree, why should they be forced to jump on a trampoline or do calisthenics? PE class is different from other classes because it involves what one does with one’s body. We acknowledge the right of individuals to control their own bodies— to determine whether and when they have an operation, to determine where they go and what they do. Why is this any different? It is a red herring to say that PE makes any serious difference to people’s health. There are more effective ways of ensuring a healthy population than

105


MCAT Practice Test 1 EXPLAINED.pages

pushing children to run laps around a freezing sports field once a week. For example, schools could be addressing the poor diets young people have today and encouraging them to walk or bicycle to school rather than rely on the car. Furthermore, sports are a waste of school time and resources. One or two PE lessons a week make very little difference to an individual’s health, but they make a huge difference in a school’s budget. Mandatory PE requires a whole extra department in schools, wasting a great deal of money and time that could be better spent on academics. It also requires schools to be surrounded by a large amount of land for playing fields, making it prohibitively expensive to build new schools in urban areas. Given the average current pupil–teacher ratios, the quality of teaching in PE classes is necessarily low, and the classes may even be dangerous to students who are not properly supervised. Our children are burdened enough in schools already, especially at the older end of the system, with multiple examinations. PE simply adds, needlessly, to this hectic schedule. Many people argue that playing team sports builds character, encourages students to work with others, teaches children how to win and lose with good grace, and builds strong school spirit through competition with other institutions. It is often, they say, the experience of playing on a team together which builds the strongest friendships at school, friendships which endure for years afterwards. Many say the same benefits derive from the common endurance of prison. Injuries sustained through school sport and the psychological trauma of being bullied for sporting ineptitude can mark people for years after they have left school. On that note, in an increasingly litigious age, a compulsory rather than voluntary sports program is a liability. More and more schools are avoiding team games such as rugby, soccer, hockey, and football due to the realistic fear of lawsuits. Teamwork can be better developed through music, drama, and community projects without the need to encourage an ultra-competitive ethos. As for the argument that without compulsory PE, many members of society wouldn’t find out that they had a talent for a sport or even that they enjoyed it, students can discover this aptitude outside of school, without also

106


MCAT Practice Test 1 EXPLAINED.pages

discovering the bullying and humiliation that comes with PE classes more than with other lessons. The aim of compulsory PE isn’t being fulfilled at present in any case, as “sick notes” are produced with alarming regularity by parents complicit in their children’s wish to avoid it. Greater efforts to enforce it will only result in more deceit, children missing school for the entire day, or, in the most extreme cases, children being withdrawn from state education. Adapted from A. Deane, “Physical Education, Compulsory,” Creative Commons. ©2011 Creative Commons. 12. What is the function of the statement in the first paragraph that “PE class is different from other classes”? Please choose from one of the following options. 1 It is part of an argument why PE classes should be required. 2 It is part of an argument that PE classes improve people’s health. 3 It explains why students should only be exempt from PE with parental permission. 4 *It explains why students should have a choice about whether to take PE while not having a choice about taking other compulsory classes. This is a “Reasoning Within the Text” question, which means that it wants you to explore the key idea, claim, or theme that is being expressed in a passage. You will have to evaluate the soundness of an argument, the reasonableness of its conclusions, and/or the appropriateness of its generalizations. It is crucial to carefully examine the strengths and weaknesses of the key ideas based on the information given. Hint #2 The author does not state that “PE class is different” as a reason why it should be mandatory (option A). On the contrary, the author is asserting that students should have a choice because PE class is different. The passage states, “I believe that students should be allowed a choice…PE class is different from other classes because it involves what one does with one’s body” (paragraph 1). The author frames a philosophical question of choice around physical education, but doesn’t believe that this choice extends to other classes that do not primarily affect the human body (option D). The author dismisses the claim that PE makes any serious difference to

107


MCAT Practice Test 1 EXPLAINED.pages

people’s health as a “red herring”, and he does not use the statement that PE class is different as part of this argument (option B). The author is not using the statement to argue why parental permission might be necessary (option C). Hint #3 Thus, option D is correct.

13. Which of the following assumptions is made by the author in relation to the argument about students’ hectic schedules? Please choose from one of the following options. 1 PE tends not to have a final examination. 2 PE tends not to have a heavy homework burden. 3 *Compulsory PE, if eliminated, would not be replaced by another compulsory course. 4 It is unfair to require students in the higher grades to prepare for multiple examinations. This is a “Reasoning Beyond the Text” question, which means that it wants you to either apply or extrapolate the ideas in the passage to new situations or to assess how new information would impact the ideas presented in the

108


MCAT Practice Test 1 EXPLAINED.pages

passage. It is important to understand the assumptions underlying the article, and how new information may or may not shift the central thesis. Hint #2 The author describes the busy schedules of students, and argues for elimination of PE to help alleviate that burden. This would only be true if PE was not replaced by another compulsory course with a similar amount of work involved (option C). The fact that PE tends not to have a final examination (option A) and does not have heavy homework burden (option B), suggest that PE might be relatively less likely to cause additional stress for a student relative to another class. These are not part of the author’s assumptions. The point about requiring students in the higher grades to prepare for multiple examinations (option D) is also not addressed or assumed by the author. Hint #3 Thus, option C is correct.

14. Assume as true that students are more likely to obtain specialist coaching at sports clubs outside of school than in school. How would this information be relevant to the passage? Please choose from one of the following options. 1 It would restate an objection to compulsory physical education classes. 2 *It would support a point about discovering sports aptitude made in rebuttal. 3 It would directly challenge one of the author’s claims. 4 It would contradict one of the author’s examples. Hint #1 This is a “Reasoning Beyond the Text” question, which means that it wants you to either apply or extrapolate the ideas in the passage to new situations

109


MCAT Practice Test 1 EXPLAINED.pages

or to assess how new information would impact the ideas presented in the passage. It is important to understand the assumptions underlying the article, and how new information may or may not shift the central thesis. Hint #2 The author writes, “As for the argument that without compulsory PE, many members of society wouldn’t find out that they had a talent for a sport or even that they enjoyed it, students can discover this aptitude outside of school, without also discovering the bullying and humiliation that comes with PE classes more than with other lessons” (paragraph 6). The author is offering a rebuttal to the argument that students have no alternative to finding a talent for sports in school settings. If it is true that students are obtaining specialist coaching at sports clubs outside of school more often than in school, then it would further support this point that was made in rebuttal. The fact would be more than a simple restatement of an objection to compulsory physical education classes (option A), because it would actually serve as a clear example that supports a point that is already made. This fact would not challenge any of the author’s claims (option C) or contradict any of the author’s examples (option D). Hint #3 Thus, option B is correct.

15. The author’s central theme for the whole passage is: Please choose from one of the following options. 1 opposing formal educational mandates. 2 describing the consequences of making PE compulsory. 3 *presenting reasons for why PE should not be compulsory. 4 advocating that PE be abolished in UK state schools. Hint #1 This is a “Foundations of Comprehension” question, which means that it wants you to understand the central theme or idea and be able to figure out what specific words or phrases mean within that context. It is important to identify specific parts of the passage such as the thesis, example, and counterexample.

110


MCAT Practice Test 1 EXPLAINED.pages

Hint #2 The author states that current law in the United Kingdom makes PE compulsory in state schools for students under age 16, and then argues in the third sentence that “students should be allowed a choice.” The author describes the right of individuals to control their own bodies in other settings, in order to argue for an exception from mandates for PE classes. His main point is not opposing educational mandates or compulsory education in general, but only in the instance of PE (option A). Although the author describes a number of possible negative consequences of PE, this is not his central point (option B). This is just one component of the larger argument that PE should not be compulsory (option C). Finally, the author argues that sports are a waste of time and money. Many points made in the third paragraph could be used to suggest eliminating both PE and sports altogether, however the author never explicitly advocates fully abolishing PE in UK schools (option D), rather the central theme that underlies all of the claims made in the passage is that PE should not be compulsory. Hint #3 Thus, option C is correct.

Although business people deserve more respect for their honesty than they receive, a common complaint is that they take advantage of consumers through dishonest advertising. Instead of providing useful information for making rational choices, advertisements often appeal to consumers’ emotions to persuade them to buy products regardless of need. This complaint is true and obvious to all but the most naïve people. Advertisements are designed to convince consumers to favor one product over others, and presenting solely unbiased and unemotional information would seldom be the best way to accomplish this goal.

111


MCAT Practice Test 1 EXPLAINED.pages

Thoughtful people recognize that politicians advertise themselves and their policy recommendations in similarly biased and emotional ways. The question is not whether businesspeople or politicians have the strongest moral commitment to truthfulness in advertising. Both groups will deviate from honest practices when they expect that the benefits of doing so will exceed the costs. The important question is “Who can most easily mislead their customers with emotional statements, unrealistic promises, and biased information: businesspeople or politicians?” People are less likely to be swayed by dishonesty and emotion when responding to business ads than when responding to political ads for two reasons. First, businesspeople are attempting to persuade people who are usually spending their own money; politicians are trying to persuade people who are deciding how they want to spend other people’s money. The motivation to minimize mistakes by carefully considering claims about costs and benefits before a decision is made and by evaluating those claims in light of post-decision experience is greater when one is bearing all of the cost of the decision than when others are bearing most of the cost. The second reason why misleading claims are less effective in promoting commercial products than in promoting political products is because the choices that consumers of commercial products make have more decisive effects on outcomes than do the choices of consumers of political products. When people purchase a product in the marketplace, they get the product they choose, and they get it because they chose it. The probability that a voter’s choice will be decisive is increasingly small in state and federal elections, and seldom greater than a fraction of one percent in most local elections. Given such a low probability of any one person’s vote determining the outcome of the election, voters have little motivation to be concerned about the accuracy of the political claims being made. One might think that professors would be more honest than both businesspeople and politicians when promoting their products’ value (that is, in their teaching and research). Unlike politicians, professors try to sell their products to customers who can decisively accept or reject them without being directly affected by how many others make different choices. However, many undergraduate students are glaringly indifferent to what

112


MCAT Practice Test 1 EXPLAINED.pages

professors have to say, so professors have more latitude than businesspeople to benefit from exaggerated and duplicitous claims. Professors have to be more restrained when publishing than when teaching because other professors will evaluate the truth of their published claims. It is true that academic promotions may be earned and scholarly reputations enhanced by exposing the errors in published work. However, professors are often less concerned with the truthfulness of articles written by other professors than one might think. Professors anxious to get their own articles and books published are often less interested in whether the publications they cite are correct than in whether the publications are accepted as correct by academics with views similar to their own—the people most likely to decide whether their books and articles will be published and cited. Adapted from D. Lee, “Why Businessmen Are More Honest than Preachers, Politicians, and Professors.” ©2010 The Independent Review. 16. Which of the following assumptions is most central to the author’s argument? Please choose from one of the following options. 1 Most products are designed to appeal to naïve and emotional consumers. 2 Products are more likely to be purchased when they are advertised than when they are not. 3 If businesspeople manufactured only products that people need, there would be few products on the market. 4 *If products were evaluated according to objective information about them, people would often not prefer one over the other. This is a “Reasoning Within the Text” question, which means that it wants you to explore the key assumption(s), claim(s), or theme that is being expressed in a passage. You will have to evaluate the soundness of an argument, the reasonableness of its conclusions, and/or the appropriateness of its generalizations. It is crucial to carefully examine the strengths and weaknesses of the key ideas based on the information given. Hint #2

113


MCAT Practice Test 1 EXPLAINED.pages

In the second sentence the author writes “Instead of providing useful information for making rational choices, advertisements often appeal to consumers’ emotions to persuade them to buy products regardless of need.” Although the author suggest that advertisements appeal to consumers’ emotions, the passage does not discuss products being designed to appeal to emotions (option A). The passage does not draw any conclusions based on whether products are more likely to be purchased when they are advertised over when they are not advertised (option B). The author writes that the complaint of being convinced to buy things that are not needed is “true and obvious to all but the most naïve people” (paragraph 1). Although this implies that some products may not be needed, the author does not suggest that most products are not needed (option C). The author writes that “Advertisements are designed to convince consumers to favor one product over others, and presenting solely unbiased and unemotional information would seldom be the best way to accomplish this goal” (paragraph 1). If the goal of advertising creates a bias for one product over another, then this sentence suggests that in the absence of advertising people would rely on objective information which would not result in any preference, which is consistent with option D. Hint #3 Thus, option D is correct.

17. The author implies which of the following about businesspeople and politicians? Please choose from one of the following options. 1 Neither are very thoughtful people. 2 *Neither have a strong moral commitment to truthfulness. 3 Both have biased views about their customers and constituents, respectively.

114


MCAT Practice Test 1 EXPLAINED.pages

4

Both are more concerned about advertising themselves than their products, respectively. Hint #1 his is a “Foundations of Comprehension” question, which means that it wants you to be able to figure out what specific words or phrases mean within that context. Questions may require you to infer meanings that can’t be determined from a superficial reading of the text, such as meanings that the author has implied but did not state directly. Hint #2 In the second paragraph, the author states “Thoughtful people recognize that politicians advertise themselves and their policy recommendations in similarly biased and emotional ways.” The passage does not imply that either business people or politicians are not thoughtful (option A), On the contrary, their choices seem to be quite deliberate and thoughtful. In the second paragraph, the author also says “The question is not whether businesspeople or politicians have the strongest moral commitment to truthfulness in advertising. Both groups will deviate from honest practices when they expect that the benefits of doing so will exceed the costs.” Which implies that although he does not take a position on which group has less commitment to truthfulness, that both groups deviate from honest practices. This implies that both groups do not have a strong moral commitment to truthfulness (option B). The passage does not discuss whether businesspeople or politicians are biased in their views of their customers or constituents. The author’s main point concerning bias is that the messages produced by businesspeople and politicians are biased (option C). Finally, while politicians may be more more likely to advertise themselves, the passage describes businesspeople as generally trying to advertise their products. This is a point of difference rather than similarity between the two groups (option D). Hint #3 Thus, option B is correct. 18. Suppose a politician is re-elected despite lying about his voting record . The passage suggests which of the following explanations? Please choose from one of the following options. 1 The politician made many contradictory statements during his or her campaign.

115


MCAT Practice Test 1 EXPLAINED.pages

2

Voter demographics for the second election were significantly different than for the first. 3 *Voters did not compare the politician’s behavior while in office with statements made during his or her campaign. 4 There was no consensus among voters regarding the costs and benefits of a second term in office for that politician. Hint #1 This is a “Reasoning Beyond the Text” question, which means that it wants you to either apply or extrapolate the ideas in the passage to new situations or to assess how new information would impact the ideas presented in the passage. It is important to understand the assumptions underlying the article, and how new information may or may not shift the central thesis. Hint #2 The author states that, “The motivation to minimize mistakes by carefully considering claims about costs and benefits before a decision is made and by evaluating those claims in light of post-decision experience is greater when one is bearing all of the cost of the decision than when others are bearing most of the cost” (paragraph 3). This is one reason that individuals are more willing to carefully evaluate the claims of a business person over the claims of a politician. A second reason, given in paragraph 4 is “Given such a low probability of any one person’s vote determining the outcome of the election, voters have little motivation to be concerned about the accuracy of the political claims being made.” If a politician is able to get reelected even though he/she lied about their voting record, then the passage suggests that the likely reason is that voters were not motivated to evaluate campaign claims in light of observed political behavior during the first term in office. The passage doesn’t suggest that the presence of contradictory statements as a reason why people are less likely to be swayed by dishonesty and emotion when responding to business ads than when responding to political ads. (option A). It is possible that a new cohort of voters elected the politician in the second term, however this alternative is not discussed by the passage (option B). The notion of costs and benefits that is mentioned in the passage refers to the costs and benefits of engaging in a careful evaluation process. The lack of consensus in cost and benefits analyses is not discussed as a reason why misleading claims are less effective in promoting commercial products than in promoting political products (option D).

116


MCAT Practice Test 1 EXPLAINED.pages

Hint #3 Thus, option C is correct. 19. The author most likely mentions probability in his discussion of voting behavior as reasoning for which of the following (paragraph 4)? I. To explain low voter turnout in state and federal elections II. To explain the prevalence of politicians’ dishonesty III. To explain why voters do not carefully consider political claims Please choose from one of the following options. 1 II only 2 III only 3 I and II only 4 I and III only Hint #1 This is a “Reasoning Within the Text” question, which means that it wants you to explore the key idea, claim, or theme that is being expressed in a passage. You will have to evaluate the soundness of an argument, the reasonableness of its conclusions, and/or the appropriateness of its generalizations. It is crucial to carefully examine the strengths and weaknesses of the key ideas based on the information given. Hint #2 The author writes that, “The probability that a voter’s choice will be decisive is increasingly small in state and federal elections, and seldom greater than a fraction of one percent in most local elections. Given such a low probability of any one person’s vote determining the outcome of the election, voters have little motivation to be concerned about the accuracy of the political claims being made” (paragraph 4). The individual voter has a very small impact on the overall number of votes and therefore each voter has only a small stake in the overall decision. The distributed sense of ownership is used to explain why voters do not carefully consider political claims, which is consistent with Statement III. The reference to probability appears as part of the fourth paragraph, and the author states in the first sentence of that paragraph that he is attempting to explain “why misleading claims are less effective in promoting commercial products than in promoting political products.” The author is not trying to explain low voter turnout. This is contrary to statement I. Although it is implied that politicians

117


MCAT Practice Test 1 EXPLAINED.pages

are dishonest in the sentence “Both groups will deviate from honest practices when they expect that the benefits of doing so will exceed the costs”, the author is not attempting to explain the prevalence of politicians’ dishonesty by discussing the probability. The first sentence in paragraph 4 sets up that the main behavior that the author is trying to explain is “why misleading claims are less effective in promoting commercial products than in promoting political products.” which is about voter reaction to the claims, not about the dishonesty of politicians (statement II). Hint #3 Thus, statement III is the only correct one, corresponding to option B.

Imagine that someone offers you and some other anonymous person $100 to share. The rules are strict and known to both players. The two of you are

118


MCAT Practice Test 1 EXPLAINED.pages

in separate rooms and cannot exchange information. A coin toss decides which of you will propose how to share the money. If you are the proposer you can make a single offer of how to split the sum, and the other person— the responder—can say yes or no. If the responder’s answer is yes, the deal goes ahead. If the answer is no, neither of you gets anything. In both cases, the game is over and will not be repeated. What will you do? Instinctively, many people feel they should offer 50 percent, because such a division is “fair” and therefore likely to be accepted. More daring people, however, think they might get away with offering somewhat less than half of the sum. You may not be surprised to learn that two-thirds of the offers are between 40 and 50 percent. Only four in 100 people offer less than 20 percent. Proposing such a small amount is risky, because it might be rejected. More than half of all responders reject offers that are less than 20 percent. But why should anyone reject an offer as “too small”? The responder has just two choices: take what is offered or receive nothing. The only rational option for a selfish individual is to accept any offer. A selfish proposer who is sure that the responder is also selfish will therefore make the smallest possible offer and keep the rest. This game-theory analysis, which assumes that people are selfish and rational, tells you that the proposer should offer the smallest possible share and the responder should accept it. But this is not how most people play the game. The scenario just described, called the Ultimatum Game, was devised some twenty years ago. Experimenters subsequently studied the Ultimatum Game intensively in many places using diverse sums. The results proved remarkably robust. Behavior in the game did not appreciably depend on the players’ sex, age, schooling, or numeracy. Moreover, the amount of money involved had surprisingly little effect on results. Yet the range of players remained limited, because the studies primarily involved people in more developed countries and often university students. Recently, a cross-cultural study in fifteen small-scale societies showed that there were sizable differences in the way some people play the Ultimatum Game. Within the Machiguenga tribe (from the Amazon) the mean offer

119


MCAT Practice Test 1 EXPLAINED.pages

was considerably lower than in typical Western-type civilizations—26 percent instead of 45 percent. Conversely, many members of the Au tribe (from Papua New Guinea) offered more than 50 percent. Cultural traditions in gift giving, and the strong obligations that result from accepting a gift, play a major role among some tribes, such as the Au. Yet despite these cultural variations, the outcome was always far from what rational analysis would dictate for selfish players. Most people all over the world place a high value on fair outcomes. For a long time, theoretical economists postulated a being called Homo economicus—a rational individual relentlessly bent on maximizing a purely selfish reward. But the lesson from the Ultimatum Game and similar experiments is that real people are a crossbreed of H. economicus and H. emoticus, a complicated hybrid species that can be ruled as much by emotion as by cold logic and selfishness. An interesting challenge is to understand how Darwinian evolution would produce creatures instilled with emotions and behaviors that do not immediately seem geared toward reaping the greatest benefit for individuals or their genes. Adapted from K. Sigmund, E. Fehr, and M.A. Nowak, “The economics of fair play.” ©2002 by Scientific American, Inc. 20.Which of the following statements is NOT as strongly supported by the passage? Please choose from one of the following options. 1 The rules of the Ultimatum Game are strict. 2 The results of the Ultimatum Game tend to be consistent. 3 *Responders reject offers that are less than 20 percent because they consider such offers unfair. 4 Studies of the Ultimatum Game show sizable differences in the way some people play. This is a “Reasoning Within the Text” question, which means that it wants you to explore the key idea, claim, or theme that is being expressed in a passage. You will have to evaluate the soundness of an argument, the reasonableness of its conclusions, and/or the appropriateness of its

120


MCAT Practice Test 1 EXPLAINED.pages

generalizations. It is crucial to carefully examine the strengths and weaknesses of the key ideas based on the information given. Hint #2 The second sentence explicitly states that “The rules are strict …” (option A). According to the third paragraph, the game has been studied “… intensively in many places using diverse sums. The results proved remarkably robust. Behavior did not appreciably depend on the players’ sex, age, schooling, or numeracy. Moreover, the amount of money had surprisingly little effect…” This evidence supports the claim that the results of the Ultimate Game tend to be consistent (option B). The first sentence in paragraph 5 explicitly states that a recent study of the Ultimate Game showed that there were “sizable differences in the way some people play the Ultimatum Game.” (option D). There is a statement about people who “feel they should offer 50 percent because such a division is ‘fair’”, however, the passage provides no analysis of the decision-making process involved in responders. The passage is focusing on the decision-making process of the proposers. Hint #3 Thus, option C is correct.

121


MCAT Practice Test 1 EXPLAINED.pages

21. In some trials of the Ultimatum Game, the proposed split is determined by a computer. When responders are aware of this, they are willing to accept considerably lower offers. Based on the passage, compared to the standard game played without a computer, these responses are more: Please choose from one of the following options. 1 *in keeping with what rational analysis would dictate. 2 out of keeping with what rational analysis would dictate, 3 in keeping with what one would expect from Homo emoticus. 4 in keeping with what researchers would expect of the responses from people of non-Western cultures. Hint #1 This is a “Reasoning Beyond the Text” question, which means that it wants you to either apply or extrapolate the ideas in the passage to new situations or to assess how new information would impact the ideas presented in the passage. It is important to understand the assumptions underlying the article, and how new information may or may not shift the central thesis. Hint #2 Two of the response options (options A and B) are concerned with what rational analysis would dictate for responders. According to the third paragraph: “The responder has just two choices: take what is offered or receive nothing. The only rational option for a selfish individual is to accept any offer… This game-theory analysis, which assumes that people are selfish and rational, tells you that the proposer should offer the smallest possible share and the responder should accept it.” When a computer determines the proposed split, we learn that responders are willing to accept considerably lower offers. Therefore, the responders are acting more in keeping with what rational analysis would dictate because they are more likely accept any offer (option A). In contrast, the responders are out of keeping with what one would expect from Homo emoticus, which is a term used to reflect emotional thinking in paragraph 6 to explain why people might not act in a purely selfish manner (option C). Since the two studies done in small-scale societies did not show consistent results, and the 15 studies showed sizeable differences, there is not a clear expectation set up for non-Western cultures (assuming this term is being used as a

122


MCAT Practice Test 1 EXPLAINED.pages

synonym for small-scale societies). Since there is no expectation, a new finding cannot be either consistent or inconsistent with it (option D). Hint #3 Thus, option A is correct. 22. Assume a “fair” offer is defined as 50 percent, and responders behave rationally. Based on the discussion in paragraph 5, it can be reasonably assumed that the author believes that the Machiguenga tribe had a: Please choose from one of the following options. 1 lower percentage of fair offers than both typical Western societies and rational selfish players. 2 higher percentage of fair offers than both typical Western societies, and rational selfish players. 3 *lower percentage of fair offers than typical Western societies, and a higher percentage of fair offers than rational selfish players. 4 higher percentage of fair offers than typical Western societies, and a lower percentage of fair offers than rational selfish players. Hint #1 This is a “Reasoning Beyond the Text” question, which means that it wants you to either apply or extrapolate the ideas in the passage to new situations or to assess how new information would impact the ideas presented in the passage. It is important to understand the assumptions underlying the article, and how new information may or may not shift the central thesis. Hint #2 All responses make an assertion about the percentage of fair offers compared to typical Western societies. The passage states that among the Machiguenga tribe, the “mean offer was considerably lower than in typical Western-type civilizations—26 percent instead of 45 percent.” (Paragraph 5) Since the mean (or average) Machiguenga tribe offer is 26 percent, it seems very unlikely that there would be a higher percentage of individual offers above 50 percent as compared to typical Western societies with a mean offer of 45 percent. This excludes options B and D. The second part of all responses make an assertion about the percentage of fair offers that would be made by rational selfish players. This requires determining what that expected percentage might be from the text. Based on game-theory analysis (which assumes that people are selfish and rational), the rational selfish player should propose the smallest possible offer and the responder should accept this offer. This is based on the fact the responder has only

123


MCAT Practice Test 1 EXPLAINED.pages

two choices: take what is offered or receive nothing. This means that the rational selfish player should propose an offer of 1% (or 0.01% which would be one cent), and never offer a “fair” offer of 50%. The Machiguenga tribe had a mean offer of 26%, which is higher than the 1% we might expect for rational selfish players. This excludes options A and D. Hint #3 Thus, option C is correct. 23. In the discussion of the Ultimatum Game, what is the significance of the statement that “the range of players remained limited” (paragraph 4)? Please choose from one of the following options. 1 *The limited sample did not allow the experimenters to generalize about all people. 2 Limiting the range of players allowed the experimenters to better control the outcome of the game. 3 Limitations on the game led to mistaken conclusions by experimenters at that time. 4 The limited sample allowed the experimenters to better control the range of variables in the experiment. Hint #1 This is a “Foundations of Comprehension” question, which means that it wants you to interpret the meaning of a particular statement made by the author. Hint #2 The full statement reads “Yet the range of players remained limited, because the studies primarily involved people in more developed countries and often university students.” (paragraph 4) The cause for the limited range of players is explained by the fact that they came from specific countries and educational settings. The following paragraph describes a cross-cultural study that showed “sizable differences in the way some people play,” (paragraph 5) demonstrating that the previous body of literature could not be generalized to all people. There is no evidence the experimenters wanted to limit the range of players to better control the game (option B) or range of variables (option D) or that there were any limitations on the game itself that led to mistaken conclusions (option C). Hint #3

124


MCAT Practice Test 1 EXPLAINED.pages

Thus, option A is correct.

The tools we use to think change the ways in which we think. The invention of written language brought about a radical shift in how we process, organize, store, and transmit representations of the world. Although writing remains our primary information technology, today when we think about the impact of technology on our habits of mind, we think primarily of the computer. My first encounters with how computers change the way we think came soon after I joined the faculty at the Massachusetts Institute of Technology at the end of the era of the slide rule and the beginning of the era of the personal computer. At a lunch for new faculty members, several senior professors in engineering complained that the transition from slide rules to calculators had affected their students’ ability to deal with issues of scale. When students used slide rules, they had to insert decimal points themselves. The professors insisted that doing that required students to maintain a mental sense of scale, whereas those who relied on calculators made frequent errors in orders of magnitude. Additionally, the students with calculators had lost their ability to do “back of the envelope” calculations, and with that, an intuitive feel for the material. That same semester, I taught a course in the history of psychology. There, I experienced the impact of computational objects on students’ ideas about their emotional lives. My class had read Freud’s essay on slips of the tongue, with its famous first example: The chair of a parliamentary session opens a meeting by declaring it closed. The students discussed how Freud interpreted such errors as revealing a person’s mixed emotions. A computer science major disagreed with Freud’s approach. The mind, she argued, is a computer. And in a computational dictionary—like we have in the human mind—closed and open are designated by the same symbol, separated by a sign for opposition. Closed equals minus open. To substitute closed for open does not require the notion of ambivalence or conflict. “When the chairman made that substitution,” she declared, “a bit was dropped; a minus sign was lost. There was a power surge. No

125


MCAT Practice Test 1 EXPLAINED.pages

problem.â€? The young woman turned a Freudian slip into an informationprocessing error. An explanation in terms of meaning had become an explanation in terms of mechanism. Today, starting in elementary school, students use e-mail, word processing, computer simulations, and virtual communities. In the process, they are absorbing more than the content of what appears on their screens. They are learning new ways to think about what it means to know and understand. There are a number of areas where I see information technology encouraging changes in thinking. There can be no simple way of cataloging whether any particular change is good or bad. That is contested terrain. At every step we have to ask, as educators and citizens, whether current technology is leading us in directions that serve our human purposes. Such questions are not technical; they are social, moral, and political. For me, addressing that subjective side of computation is one of the more significant challenges for the next decade of information technology in higher education. Technology does not determine change, but it encourages us to take certain directions. If we make those directions clear, we can more easily exert human choice. Adapted from S. Turkle, How computers change the way we think. Š2004 by The Chronicle of Higher Education. 24. Based on the passage, the author most likely believes that it is important to understand the influence computers have on people because such understanding will: Please choose from one of the following options. 1 *enable people to make computers serve human purposes. 2 increase the importance of information technology in the next decade. 3 improve people’s ability to deal with issues of scale. 4 help prove that the human mind is a computational object.

126


MCAT Practice Test 1 EXPLAINED.pages

This is a “Foundations of Comprehension” question, which means that it wants you to understand the author’s central theme or ideas or point-ofview. Hint #2 In the last paragraph the author asserts “At every step we have to ask, as educators and citizens, whether current technology is leading us in directions that serve our human purposes” The author then closes with the statement, “Technology does not determine change, but it encourages us to take certain directions. If we make those directions clear, we can more easily exert human choice” (paragraph 5). Her premise is that articulating and understanding the effect that tools have on our way of thinking helps us to better optimize those tools to serve our needs (option A). The passage does not offer any conjecture about how this understanding might change the importance of technology in the next decade (option B). Although the example of how using a slide rule might have led to better understanding of scale than using a calculator is included (option C), this is not offered as a reason to better understand the influence of computers. The author cites an example of a student comparing the human mind to a computer, but the key idea is to show how learning about a tool can shape how we think about something. The author doesn’t discuss the desire to test theories about the human mind as a computational object as the main reason to better understand the influence that computers have on people (option D). Hint #3 Thus, option A is correct. 25. Which of the following passage assertions is presented as evidence that computers are affecting people’s conception of the mind? Please choose from one of the following options. 1 Engineering students using calculators frequently make mistakes regarding orders of magnitude. 2 Students who used calculators lost their ability to do “back of the envelope” calculations. 3 *A computer science major interpreted a Freudian slip as an information-processing error.

127


MCAT Practice Test 1 EXPLAINED.pages

4

Addressing the subjective side of computation is a significant challenge for the next decade of information technology in higher education. Hint #1 This is a “Reasoning Within the Text” question, which means that it wants you to explore the key idea, claim, or theme that is being expressed in a passage. You will have to evaluate the soundness of an argument, the reasonableness of its conclusions, and/or the appropriateness of its generalizations. It is crucial to carefully examine the strengths and weaknesses of the key ideas based on the information given. Hint #2 The author presents the examples of engineering students making more mistakes of scale when using calculators (option A), and losing their intuitive sense for calculation (option B) to characterize the loss of particular skill sets. These examples do not provide evidence that computers are affecting people’s thinking about the mind. The author uses the example of a computer science major who interpreted a Freudian slip as an information-processing error as evidence that computers are affecting people’s conception of the mind. The student said, “a bit was dropped; a minus sign was lost. There was a power surge. No problem” (paragraph 3). This offers an alternative to the Freudian paradigm of the mind, and thus serves as evidence that computers are offering another metaphor for understanding the of the human mind. The passage does mention retention that evaluating the impact of technology on thinking will be a challenge in higher education (option D), but this is not related to the idea that computers may be changing people’s conception of the mind. Hint #3 Thus, option C is correct. 26. Of the following scenarios, which represents an example most similar to what the author probably means by the opening statement, “The tools we use to think change the ways in which we think”? Please choose from one of the following options. 1 After a power outage, a person creates a plan for coping with such events in the future. 2 An analysis of the sequences of clicks emitted by dolphins reveals structural similarity to aspects of human language.

128


MCAT Practice Test 1 EXPLAINED.pages

3

A person gains a new appreciation for abstract painting after learning about a new theory of complementary colors. 4 An office manager increases productivity by installing new accounting software on the company’s computers. Hint #1 This is a “Reasoning Beyond the Text” question, which means that it wants you to either apply or extrapolate the ideas in the passage to new situations or to assess how new information would impact the ideas presented in the passage. It is important to understand the assumptions underlying the article, and how new information may or may not shift the central thesis. Hint #2 Having an event like a power outage (option A), or recognizing a similar pattern in communication systems across two species (option B) do not change the way we think by providing humans with fundamental new tools. Although new accounting software represents a new tool (option D), any benefit of increased productivity may be a direct effect from the software, and may not necessarily reflect fundamental changes in the thinking of the humans who use it. However, exposure to a new way of thinking about complimentary colors can change the way that an abstract painting is appreciated. In this case, the new theory can be seen as the tool that alters perception and thought about the painting (option C). Hint #3 Thus, option C is correct. 27. Which of the following passage assertions is most supported in the passage by evidence or examples? Please choose from one of the following options. 1 *Computational objects have an impact on students’ ideas about their emotional lives. 2 When students used slide rules, they had to insert decimal points themselves. 3 Students who used calculators lost their ability to do “back of the envelope” calculations. 4 The invention of written language brought about a radical shift in how we process, organize, store, and transmit representations of the world. Hint #1

129


MCAT Practice Test 1 EXPLAINED.pages

This is a “Reasoning Within the Text” question, which means that it wants you to explore the key idea, claim, or theme that is being expressed in a passage. You will have to evaluate the soundness of an argument, the reasonableness of its conclusions, and/or the appropriateness of its generalizations. It is crucial to carefully examine the strengths and weaknesses of the key ideas based on the information given. Hint #2 In the third paragraph, the author asserts that computational objects have an impact on student’s ideas about their emotional lives, and offers evidence by including the example of a computer science major in her class who viewed Freudian slips as an information processing error, rather than as the consequence of repressed emotions (option A). The potential downsides to calculator use she cites from past conversations with colleagues are anecdotal examples, not assertions that are further supported by evidence (options B and C). The author does assert that the invention of writing systems brought about radical changes in thinking in the first paragraph, but no evidence is offered for this statement (option D). Hint #3 Thus, option A is correct.

130


MCAT Practice Test 1 EXPLAINED.pages

Deconstructionism, as applied to literary criticism, is a paradox about a paradox: It assumes that all discourse, even all historical narrative, is essentially disguised self-revelatory messages. Being subjective, the text has no fixed meaning, so when we read, we are prone to misread. Deconstructionism emerged from Paris and, notwithstanding its claim to universality, has an evident history. It is a manifestation of existential anxieties about presence and absence, reality and appearance. It developed via structuralism, with its emphasis on semantics and symbolism. From these sources it derived its fundamental premise: the endless slippage of the subject, the futility of any attempt to name reality. The premise suggests the disillusion attendant on the collapse of the two major forces in twentieth-century European thought: enlightened humanism and idealistic Marxism. Despite its origins, deconstructionism found its own best home in the United States, that historically dissociated construction of random meanings. (“America is deconstruction,” said its leading proponent, Jacques Derrida.) By the 1970s, deconstruction filled—perhaps better, emptied—the gap left in the humanities in the U.S. by the demise of the old “new criticism.” But what began as brilliant and creative analytic performances soon became classroom pedagogy. Throughout the decade, the seminar rooms on U.S. campuses—and then campuses worldwide—became workshops in deconstructionist practice. Junior misreaders worked away, becoming ever more like C.I.A. operatives, decoding false signals sent by a distant enemy, the writer.

131


MCAT Practice Test 1 EXPLAINED.pages

Deconstruction exalted itself with ever higher pretensions. As one academic critic exulted, “The history of literature is part of the history of criticism.� Deconstruction transformed everything into social commentary, easily making affinities with sexual and racial politics, two other militant philosophies that challenge the sanctity of text. It presented itself as a supra-ideological mode of analysis, exposing the ideological aberrations of others while seemingly possessing none itself. Any resistance that deconstructionists encountered was usually interpreted as censorious ignorance. As their approach prevailed, gangs of neodeconstructionists descended on the library with their critical services. One would demythologize, another decanonize, another dephallicize, another dehegemonize, another defame. Literature, as the deconstructionists frequently proved, had been written by entirely the wrong people for entirely the wrong reasons. Soon all that would be left of it would be a few bare bones of undecidable discourse and some tattered leather bindings. This frenzy would be called a conference of the Modern Language Association. The point that needs to be reaffirmed is that writing is an existential act, an imaginative exploration of ideas. It is, in fact, an expression of moral responsibility. Literature is not a subordinate category of social criticism. When writers are censored, imprisoned, killed, or threatened with death for their writings, it is not because their discourse is undecidable. If we are to take authors and their fate seriously, we must recognize that fiction is more than an opportunity for word games; we must honor it as a mode of radical discovery. We need an ambiance around writing that affirms its nature as creativity, as art, and that in a larger sense considers creativity a prime power in the making of intelligence, feeling, and morality. This was the position from which Jean-Paul Sartre with his freedom-affirming existentialism started the postwar debate of which deconstruction is a latter-day development. He started it because during the 1930s the word had been defamed and disfigured, the book burned, the writer erased, by forces that lay outside criticism, in history.

132


MCAT Practice Test 1 EXPLAINED.pages

Adapted from M. Bradbury, The scholar who misread history. ©1991 by New York Times. 28. Which of the following statements best expresses the passage author’s message? Please choose from one of the following options. 1 The true meaning of texts can never be decided. 2 Most literary criticism is now deconstructionism. 3 Deconstructionist and existentialist critics compete. 4 *The most salient issue for literary critics should be creative merit. This is a “Foundations of Comprehension” question, which means that it wants you to understand the central theme or idea of the passage. Hint #2 The author describes deconstructionism as a movement rooted in “anxieties about reality and appearance” (paragraph 1), that adopts that view that “the text has no fixed meaning” (paragraph 1), and posits “the futility of any attempt to name reality” (paragraph 2). The author describes the rise of deconstructionism in order to set the stage to push back against this viewpoint. In direct opposition to the view that all text is “undecidable discourse”, he argues “When writers are censored, imprisoned, killed, or threatened with death for their writings, it is not because their discourse is undecidable” (option A). The author describes the popularity of deconstructionism especially in the 1970s, but does not explicitly state that most literary criticism today falls into this category (option B). The author highlights some tensions between existentialism and deconstructivism, but doesn’t talk about competition among critics (option C). In contrast to the deconstructionist view, the author believes that “the point that needs to be reaffirmed is that writing is an existential act, an imaginative exploration of ideas” (paragraph 6). The author believes that writing is an art and that fiction should be honored.., therefore this suggests the most salient issue for literary criticism should be the consideration of creative merit (option D). Hint #3 Thus, option D is correct.

133


MCAT Practice Test 1 EXPLAINED.pages

29. The passage implies that the declaration “America is deconstruction” (paragraph 2) means that: Please choose from one of the following options. 1 *the constant revisions typical of U.S. culture capture the spirit of deconstructionism. 2 literature produced in the U.S. is especially suited to analysis by deconstruction. 3 U.S. literary critics developed the scholarly methods adopted by deconstructionism. 4 U.S. higher education is controlled by the political agenda of deconstructionism. Hint #1 This is a “Foundations of Comprehension” question, which means that it wants you to be able to figure out what specific words or phrases mean within that context. Hint #2 The passage describes the United States as a home for deconstruction because it was known for “historically dissociated construction of random meanings.” (paragraph 2) This phrasing suggests that America and deconstructionism have in common a lack of consistency in interpretation (option A). The passage does not discuss where the literature that was analyzed by deconstructionists had been written, and does not imply that U.S. literature was particularly suited to the approach (option B). The passage states that deconstruction emerged from Paris, and that the analytic methods were first introduced as part of instruction in the US. It does not assert that the U.S. critics played any role in developing the methods that were applied (option C). The passage suggests that deconstructivism shares “affinities with sexual and racial politics” but does not assert that US higher education is controlled by any of these agendas (option D). Hint #3 Thus, option A is correct. 30. According to the views of the passage author, a responsible review of a stage production of Shakespeare's Julius Caesar would probably: Please choose from one of the following options.

134


MCAT Practice Test 1 EXPLAINED.pages

1

point out evidence that the play promoted the interests of Shakespeare's gender and class. 2 argue that Shakespeare distorted history unintentionally because of his limited knowledge. 3 *assume that Shakespeare's presentation of events reflected his artistic sensibility. 4 recommend the play as an entertaining introduction to Roman history. Hint #1 This is a “Reasoning Beyond the Text” question, which means that it wants you to either apply or extrapolate the ideas in the passage to new situations or to assess how new information would impact the ideas presented in the passage. It is important to understand the assumptions underlying the article, and how new information may or may not shift the central thesis. Hint #2 The author believes that writing is an “imaginative exploration of ideas” (paragraph 6) and that “we need an ambiance around writing that affirms its nature as creativity, as art” (paragraph 7). The author would endorse a review that assumes that Shakespeare’s presentation of events reflected his artistic sensibilities (option C). The author does not recommend that a review should recommend the play as an accurate representation of Roman history (option D), since it’s actually a creative product of Shakespeare’s imagination. Similarly, the author would be unlikely to argue that Shakespeare unintentionally distorted historical events due to limited knowledge (option B). Rather, if the primary purpose of literature is creative expression, than any historical distortions are irrelevant to the primary purpose of literature. According to the author, a responsible review of the play should not focus on evidence that it promoted Shakespeare’s gender or class which would be interpreting the play as a vehicle for social commentary. In contrast to the deconstructivist view, which he asserts has “transformed everything into social commentary” paragraph 4, the author believes that “Literature is not a subordinate category of social criticism” (paragraph 6). This excludes option A. Hint #3 Thus, option C is correct.

135


MCAT Practice Test 1 EXPLAINED.pages

31. What is the author’s primary purpose in discussing deconstructionism in paragraphs 3, 4, and 5 Please choose from one of the following options. 1 *To denounce its triviality in order to strengthen the case for a contrasting vision 2 To acknowledge its existentialist principles in order to propose an extension of these principles 3 To reveal its lack of sympathy for writers as an argument for a more courteous manner by critics 4 To contrast its analytic method with an evaluative approach that may prove equally interesting Hint #1 This is a “Foundations of Comprehension” question, which means that it wants you to be able to figure out the purpose of specific terms or ideas within the context of the passage. The question asks you to integrate separate passage components into a more generalized and complex interpretation of passage meaning, and to determine the significance of and relationships among different parts of a passage. Hint #2 The author offers a narrative of increasing deconstructionist zeal, that attacked literature until “all that would be left of it would be a few bare bones of undecidable discourse and some tattered leather bindings” (paragraph 5). The extremism of deconstructionism is then contrasted with the author’s belief that “writing is an existential act, an imaginative exploration of ideas” (paragraph 6). This juxtaposition of descriptions was done to help denounce deconstructionism and strengthen a contrasting vision. The author does not portray deconstructionism as having existentialist principles (option B). in fact he contrasts deconstructionism and existentialism in the final paragraph when he states “We need an ambiance around writing that affirms its nature as creativity, as art,... This was the position from which Jean-Paul Sartre with his freedom-affirming existentialism started the postwar debate…” There is no discussion of an evaluative approach or any acknowledgement that deconstructionism could be equally interesting as such an approach (option D). The main point of the passage is to criticize the value of deconstructionism. The author does not simply believe that the deconstructionism movement was not courteous enough to writers (option

136


MCAT Practice Test 1 EXPLAINED.pages

C). Although he argues that deconstructionists both defamed authors and claimed that literature had been written by entirely the wrong people for entirely the wrong reasons (paragraph 5), his main point is not that reviewers should be more courteous, but rather that deconstructionism lacks a fundamental appreciation of the nature and value of writing as an art form. Hint #3 Thus, option A is correct.

Is there such a thing as free will? Perhaps the philosopher who has gotten closest to a sensible understanding of free will is Daniel C. Dennett, who thinks of the phenomenon as “the power to veto our urges and then to veto our vetoes . . . the power of imagination, to see and imagine futures.” Over the last few decades, science has made small but significant advances in understanding the relationship between conscious and unconscious thought, and the data are beginning to paint a picture that seems to validate Dennett’s views. In the 1970s, Benjamin Libet wired people to an electroencephalogram and measured when they reported having a particular conscious thought about an action and when the nerve impulses corresponding to the initiation of the actual action started. Astoundingly, the subjects had actually made (unconsciously) the decision to act measurably earlier than when they became aware of it consciously. The conscious awareness, in a sense, was a “story” that the higher cognitive parts of the brain told to account for the action. So science may be showing us that free will is more a feeling than a real manifestation of independent will. As psychologist Dan Wegner put it, “We see two tips of the iceberg, the thought and the action, and we draw a connection.” Libet’s position is a bit more moderate and is akin to

137


MCAT Practice Test 1 EXPLAINED.pages

Dennett’s. Libet says that free will is a form of veto power, filtering and sometimes blocking decisions provisionally made at the unconscious level. Quantum mechanics is sometimes brought into discussions of free will by supporters of pseudoscience because it is very technical and, more important, incomprehensible enough to lend that aura of scientific credibility without committing one to specific details. Some philosophers and scientists suggest that perhaps free will can be explained by occasional quantum fluctuations that, by interfering with subcellular phenomena in the brain, create a partial decoupling of our decision-making processes from the standard macroscopic laws of causality. This is nonsense, not only because we have absolutely no evidence of “quantum fluctuations” (whatever they are) at the brain level, but because, even if they did happen, they would—at most—generate random, not free, will. And random will is not one of those varieties of free will that is, in Dennett’s words, “worth having.” Another source of confusion between science and philosophy when it comes to free will is to be found in the rather vague concept of “emergent properties,” for example, the notion of free will being an emergent property of the higher brain’s functions. Even though some scientists are predisposed to reject emergent properties, “emergence” can actually be studied scientifically and is a rather common phenomenon. For example, when hydrogen and oxygen combine to form water, the resulting molecule has emergent physical–chemical properties, in the sense that the temperatures marking transition states are not simple functions of the properties of the individual atoms. Some philosophers have argued that emergence restricts the limits of reductionism, not because it isn’t “physics all the way down,” but because, frankly, a quantum mechanical description of, say, the Brooklyn Bridge isn’t going to be very helpful. Emergence entails that certain phenomena are best studied, and understood, at some levels of analysis rather than others, and free will may well fall into this category. To say that it is an emergent property of the brain is not a call for magic or pseudoscience, just the realization that neurobiology and psychology are better positioned than quantum mechanics to understand it.

138


MCAT Practice Test 1 EXPLAINED.pages

Adapted from M. Pigliucci, Can there be a science of free will? ©2007 the Committee for Skeptical Inquiry. 32. Based on the passage, which of the following elements of Dennett’s characterization of free will best corresponds to what Libet calls “decisions provisionally made at the unconscious level”? Please choose from one of the following options. 1 *Urges 2 Vetoes 3 Vetoes of vetoes 4 Imagining futures Hint #1 This is a “Foundations of Comprehension” question, which means that it wants you to be able to figure out the purpose of a specific phrase or ideas within the context of the passage. Hint #2 The author writes that, “Libet’s position is a bit more moderate and is akin to Dennett’s. Libet says that free will is a form of veto power, filtering and sometimes blocking decisions provisionally made at the unconscious level” (paragraph 3). In comparison, Dennett thinks of free will as “the power to veto our urges and then to veto our vetoes” (paragraph 1). Both Libet and Dennett describe the use of the veto or veto power. Whereas Libet describes vetoing “decisions provisionally made at the unconscious level” (paragraph 3), Dennett describes vetoing urges (and then overriding that veto), so there is a correspondence between provisional decisions and urges (option A). In contrast to decisions made at the unconscious level, or urges, the power to veto and veto our vetoes is asserted by Dennett in paragraph 1 to represent our free will (option B and C). The phenomenon of free will is also described as being the power to imagine futures (option D). Hint #3 Thus, option A is correct. 33. With which of the following statements about “emergent properties” does the passage information suggest that the passage author would be most likely to agree?

139


MCAT Practice Test 1 EXPLAINED.pages

Please choose from one of the following options. 1 Psychology is well positioned to study emergent properties of the functioning of the brain. 2 Supporters of pseudoscience tend to favor detailed explanations of free will. 3 Free will is an emergent property that is decoupled from the standard macroscopic laws of causality. 4 Emergent properties of molecules are just as easy to study as the properties of their individual atoms. Hint #1 This is a “Foundations of Comprehension” question, which means that it wants you to understand the central theme or idea and be able to figure out what specific words or phrases mean within that context. It is important to identify specific parts of the passage such as the thesis, example, and counterexample. Hint #2 The author describes “emergent properties” (paragraph 5) as those properties that are a function of higher ordered states and cannot be directly inferred from simply understanding the component parts. The example of emergent properties that is offered is of water having physicalchemical properties that extend beyond the properties of individual hydrogen and oxygen atoms. The author writes that “To say that it is an emergent property of the brain…just the realization that neurobiology and psychology are better positioned than quantum mechanics to understand it” (paragraph 6). Although the author doesn’t state that all properties of the brain are best studied by psychology, but that some properties of the brain may be emergent and therefore lend themselves to being studied by psychology (option A). The author suggests that “Quantum mechanics is sometimes brought into discussions of free will by supporters of pseudoscience because it is very technical and, more important, incomprehensible enough to lend that aura of scientific credibility without committing one to specific details” (paragraph 4). In a later paragraph he asserts “Another source of confusion between science and philosophy when it comes to free will is to be found in the rather vague concept of “emergent properties,” If supporters of pseudoscience are motivated to adopt positions that are hard to pin down, the vagueness of this concept could lead supporters of pseudoscience to favor it. The author doesn’t talk

140


MCAT Practice Test 1 EXPLAINED.pages

about emergent properties being decoupled from the standard laws of causality (option C). Finally, the author uses the example of emergent properties in molecules to demonstrate that this is quite common, but doesn’t talk about the relative ease of studying one set of properties over another (option D). Hint #3 Thus, option A is correct. 34. Assume that the phenomenon of happiness resists being described through reductionism. Given this assumption, and based on the passage, which of the following statements is most likely to be true? Please choose from one of the following options. 1 A description of the phenomenon of happiness can only be given in terms of one level of analysis. 2 A description of the phenomenon of happiness can be given without referring to emergent properties. 3 A description of the phenomenon of happiness requires referring to technical aspects of quantum mechanics. 4 *A description of the phenomenon of happiness is more helpful in terms of psychology than in terms of physics. Hint #1 This is a “Reasoning Beyond the Text” question, which means that it wants you to either apply or extrapolate the ideas in the passage to new situations or to assess how new information would impact the ideas presented in the passage. It is important to understand the assumptions underlying the article, and how new information may or may not shift the central thesis. Hint #2 The author believes that, “certain phenomena are best studied, and understood, at some levels of analysis rather than others” (paragraph 6). If happiness is resistant to being described through reductionism, then based on the passage we can conclude that it is possible that “neurobiology and psychology are better positioned than quantum mechanics to understand it” (paragraph 6), which supports option D and refutes option C. The author does not support the idea that emergent properties can only be given at one level of analysis (option A), rather he argues that we must identify the appropriate level(s) of analysis for any given property. If happiness resists

141


MCAT Practice Test 1 EXPLAINED.pages

being described through reductionism, then that is when the author suggests it may be better understood by referring to emergent properties. The author does not suggest another alternative that would not refer to emergent properties when a reductionist approach doesn’t work (option B). Hint #3 Thus, option D is correct. 35. Suppose that a scientist were to demonstrate to the passage author that quantum fluctuations occur at the subcellular level of the brain. Based on the fourth paragraph, is the author’s opinion that the quantum mechanical description of free will is “nonsense” likely to change? Please choose from one of the following options. 1 Yes, because quantum fluctuations would have been clearly defined 2 Yes, because quantum fluctuations would be subject to standard laws of causality 3 *No, because quantum fluctuations would still generate random will 4 No, because some scientists reject quantum mechanics Hint #1 This is a “Reasoning Beyond the Text” question, which means that it wants you to either apply or extrapolate the ideas in the passage to new situations or to assess how new information would impact the ideas presented in the passage. It is important to understand the assumptions underlying the article, and how new information may or may not shift the central thesis. Hint #2 The author writes that “This is nonsense, not only because we have absolutely no evidence of “quantum fluctuations” (whatever they are) at the brain level, but because, even if they did happen, they would—at most— generate random, not free, will” (paragraph 4). He articulates that the notion that free will can be explained by quantum fluctuations is nonsense because even if quantum fluctuations occur at the subcellular level of the brain, this would generate random will, rather than free will. So proof that quantum fluctuations do occur, which would presumably also provide a definition for them, would not change his position. The author does not believe that quantum fluctuations could explain free will even if they could be shown to occur at the brain level (option A and B). His main argument is that quantum fluctuations, even if they did exist, would generate random

142


MCAT Practice Test 1 EXPLAINED.pages

will (option C). He does not discuss whether some scientists reject quantum mechanics (option D). Hint #3 Thus, option C is correct.

In a recent study, psychologist Anne Maass investigated the effects of courthouse architecture on the psychological well-being and cognitive processes of potential users. Specifically, she compared two courthouses located in Padova, Italy: the old courthouse, located in a former convent originally built in 1345, and the new courthouse, built in 1991 and designed by Gino Valle, an internationally known architect. Although serving or having served the same purpose, the two buildings have completely different styles—one is an old building with a rather residential look, warm colors, large windows, and a large wooden door, the other a massive, gray, semi-circular building, with narrow windows, and an entrance enclosed between two huge walls. When study participants were asked to imagine themselves accompanying a friend to the courthouse, they reported greater discomfort and stress when anticipating a trial in the modern building. However, contrary to predictions, this was true only when they were already familiar with the two buildings. It is possible that photographs reduced the actual impact of the

143


MCAT Practice Test 1 EXPLAINED.pages

architectural design, although this would contradict prior research by architect Gavin Stamp showing that distortions due to photographic presentation have negligible effects on preference. Another possibility for participants’ greater discomfort when imagining going to the new courthouse is that those with prior experience may have been exposed to the building from multiple angles, whereas unfamiliar participants received information only about the building’s facade. It is important to note that participants did not generally dislike the new building. From the standpoint of general aesthetic distinctions such as beauty versus ugliness, no differences emerged between the two buildings; if anything, the new building was seen by the participants as slightly more attractive. The data suggest that participants responded more to the intimidating nature of the building than to its beauty. The most important result of Maass’s research is that courthouse architecture was found to affect the estimated likelihood of conviction. Participants were more pessimistic about the trial outcome when they imagined entering the new building than when they imagined entering the old one. (This occurred regardless of whether participants had any prior familiarity with the respective buildings.) It remains unclear exactly which architectural features are responsible for the observed shift in likelihood of conviction estimates. The modern building differs on so many dimensions (size, color, shape, building materials, age, and so on) from the old building that it is impossible to isolate their individual impact. Also, it may be the interaction of features that creates the overall impression of the building as intimidating. How exactly do architectural features affect social-cognitive processes such as likelihood estimates? One possibility is that design features affect the emotional well-being or mood of the user which, in turn, biases his or her thought processes. For example, the architectural characteristics of the new courthouse seem to have made hypothetical users feel anxious and tense, and a bad mood has been shown to induce negative thoughts and expectations. However, building type affected perceived likelihood of conviction also for those participants who showed no enhanced discomfort in reaction to the new building.

144


MCAT Practice Test 1 EXPLAINED.pages

Another and more plausible possibility is that the design features of the new courthouse activated specific thoughts and mental associations related to conviction. For example, some participants spontaneously commented that the new building has greater resemblance to a prison than to a courthouse; others mentioned that the two high walls enclosing the entrance give the impression that those who enter the building are already convicted. Adapted from A. Maass, “Intimidating Buildings: Can courthouse architecture affect perceived likelihood of conviction?” Environment and Behavior. ©2000 by Sage Publications, Inc. 36. According to Maass, people’s reactions to the two buildings were independent of: Please choose from one of the following options. 1 *whether or not they considered the buildings to be attractive. 2 what they knew of the history of the buildings. 3 the relative size of the buildings. 4 the influence of their peers. This is a “Foundations of Comprehension” question, which means that it wants you to understand the central theme or idea and be able to figure out what specific words or phrases mean within that context. It is important to identify specific parts of the passage such as the thesis, example, and counterexample. Hint #2 In the second sentence of the third paragraph, Maass writes that “From the standpoint of general aesthetic distinctions such as beauty versus ugliness, no differences emerged between the two buildings; if anything, the new building was seen by the participants as slightly more attractive. The data suggest that participants responded more to the intimidating nature of the building than to its beauty” (paragraph 3). The reaction that individuals were having was not based upon their assessment of the building’s beauty, in other words it was independent of this variable (option A). The passage states that people reported “greater discomfort and stress when anticipating a trial in the modern building… when they were already familiar

145


MCAT Practice Test 1 EXPLAINED.pages

with the two buildings”. This suggests that knowing the history of the buildings could be influencing reactions, and no argument is made that knowledge of the history of the buildings did not influence reactions (option B). The passage indicates that people’s reaction to the building is unlikely to have been independent of size (option C), in fact, it may have contributed to the reaction by creating “the overall impression of the building as intimidating” (paragraph 4). There is limited information in the passage about peer influence. People are asked to imagine themselves accompanying a friend to the courthouse, but based on the passage, it’s unclear if people’s reactions were dependent or independent of their peers (option D). Hint #3 Thus, option A is correct. 37. In the passage, the author justifies rejecting the emotional-moodas-mediator explanation with which of these reasons: Please choose from one of the following options. 1 emotions do not influence our expectations nearly as much as we think they do. 2 *participants without negative emotions were affected in the same way as those with negative emotions. 3 the study did not measure and compare the participants’ moods before and after their imagined entry into the courthouse. 4 the study did not compare the mood of those entering the new building with the mood of those who imagined entering the new building. This is a “Reasoning Within the Text” question, which means that it wants you to explore the key idea, claim, or theme that is being expressed in a passage. You will have to evaluate the soundness of an argument, the reasonableness of its conclusions, and/or the appropriateness of its generalizations. It is crucial to carefully examine the strengths and weaknesses of the key ideas based on the information given. Hint #2 In the last sentence of the fifth paragraph, the author notes that, “building type affected perceived likelihood of conviction also for those participants who showed no enhanced discomfort in reaction to the new

146


MCAT Practice Test 1 EXPLAINED.pages

building” (paragraph 5). In other words, the findings were similar for individuals with and without negative emotions (option B). There is no information in the passage to support the idea that we believe that emotions do not influence our expectations nearly as much as we think they do (option A). The author does not discuss the absence of data about participant mood before and after they imagined entering the courthouse (option C), nor the absence of data of comparing participants who actually entered the building versus those that only imagined entering (option D). Hint #3 Thus, option B is correct. 38. Which of the following examples is most consistent with Maass’s suggestion that the architectural design features can affect perceptions by activating “specific thoughts and mental associations” (final paragraph)? Please choose from one of the following options. 1 *A popular bank has décor, colors, and accents suggestive of gold. 2 An unpopular car wash does not have a drive-through design. 3 A popular restaurant has a children’s play area outside. 4 An unpopular grocery store has bad produce. Hint #1 This is a “Reasoning Beyond the Text” question, which means that it wants you to either apply or extrapolate the ideas in the passage to new situations or to assess how new information would impact the ideas presented in the passage. It is important to understand the assumptions underlying the article, and how new information may or may not shift the central thesis. Hint #2 The suggestion that design features can activate specific thoughts relevant to the building’s purpose without directly impacting the functionality of the building, is best supported by the example of a bank that has has gold décor, colors, and accents that suggest financial stability and wealth. This decor, although not directly affecting the use of the building for banking, primes thoughts and mental associations related to the primary purpose of a bank which then could affect its perception and popularity (option A). The lack of several important design features that can affect the user experience, such as the lack of a drive-through design for carwashes (option B), or the lack of a play area at a restaurant (option C), could affect

147


MCAT Practice Test 1 EXPLAINED.pages

popularity more directly because the building design is less suited for its use by the consumer. Bad produce is not an architectural design feature of a grocery store (option D). Hint #3 Thus, option A is correct. Today’s parents face a tough battle. Neighborhoods are a lot more complicated than they were in the 1960s: every culture, every religion, every idea, every different standard, lives right next door. Information is received at lightning speed via the Internet, and children can be caught up in this whirlwind, subjected to things that they are still too young to understand or are emotionally unfit to handle. Censorship seems to be an answer to the growing problem of how to care for and watch over our children. But books are meant for exploration, for questioning. Within a book’s pages, children are safe to explore their feelings and reflect on their own situations. Putting the right book into the hands of the right child has great value and changes lives. It can be empowering, motivating, and inspiring. Here in the United States, an ostensibly free country, one where people are encouraged and given the legal right to speak their minds, we have been balancing personal freedoms and rights. But our media challenge this balance every day. As consumers, we respect artists and allow them the freedom of expression. At the same time, we are aware that children are seeing some unsuitable situations—but we are not always in agreement about what we want our children to watch, hear, or read. One political solution is rating systems, intended to help parents pick appropriate material for their children based on content, theme, violence, language, nudity, sensuality, drug abuse, and other elements. However, the rating systems have not stopped today’s lyrics from becoming more explicit, our cable television system from containing more swearing and sexual content, and our movies from becoming bloodier and more violent. And despite all the warnings and all the ratings, children are still listening to these songs, watching these television shows, and renting these movies. The rating system may have convinced politicians, parents, and librarians

148


MCAT Practice Test 1 EXPLAINED.pages

that it could do the job of protecting their young. It may have given people a false sense of security. But in reality, it means nothing when no one is there to monitor children’s actions and discuss appropriate behavior. Parents have a vested interest in their children. Creating a home in which a child feels safe is their responsibility. Creating a home where a child can safely make mistakes is their responsibility. Home is the first place where a child learns right from wrong, good from bad, healthy from unhealthy. It is the parents’ job to give their child a good defense by helping them establish boundaries. School helps to reinforce these lessons. Teachers help children by challenging them, instructing them, and helping them move on to the next level of maturity and understanding. A teacher may know, before a parent, when a child is ready for the next level or is mature enough to handle a theme or topic. When there is communication and respect between parent and teacher, the child’s development is the winner. America is a free society and has plenty of forums where people can express their views: newspapers, radio, billboards, and the Internet. People can discuss their differences and learn from each other. Why shouldn’t we allow our children that same rich experience? Banning a book is about as helpful as using a match in a hurricane. It does not shed light on anything and gets blown around by a lot of wind. Nor does sticking a label on a problem make it go away. Only in discussing, in sharing comments and concerns, is there growth and understanding. Let us show our children that knowledge is the most empowering censor they can use. Adapted from L. Caravette, “Censorship: An unnecessary evil,” The Looking Glass : New Perspectives on Children's Literature. ©2008 New Perspectives on Children's Literature. 39. Which of the following explains the relationship between the claim "Children’s lives should be very structured, so that they cannot be exposed to any uncontrolled sources of information" and the claims in the opening paragraph of the passage? Please choose from one of the following options.

149


MCAT Practice Test 1 EXPLAINED.pages

1 2 3 4

It provides an explanation for a difficulty implied in the opening paragraph. It illustrates an example of the author’s preferred solution. *It both contradicts and resolves a difficulty implied in the opening paragraph. It neither supports nor challenges claims made in the passage.

This is a “Reasoning Beyond the Text” question, which means that it wants you to either apply or extrapolate the ideas in the passage to new situations or to assess how new information would impact the ideas presented in the passage. It is important to understand the assumptions underlying the article, and how new information may or may not shift the central thesis. Hint #2 The opening paragraph describes a particular challenge of parenting as being due to children being caught up in a myriad of influences coming from their neighborhoods and community as well as from global sources accessed through the Internet. If children’s lives were very structured, so that they couldn’t be exposed to any uncontrolled sources of information, this would stand in juxtaposition to the situation described in the opening paragraph (option C). To the extent that parents face difficulty specifically because of uncontrolled sources of information, than that difficulty would be eliminated with this new situation. This also means that option D is incorrect since the claim “that parents face a tough battle” made in the opening paragraph would no longer have support. The new information would not provide an explanation for the current conditions (option A). The opening paragraph does not discuss the author’s view on censorship (option B). Hint #3 Thus, option C is correct. 40. The author’s attitude toward the censorship of children’s books is one of: Please choose from one of the following options. 1 cautious support. 2 staunch neutrality. 3 general disapproval. 4 confused ambivalence.

150


MCAT Practice Test 1 EXPLAINED.pages

Hint #1 This is a “Foundations of Comprehension” question, which means that it wants you to understand the central theme or idea of the passage. Hint #2 In the second paragraph, the author writes, “Censorship seems to be an answer to the growing problem of how to care for and watch over our children” which he then continues with “but books are meant for exploration, for questioning…” and a number of other counter arguments against censorship and rating systems. In the last paragraph, she summarizes her main views on censorship by stating “Banning a book is about as helpful as using a match in a hurricane. It does not shed light on anything and gets blown around by a lot of wind” (paragraph 7). Thus the author expresses general disapproval for censorship of children’s books. The author is not supportive (option A), neutral (option B), nor does the author express confused ambivalence (option D) on this point. Hint #3 Thus, option C is correct.

41. Which of the following is most like the rating systems described in paragraph 4? Please choose from one of the following options. 1 Teachers’ grades and comments that appear on student assignments. 2 Library catalogs that classify books according to various fiction and non-fiction genres. 3 Road signs that inform drivers of dangers and speed limits. 4 *Nutritional information about calories and vitamin content appearing on product packages. Hint #1 This is a “Reasoning Beyond the Text” question, which means that it wants you to either apply or extrapolate the ideas in the passage to new situations

151


MCAT Practice Test 1 EXPLAINED.pages

or to assess how new information would impact the ideas presented in the passage. It is important to understand the assumptions underlying the article, and how new information may or may not shift the central thesis. Hint #2 The rating systems described are a way of helping parents pick appropriate items for their children, however, “despite all the warnings and all the ratings, children are still listening to these songs, watching these television shows, and renting these movies” (paragraph 4). The author believes that rating systems “may have given people a false sense of security” (paragraph 4). Similar to protecting children from inappropriate media, many parents are vigilant about making sure that their children are protected from inappropriate foods. In addition, this is another area where content about “about calories and vitamin content on children’s snacks” have not curbed the childhood obesity epidemic. The strong parallel between media rating systems and nutritional labeling makes this the most similar of the options. Teachers’ grades and comments do not protect the child from consuming inappropriate content (option A). Library catalogs help to organize books, but do not protect the child from consuming inappropriate content (option B). Road signs help drivers to travel more safely, but do not protect the child from consuming inappropriate content (option C). Hint #3 Thus, option D is correct. 42. The author mentions both the legal right to speak our minds and our lack of agreement about what we want our children to watch, hear, and read most likely in order to: Please choose from one of the following options. 1 illustrate the need for censorship. 2 explain why librarians should monitor what children read. 3 characterize the culture of the United States relative to that of other countries. 4 *explain why rating systems have been developed Hint #1 This is a “Reasoning Within the Text” question, which means that it wants you to explore the key idea, claim, or theme that is being expressed in a passage. You will have to evaluate the soundness of an argument, the

152


MCAT Practice Test 1 EXPLAINED.pages

reasonableness of its conclusions, and/or the appropriateness of its generalizations. It is crucial to carefully examine the strengths and weaknesses of the key ideas based on the information given. Hint #2 The author describes the balancing act between “personal freedoms and rights,” and the desire to protect our children from unsuitable material in paragraph 3. Acknowledging people’s legal right to speak their mind is in direct opposition to the use of censorship, so this does not support option A. There is no proposal in this passage that librarians should monitor students’ choices, so this cannot be offered in explanation of that point (option B). and the same time as the There is no direct comparison made between the United States and other countries (option C). After mentioning the right to speak our mind, and our lack of agreement on what might suitable for children in the third paragraph, the next paragraph begins, “One political solution is rating systems, intended to help parents pick appropriate material for their children” (paragraph 4). Thus, this information serves to introduce and explain why the United States has adopted rating systems intended to support parental choice over what their children are exposed to, while also preserving free speech (option D). Hint #3 Thus, option D is correct.

A throng of bearded men, in sad-colored garments, and gray, steeplecrowned hats, intermixed with women, was assembled in front of a wooden prison. The founders of a new colony, whatever Utopia of human virtue and happiness they might originally project, have invariably recognized it among their earliest practical necessities to allot a portion of the virgin soil as the site of a prison. Before this ugly edifice was a grass-plot, much overgrown with burdock, pig-weed, apple-peru, and such unsightly vegetation which evidently found something congenial in the soil that had so early borne the black flower of civilized society, a prison. But, on one side of the portal was a wild rose-bush, covered with its delicate gems,

153


MCAT Practice Test 1 EXPLAINED.pages

which might be imagined to offer their fragrance and fragile beauty to the prisoner as he went in, and to the condemned criminal as he came forth to his doom. This rose-bush, by a strange chance, has been kept alive in history. It may serve to symbolize some sweet moral blossom that may be found along the track, or relieve the darkening close of a tale of human frailty and sorrow. The door of the jail being opened from within, there appeared, like a black shadow emerging into sunshine, the grim and grisly presence of the townbeadle. He laid his right hand upon the shoulder of a young woman; until, on the threshold of the prison-door, she repelled him, by an action marked with natural dignity and force of character, and stepped into the open air, as if by her own free will. Hester Prynne bore in her arms an infant, who winked and turned aside its little face from the too vivid light of day. When the young woman- the mother of this child- stood fully revealed before the crowd, it seemed to be her first impulse to clasp the infant closely to her bosom; not so much by an impulse of motherly affection, as that she might thereby conceal a certain token, which was wrought or fastened into her dress. In moment, however, wisely judging that one token of her shame would but poorly serve to hide another, she took the baby on her arm, and, with a burning blush, and yet a haughty smile, and a glance that would not be abashed, looked around her townspeople and neighbors. On the breast of her gown, in fine red cloth, surrounded with an elaborate embroidery and fantastic flourishes of gold thread, appeared the letter A. It was so artistically done, and with so much fertility and gorgeous luxuriance of fancy; and which was of a splendor in accordance with the taste of the age, but greatly beyond what was allowed by the sumptuary regulations of the colony. The young woman was tall, with a figure of perfect elegance on a large scale. Those who had before known her and had expected to behold her dimmed and obscured by a disastrous cloud, were astonished to perceive how her beauty shone out, and made a halo of the misfortune and ignominy in which she was enveloped. But the point which drew all eyes, so that both men and women, who had been familiarly acquainted with

154


MCAT Practice Test 1 EXPLAINED.pages

Hester Prynne, were now impressed as if they beheld her for the first timewas that SCARLET LETTER, so fantastically embroidered and illuminated upon her bosom. It had the effect of a spell, taking her out of the ordinary relations with humanity and enclosing her in a sphere by herself. 43. Information in the passage tells the reader that which of the following are Hester Prynne’s two tokens of shame? Please choose from one of the following options. 1 *The letter “A” on her chest and her baby. 2 Her public humiliation and her baby. 3 The letter “A” on her chest and her criminal record. 4 Her ostentatious dress and her baby. This is a “Foundations of Comprehension” question which is asking you to recognize an accurate paraphrase or summary of the central theme or of particular statements in the passage, or to interpret word choices made by the author. Hint #2 Hester tries to use her baby to hide the scarlet letter sewn on her chest “which was wrought or fastened into her dress” but then realizes that “her shame would but poorly serve to hide another.” Thus, the author refers to the baby as one source of her shame, and the letter sewn to her chest as a second source of shame (option 1). The passage does not directly reference her public humiliation (option 2), her criminal record (option 3), or her dress as sources of shame for Hester (option 4). Hint #3 Option 1 is correct. 44. Which of the following statements would the author most likely agree with? Please choose from one of the following options. 1 Hester’s rejection of the grim and grisly town beadle underscores the danger of anarchy in small communities. 2 The sad-colored garments worn in the community are meant to portray the destitution of early Puritan society. 3 The rose-bush among the unsightly vegetation in front of the prison represents the merciful tendencies of Puritan society.

155


MCAT Practice Test 1 EXPLAINED.pages

4

*The rosebush symbolizes that there is a natural moral sense that transcends the strict and less understanding moral order of the Puritans.

This is a “Foundations of Comprehension” question which is asking you to recognize an accurate paraphrase or summary of the central theme or of particular statements in the passage, or to interpret word choices made by the author. Hint #2 The author does not present Hester Prynne as an anarchist just because she seems insufficiently ashamed at breaking one specific law (option 1). The author uses darkness and dreariness when characterizing the town and townspeople to emphasize the austere customs of the community, not its destitution or poverty (option 2). The typical vegetation around the prison was “unsightly”,” while the “wild rose-bush” offers its “fragrance and beauty” to prisoners. It symbolizes “some sweet moral blossom” brought forth by nature in contrast to the Puritan justice system and their prison portrayed as “a black flower of civilized society”(option 4). Finally, the rose-bush is “wild” and outside Puritan control and thus not a token of their mercy (option 3). Hint #3 Option 4 is correct.

45. All of the following examples of imagery are mentioned, but which one is NOT used by the author to support the theme of dignity? Please choose from one of the following options. 1 “halo of misfortune” 2 “she repelled him...and stepped into the open air” 3 “haughty smile” 4 *“burning blush” Hint #1

156


MCAT Practice Test 1 EXPLAINED.pages

This is a “Reasoning Within the Text” question. These items generally ask you to think about the author’s reasoning, such as what claim an author is trying to support with a piece of evidence, what assumption underlies a specific statement, or whether an argument is flawed. Hint #2 Dignity is the state or quality of being worthy of respect. You must ask yourself whether the selected imagery conveys the fact that Hester’s persecution is accompanied by a sense of self-respect. A “halo” is something that surrounds the head of a saint or holy person, and is meant to indicate their worthiness and grace. Hester’s “halo of misfortune” is an image that portrays how she bears her misfortune with saintly grace, a quality worthy of respect (option 1). The author describes Hester’s rejection of the beadle as “an action marked with natural dignity and force of character” (option 2). Finally, the author writes, “she took the baby on her arm, and, with a burning blush, and yet a haughty smile, and a glance that would not be abashed, , looked around her townspeople and neighbors. ” Thus, her “blush” which reflects her embarrassment (option 4) is contrasted by the word “yet” against her “haughty smile”. The burning blush denotes (option 3) a sense of shame contrasted against a sense of pride in the face of persecution. Hint #3 Option 4 is correct.

46. The author would most likely agree with which of the following statements about the prison system in modern-day society: Please choose from one of the following options. 1 Eliminating most crime from society is possible with a strong prison system in place. 2 *Creating a perfect society completely free of crime is unrealistic. 3 Prisons are unnecessarily expensive and a waste of taxpayer dollars.

157


MCAT Practice Test 1 EXPLAINED.pages

4

Incarceration is inhumane and should be forbidden by governments internationally. Hint #1 This is a “Reasoning Beyond the Text” question, which means that it wants you to either apply or extrapolate the ideas in the passage to new situations or to assess how new information would impact the ideas presented in the passage. It is important to understand the assumptions underlying the article, and how new information may or may not shift the central thesis. Hint #2 In the passage, while the author is describing an experience of persecution and marginalization, there is never any mention of the cost of having a prison (option 3) or about the plausibility of eliminating all crime with a strong system of incarceration (option 1). The author does not argue in favor of forbidding incarceration (option 4), in fact, the author describes prison as a social necessity when he writes, “The founders of a new colony, whatever Utopia of human virtue and happiness they might originally project, have invariably recognized it among their earliest practical necessities to allot a portion of the virgin soil as the site of a prison.” The author thus recognizes that prisons are a necessary part of human society, regardless of the utopian ideas that might motivate a community’s social system (option 2). Hint #3 Option 2 is correct.

47. Suppose there is a newcomer to the community in which Hester Prynne was imprisoned. The newcomer dresses lavishly, eats voraciously, and spends excessively. It is MOST LIKELY that the newcomer: Please choose from one of the following options. 1 would be embraced as a new member of the community.

158


MCAT Practice Test 1 EXPLAINED.pages

2 would be swiftly tried, convicted, and executed. 3 would be someone who shares and respects Puritan values. 4 *would be met with disapproval by the community. Hint #1 This is a “Reasoning Beyond the Text” question, which means that it wants you to either apply or extrapolate the ideas in the passage to new situations or to assess how new information would impact the ideas presented in the passage. It is important to understand the assumptions underlying the article, and how new information may or may not shift the central thesis. Hint #2 There are several descriptions in the passage that suggest self-indulgence is frowned upon in the community. The gold-threaded scarlet letter’s “gorgeous luxuriance of fancy” is what marginalizes Hester from the rest of society, who wear gray, “sad-colored garments.” The author also refers to the “sumptuary regulations of the colony,” indicating that excessive spending on food and personal items is not acceptable and would not be embraced (option 1), and thus this person would not seem to share those values of the community (option 3). However, there is no mention in the passage that such behavior would be met with more extreme measures such as execution (option 2). All we know is that such behavior is unacceptable and contrary to the laws and customs of the Puritan colony, so it is reasonable to assume the newcomer’s ways would be disapproved of (option 4). Hint #3 Option 4 is correct.

48. Which of the following assertions best captures an idea from the passage? Please choose from one of the following options. 1 Puritan society allowed only men to adorn bright colors during certain times of the year.

159


MCAT Practice Test 1 EXPLAINED.pages

2

*The townspeople were expecting Hester Prynne’s imprisonment to have more of a negative impact on her than it did. 3 Hester Prynne’s birth of a baby girl marked her formal incorporation into society as a valued mother. 4 The Scarlet Letter worn on Hester’ Prynne’s chest is an emblem worn by many of the women in the community. Hint #1 This is a “Foundations of Comprehension” question which is asking you to recognize an accurate paraphrase or summary of the central theme or of particular statements in the passage, or to interpret word choices made by the author. Hint #2 The passage opens with the line, “A throng of bearded men, in sad-colored garments, and gray, steeple-crowned hats…” In contrast Hester Prynne’s mark of shame, the letter A sewn on her chest, is “in fine red cloth, surrounded with an elaborate embroidery and fantastic flourishes of gold thread.” The author continues, “It was so artistically done, and with so much fertility and gorgeous luxuriance of fancy...but greatly beyond what was allowed by the sumptuary regulations of the colony.” This suggests that bright colors are not generally worn in this society, and there is no suggestion that this custom was applied differently to men at certain times of the year (option 1). When Hester emerges from the prison, the baby is presented as “a token of her shame”, rather than the mark of her being valued as a mother. There is no language to suggest that she is valued as a mother by the townspeople (option 3). The author provides no evidence that others in the community also wear emblems (option 4). The author writes that the scarlet letter had the effect of “taking [Hester] out of the ordinary relations with humanity and enclosing her in a sphere by herself” which suggests she might be the only one to wear such an emblem. Finally, the passage suggests that the townspeople are surprised by Hester’s demeanor as she emerges from the prison. The author writes, “Those who had before known her and had expected to behold her dimmed and obscured by a disastrous cloud, were astonished to perceive how her beauty shone out, and made a halo of the misfortune and ignominy in which she was enveloped” which shows that their expectations of negative effects were violated (option 2). Hint #3

160


MCAT Practice Test 1 EXPLAINED.pages

Option 2 is correct.

Writer Thomas Hardy fits neatly into the general scheme of the literature of the second half of the nineteenth century, for he was about midway between writers Matthew Arnold and Walt Whitman. He believed in the efficacy of the knowledge given to one by an understanding of science; but

161


MCAT Practice Test 1 EXPLAINED.pages

he felt that new points of view should be impregnated with the ancient lore of the past. He believed that old wine should be kept in new bottles. He refused to follow the writers who called themselves realists into the morass of words that they accumulated in their attempts to portray life and character as they thought they were. Likewise, he refused to give himself to pure impressionism. He endeavored to preserve a balance between objective reality and his interpretation of it. He was in the old sense of the word a literary artist. Hardy, of course, read the events of human life in the light of his conception of the Immanent Will. He believed that the skein of circumstance, woven blindly and flung forth indifferently, caught up in its web all human beings from the emperor in his palace to the unconscious lout lying drunk in the ditch. But Hardy was not satisfied to hold this view conjecturally. He scanned the pages of philosophy, of science, and of history to be certain that he read life aright. From them he evolved a view of life which has been called scientific determinism. It seemed to him that men moved as automata, each within his own sphere. Unseen forces played upon them; unseen powers directed them. These forces, the physical manifestation of the metaphysical Immanent Will, were three in number, to which all others were subordinated. They were the power of heredity, the shaping power of education, and the influence of environment. From them there was no escape, for every choice seemingly made by the individual, Hardy thought, was dictated by so many thousand unseen circumstances so interwoven that it was almost impossible to realize the extent to which one was enmeshed in them. For evidence to substantiate this conclusion he could point to the past and to the present. The Greeks, for example, believed that the three Fates directed every action, no matter how minute, of mortals, and Immortals- of the peasant plodding in the field, and of Zeus waving his machinations on the cloud-kissed brow of Olympus. The Christian era had introduced into the intellectual world the contrary idea of Free Will; but the world had split on that interpretation of life during the Reformation, when John Calvin gave to the world from the dark caverns of his mind the gloomy doctrine of Predestination. In the nineteenth century, the western world was probably equally divided between the theory of Free Will maintained by the Roman

162


MCAT Practice Test 1 EXPLAINED.pages

Catholic Church and a few protestant denominations, and the theory of Predestination held by all churches stemming from Calvinism. Unexpectedly, aid from an unsought source came to those who maintained that human actions were predetermined, for the evolutionary theory, expounded by Darwin and Huxley, and the psychology which grew from it, gave weight to the idea that Predeterminism fitted better with facts than the theory of Free Will. Once an anthropomorphic God was out of the picture and His place taken by an evolving Consciousness, or whatever the mind of man chose to substitute, it was almost necessary to believe in a theory of life similar to that held by Hardy. 49. Which of the following arguments would strengthen the author’s claim about the relationship between predestination and evolutionary theory? Please choose from one of the following options. 1 Evolutionary theory predicts change over time, not daily decisionmaking. 2 Calvinists believed in God and championed Predestination simultaneously. 3 Evolutionary theory is scientific, while Predestination often involves divine forces. 4 *All human decisions are determined by the brain which is a byproduct of biological evolution. This is a “Reasoning Beyond the Text” question, which means that it wants you to either apply or extrapolate the ideas in the passage to new situations or to assess how new information would impact the ideas presented in the passage. It is important to understand the assumptions underlying the article, and how new information may or may not shift the central thesis. Hint #2 In paragraph 3, the author talks about the divide between the theory of Free Will and the theory of Predestination. The author mentions Calvinism as a religion that advanced the truth of Predestination in the nineteenth century. The author then argues that not only was there growing support for Predestination within the religious sphere, but also within the realm of science, specifically the theory of evolution. The author writes, “...the evolutionary theory, expounded by Darwin and Huxley...gave weight to the

163


MCAT Practice Test 1 EXPLAINED.pages

idea that Predeterminism fitted better with facts than the theory of Free Will.” The author argues that evolutionary theory gave way to a sort of “evolving consciousness” that replaced the traditional “anthropomorphic God” making it “almost necessary to believe in a theory of life similar to that held by Hardy.” First, the argument that evolutionary theory predicts change over time and not daily decision-making (option 1) weakens the authors’ claim because that would make the theory irrelevant to the decisions that the notion of Free-Will deals with.. Second, the argument that Calvinists believe in both God and Predestination (option 2) would have no effect on the author’s claim because it does not involve evolutionary theory. Third, emphasizing how evolution and religious notions differ (option 3), weakens the similarity the passage tries to draw between religious and scientific notions of determinism that contrast against Free Will. The last argument gives more specific support to the passages general claim that evolutionary theory and “the psychology that grew from it” support the ideas that human actions are predetermined by natural forces (option 4). Hint #3 Option 4 is correct. 50. According to the passage, which two viewpoints held contrary positions regarding the existence of Free-Will? Please choose from one of the following options. 1 *Catholicism and Calvinism 2 Calvinism and Evolutionary Theory 3 Evolutionary Theory and Psychology 4 Ancient Greek philosophy and science Hint #1 This is a “Foundations of Comprehension” question which is asking you to recognize an accurate paraphrase or summary of the central theme or of particular statements in the passage, or to interpret word choices made by the author. Hint #2 The opposing positions described in the text are that of accepting that Free Will exists versus some form of determinism in which actions are outside the control of humankind. The text says that Hardy “scanned the pages of philosophy, of science”, and that “from them he evolved a view of life which has been called scientific determinism.” This suggests that the view of

164


MCAT Practice Test 1 EXPLAINED.pages

science was similar (not contrary) to that of the ancient Greeks who “believed that the three Fates directed every action” (option 4). Psychology is mentioned only as something that “grew from” and thus was not contrary to evolutionary theory (option 3). Also, evolutionary theory is claimed to “aid” the “doctrine of Predeterminism” espoused by Calvinism (option 2). That Calvinist view is described as one half of the “split on that interpretation of life” during the Reformation, with the contrasting half being the “theory of Free Will maintained by the Catholic Church” (option 1). Hint #3 Option 1 is correct. 51. Which of the following endeavors would BEST exemplify the sense of impressionism alluded to in paragraph 1? Please choose from one of the following options. 1 A sculpture of a young woman crying as she clutches the figure of a small baby in her arms. 2 A novel describing the day-to-day shopping habits of a middle-class housewife. 3 A poem detailing the dangerous and unsanitary working conditions of industrial society. 4 *A painting of teenagers suspended in space to portray their social disconnection. Hint #1 This is a “Reasoning Beyond the Text” question, which means that it wants you to either apply or extrapolate the ideas in the passage to new situations or to assess how new information would impact the ideas presented in the passage. It is important to understand the assumptions underlying the article, and how new information may or may not shift the central thesis. Hint #2 In paragraph 1, the author describes Hardy as a writer who refused to follow the realists who aim “to portray life and character as they thought they were. Likewise, he refused to give himself to pure impressionism. He endeavored to preserve a balance between objective reality and his interpretation of it.” Based on the last sentence, you can infer that the author believes that impressionists are less concerned with reality and more with “their interpretation of” reality. Option 4 is the only artistic endeavor that does not aim to objectively capture life as it actually appears.

165


MCAT Practice Test 1 EXPLAINED.pages

Rather, it is an interpretation of the emotional experience of being a teenager. Hint #3 Option 4 is correct. 52. Based on the philosophical beliefs of Thomas Hardy described in the passage, what can be inferred about the beliefs of writers Matthew Arnold and Walt Whitman? Please choose from one of the following options. 1 One believed in hedonism while the other believed in asceticism. 2 One valued ancient tradition while the other valued modern progress. 3 One preferred new bottles and the other preferred old wine. 4 One believed in fate and the other believed in free will. Hint #1 This is a “Foundations of Comprehension” question which is asking you to recognize an accurate paraphrase or summary of the central theme or of particular statements in the passage, or to interpret word choices made by the author. Hint #2 The author writes in paragraph 1 that Thomas Hardy’s beliefs were “about midway between writers Matthew Arnold and Walt Whitman.” However, Arnold and Whitman’s beliefs are never directly discussed, so you must estimate their beliefs based on how the author describes Hardy’s beliefs. In paragraph 1, the author writes, that Hardy “believed in the efficacy of the knowledge given to one by an understanding of science; but he felt that new points of view should be impregnated with the ancient lore of the past.” Thus, we can assume that Arnold and Whitman must have had opposite perspectives about the value of older knowledge versus the value of contemporary knowledge (option 2). Additionally, the phrase “old wine should be kept in new bottles” illustrates that the key contrast is old versus new, but this is a metaphor for Hardy’s beliefs and not to be taken literally (option 3). Hedonism refers to the pursuit of pleasure and self-indulgence, while asceticism refers to avoidance of pleasure and self-indulgence (option 1). These concepts are not directly related to what is discussed in paragraph 1. The discussion of fate and free will (option 4) does not arise until later in the passage and is not directly related to the differing views of Whitman and Arnold.

166


MCAT Practice Test 1 EXPLAINED.pages

Hint #3 Option 2 is correct. 53. A central thesis in the passage is that Hardy: Please choose from one of the following options. 1 was persuaded by science that only heredity guided our choices. 2 *embraced the cross-pollination of scientific, philosophical, and artistic views of human life. 3 favored Catholicism over Calvinism because it offered a more complete explanation of the divine. 4 preferred an anthropomorphic God over an evolving Consciousness. Hint #1 This is a “Foundations of Comprehension” question which is asking you to recognize an accurate paraphrase or summary of the central theme or of particular statements in the passage, or to interpret word choices made by the author. Hint #2 The author never asserts that any single school of thought, whether artistic, scientific, or religious, dominates Hardy’s beliefs or outlook on life. Hardy is not described as a purist, but rather as a writer who had a worldview shaped by myriad belief systems. The entire passage discusses how Hardy’s scientific determinism was shaped by art (impressionism and realism), science (evolutionary theory), and religion (Calvinism) (option 2). Although Hardy is interested in the interplay between heredity, education, and the environment, he did not believe that choices were governed by any single factor (option 1). Contrary to option 3, Hardy was a determinist and Calvinism accepted a form of determinism, whereas Catholicism took the opposing view in favor of Free Will. An “evolving Consciousness” is presented as a more modern and scientific version of determinism replacing Calvinism’s anthropomorphic God. Although both share a determinism generally compatible with Hardy’s view, Hardy is said to have “evolved a view of life which has been called scientific determinism” and preferred old ideas modernized with science. Thus, contrary to option 4, any preference would likely favor the scientific version of an evolved consciousness. Hint #3 Option 2 is correct.

167


MCAT Practice Test 1 EXPLAINED.pages

Pedigree charts are used to analyze the inheritance of traits within a human population. If enough progeny data is available, careful analysis of the phenotypes can provide information about the genetic inheritance of a disease. Immunodeficiency disorders are caused when the immune system’s ability to fight infectious disease is compromised. While immunodeficiency disorders vary widely, they are typically characterized by the person’s inability to make sufficient amounts of immune cells or by the production of immune cells that are mutated such that they cannot function properly. Given the wide variety of disorders, pedigree charts can be useful in determining what form of Mendelian inheritance a particular disease follows. Bruton syndrome, is a rare genetic disorder that is characterized by a mutation in the Bruton’s tyrosine kinase gene, which blocks the ability of Bcells to develop. To better understand the inheritance patterns of Bruton’s syndrome, a family was followed for three generations (Figure 1). Figure 1. Pedigree of family with cases of Bruton’s syndrome

168


MCAT Practice Test 1 EXPLAINED.pages

Figure legend: Circles are female, squares are male, black-filled shapes indicate individuals with Bruton’s syndrome. Another study followed a family with a different immunodeficiency disorder called hyper-IgE syndrome (Figure 2). There are several forms of hyper-IgE syndrome, which is characterized by mutations that prevents proper neutrophil chemotaxis. This disease results in recurrent cold infections, eczema-like skin rashes, and lung infections. In this pedigree, the hyperIgE syndrome was characterized by a mutation in the STAT3 gene. Figure 2. Pedigree of family with cases of hyper-IgE syndrome

169


MCAT Practice Test 1 EXPLAINED.pages

Figure legend: Circles are female, squares are male, black-filled shapes indicate individuals with hyper-IgE syndrome.

1. Based on the pedigree analysis, what type of Mendelian inheritance does Bruton’s syndrome MOST LIKELY exhibit? Please choose from one of the following options. 1 Autosomal dominant 2 Autosomal recessive 3 X-linked dominant 4 *X-linked recessive In autosomal inheritance, each offspring receives one allele from the father and one from the mother. Hint #2

170


MCAT Practice Test 1 EXPLAINED.pages

In X-linked inheritance, males receive one allele from the mother (on the X chromosome) and none from the father (since the father provides the Y chromosome and no X chromosomes). In contrast females receive one allele from the father and one allele from the mother. Hint #3 Can we eliminate any of the possible types of Mendelian inheritance? Look at how Bruton's syndrome skips a generation from female 2 (affected) to her daughter female 4 (unaffected) to her grandson male 10 (affected). Hint #4 This indicates that Bruton's syndrome is likely not dominant, as both Xlinked dominant and autosomal dominant disorders do not typically skip generations. Hint #5 Now that we know Bruton's syndrome is likely recessive, we must decide whether it is likely autosomal or X-linked. Hint #6 Consider a few aspects of the pedigree: 1 The affected female 2 has two affected sons and one unaffected daughter. Mother to son transmission is a hallmark of X-linked disorders. 2 The unaffected female 4 must be a carrier for Bruton's disease as she has an affected son (10). 3 For the affected male 10 to have acquired Bruton's disease, he must have either received a diseased allele from each parent (if Bruton's is autosomal) or just from his mother (if X-linked). Hint #7 For Bruton's disease to be autosomal recessive, both male 1 and male 3 must have been carriers (see if you can convince yourself of this fact). As Bruton's disease is extremely rare, the likelihood of two carriers marrying into one family is even more exceedingly rare. Hint #8 However, if Bruton's disease is X-linked recessive, the pedigree is explained quite simply. The affected female 2 passed on a diseased X chromosome to both of her affected sons (5 and 6), as well as her unaffected daughter (4) making her a carrier. The unaffected daughter (4) then passed on a copy of her diseased X chromosome to her affected son (10) and a copy of her normal X chromosome to her

171


MCAT Practice Test 1 EXPLAINED.pages

unaffected son (8). Thus, Bruton's disease is most likely an X-linked recessive disorder. 2. Hyper-IgE syndrome shows an autosomal recessive pattern of inheritance. Based on the pedigree analysis in Figure 2, which member of the family is most likely NOT a carrier? Please choose from one of the following options. 1 Person number 8 2 Person number 5 3 *Person number 3 4 Person number 1 Hint #1 In autosomal inheritance, each offspring receives one allele from the father and one from the mother. Hint #2 In an autosomal recessive disorder, the offspring has to receive two disease alleles to exhibit the disease phenotype. Hint #3 A carrier of an autosomal recessive disorder possesses one diseased allele and one normal allele. Hint #4 Person 4 and 6, the children of Person 1 and Person 2 are affected by the disease, meaning they must have received disease alleles from both parents. Person 1 must have been a carrier in order to transfer a diseased allele to his children. Hint #5 Person 5 must have received a disease allele from his mother Person 2 and a normal allele from Person 1, making Person 5 a carrier. Hint #6 Person 8 similarly must have received a disease allele from his mother, Person 4, and a normal allele from his father, Person 3, making Person 8 a carrier. Hint #7 Person 3 is most likely not a carrier, as he passed on normal alleles to all three of his children.

172


MCAT Practice Test 1 EXPLAINED.pages

3. In Figure 1, what is the genotype of the Bruton’s tyrosine kinase allele in person number 4 ? Please choose from one of the following options. 1 This person has two normal alleles 2 This person has two disease alleles 3 *This person has one normal and one disease allele 4 This person only has one allele and it is normal In X-linked inheritance, males receive one allele from the mother and none from the father. Hint #2 The progeny of person number 4 includes one male exhibiting the disease and one who is normal. Hint #3 Person number 4 has one normal and one disease allele. 4. In Figure 2, what is the most likely genotype of the STAT3 gene in person number 3 ? Please choose from one of the following options. 1 *This person has two normal alleles 2 This person only has one allele and it is normal 3 This person has one normal and one disease allele 4 This person has two disease alleles Hint #1 In autosomal inheritance, each offspring receives one allele from the father and one from the mother. Hint #2 In autosomal recessive diseases, an individual must have two disease alleles to exhibit the disease phenotype. Hint #3 None of the progeny of person number 3 and person number 4 exhibit the disease, even though each of the progeny received one disease allele from person 4.

173


MCAT Practice Test 1 EXPLAINED.pages

Hint #4 Person number 3 has two normal alleles.

5. If a female who has Bruton’s syndrome married a male who did not have the disease, what phenotypes would their progeny exhibit? Please choose from one of the following options. 1 100% of the females will have Bruton’s syndrome, while 100% of the males will be normal 2 50% of the males and females will be normal and 50% will have Bruton’s syndrome. 3 There is not enough information to answer this questions. 4 *100% of the females will be normal, while 100% of the males will have Bruton’s syndrome. In X-linked inheritance, males receive one allele from the mother and none from the father, while females receive one allele from the father and one allele from the mother. Hint #2 In X-linked recessive inheritance, a female must have two disease alleles to exhibit the disease. Hint #3 In this case, all of the males will receive a disease allele from the mother, while the females will receive one disease allele from the mother and one normal allele from the father. 100% of the females will be normal, while 100% of the males will have Bruton’s syndrome.

174


MCAT Practice Test 1 EXPLAINED.pages

Helicobacter pylori is a Gram negative, microaerophilic bacterium characterized by its ability to survive in highly acidic conditions. This infectious pathogen, most commonly associated with Peptic Ulcer Disease (PUD), is responsible for the development of 70% of gastric ulcers, and an estimated 80-95% of duodenal ulcers, which may lead to the development of certain cancers. Once the bacteria reach the stomach, they burrow through the protective mucosal layer and adhere to the gastric epithelium, causing local cell destruction and inflammation. Most commonly, H. pylori colonizes the antrum of the stomach—the distal end of the organ which is most closely associated with the opening to the small intestine, the pyloric sphincter. Figure 1: Anatomical distribution of various secretory cell types throughout the stomach.

175


MCAT Practice Test 1 EXPLAINED.pages

Due to the relatively localized nature of H. pylori infection, it is not surprising that on microscopic analysis of a stomach afflicted by PUD, a marked destruction in somatostatin-producing D-cells is noted. The decreased presence of D-cells relative to other cell types within the stomach is accompanied by measurable increases in gastric acid production, and a decrease in luminal pH. H. pylori is capable of surviving in these harsh conditions by two mechanisms. First, the mucous layer through which it burrows to reach the epithelial surface is characterized by a more neutral pH than the gastric lumen. Second, the organism is capable of producing the enzyme urease, the effect of which is to produce a more neutral environment in the area immediately surrounding the bacterium. A strong association between chronic H. pylori infection and the development of gastric cancer has been demonstrated, though few studies have been able to accurately assess whether this risk is more strongly associated with specific microscopic subtypes of cancer or their anatomical distribution. Retrospective studies have been of limited use in this regard, 176


MCAT Practice Test 1 EXPLAINED.pages

as H. pylori does not colonize existing areas of cancer or atrophy, and thus such a model cannot fully assess the etiology of a long established gastric cancer. A 2001 review of twelve prospective studies attempted to illustrate the association between gastric cancer risk and anatomical distribution (Cardia vs. non-Cardia) in patients with a known H. pylori infection. The results of this study are shown in Figure 2, which map the Odds Ratio (a measure of association between two variables, wherein a ratio not equal to “one� denotes some association) and 95% confidence interval for the relationship between cancer development and H. pylori for the given anatomical area. Figure 2: Association between chronic H.pylori infection and the development of gastric cancers compared across twelve prospective studies. Matched Odds Ratio (OR) and 95% confidence intervals for association between H.pylori infection and non-cardia cancer (Panel A) and cardia cancer (Panel B). The area of the black squares is proportional to the study size. The white diamond shows the OR value for all studies combined, with 95% confidence interval represented by horizontal spread.

177


MCAT Practice Test 1 EXPLAINED.pages

178


MCAT Practice Test 1 EXPLAINED.pages

Data adapted from: Webb, P. M., Law, M., Varhese, C., & Forman, D. Gastric cancer and Helicobacter pylori : a combined analysis of 12 case control studies nested within prospective cohorts. Gut, 49, 347-353. 6. What is the most likely sequelae in generating the decreased luminal pH following the destruction of antral D-cells in H. pylori infection? Please choose from one of the following options. 1 Decreased production of somatostatin→ decreased inhibition of chief cells→ increased gastric acid production by chief cells 2 Decreased production of somatostatin→ decreased inhibition of Dcells→ increased gastric acid production by D-cells 3 Decreased production of somatostatin→ decreased inhibition of parietal cells→ increased gastrin production by parietal cells 4 *Decreased production of somatostatin→ decreased inhibition of Gcells→ increased gastrin production→ increased numbers of mature parietal cells Somatostatin does not have any autocrine activity, i.e. it does not feedback inhibit itself. Hint #2 Chief cells are responsible for producing pepsinogen and gastric lipase not gastric acid, which is the function of parietal cells. Hint #3 With less somatostatin, essentially all other gastric cells are free to operate with relatively greater freedom. Hint #4 (Gastrin :: G-cell). In addition to promoting increased HCl release from parietal cells, this chemical messenger also stimulates parietal cell maturation and growth. Hint #5 The correct sequelae is: decreased production of somatostatin→ decreased inhibition of G-cells→ increased gastrin production→ increased numbers of mature parietal cells

179


MCAT Practice Test 1 EXPLAINED.pages

7. Under normal conditions, what stimulates the release of somatostatin from Antral D-cells? Please choose from one of the following options. 1 Low fat content in gastric chyme 2 Acetylcholine 3 *Low gastric pH 4 Distention of the stomach Hint #1 Somatostatin is an inhibitory hormone, the function of which is to produce a net decrease in gastric/duodenal digestive activity. Active gastric digestion is characterized by gastric distention and the secretion of hydrogen ions (HCl), and thus a low gastric pH. Hint #2 Acetylcholine is a parasympathetic neurotransmitter; the physiologic effects of parasympathetic activity can reasonably be simplified to the aphorism “rest and digest�. Hint #3 Other answers have been eliminated. In addition, a low gastric pH corresponds to a high concentration of hydrogen ions (HCl). Hint #4 Low gastric pH stimulates the release of somatostatin from Antral Dcells. 8. What is the function of urease? Please choose from one of the following options. 1 Catalyzes the formation of somatostatin from urea and CO2 2 *Catalyzes the degradation of urea to ammonia and CO2 3 Catalyzes the formation of urea from ammonia and CO2 4 Catalyzes the degradation of somatostatin to urea and CO2 The passage describes urease as a protective mechanism which neutralizes the acidic environment. Intuitively, the products formed must be basic. Hint #2 The passage describes a net decrease in somatostatin. Hint #3 (NH2)2CO + H2O -> CO2 + 2NH3 180


MCAT Practice Test 1 EXPLAINED.pages

Hint #4 Recall that the function of an enzyme is often denoted by its name. The suffix “-ase” describes an enzyme which participates in degrading the root word; “urea-” in this instance, so ‘urease’ must be an enzyme which catalyzes the degradation of urea.

9. What is the best conclusion to draw from the data shown in Figure 2? Please choose from one of the following options. 1 H. pylori is positively associated with the formation of non-cardia gastric cancers and negatively associated with the formation of cardia gastric cancers 2 *H. pylori is positively correlated with the formation of non-cardia gastric cancers and is not correlated with the formation of cardia gastric cancers 3 H. pylori is positively associated with the formation of cardia gastric cancers and negatively associated with the formation of non-cardia gastric cancers 4 H. pylori is positively correlated with the formation of cardia gastric cancers and is not correlated with the formation of non-cardia gastric cancers Hint #1 A positive Odds Ratio (OR) implies that as the magnitude of one variable increases or decreases, the magnitude of the second variable changes in the same direction. Hint #2 An OR distribution with a confidence interval which straddles “one” suggests no association between variables. Hint #3 The combined OR for all studies (diamond) accounts for study size (area of dot), individual OR results (distance along X axis), and statistical weight (these factors combined). Statistical outliers are easily identified by comparing individual data plots to the combined data plot. Hint #4 Examining the combined data, it is plain to see a positive correlation in the non-cardia group, and no correlation in the cardia group, in spite of several

181


MCAT Practice Test 1 EXPLAINED.pages

studies which independently found either a positive or negative correlation for cardia cancer. Hint #5 H. pylori is positively correlated with the formation of non-cardia gastric cancers and is not correlated with the formation of cardia gastric cancers. 10. Which of the following characteristics does H. pylori likely demonstrate? Please choose from one of the following options. 1 *Utilizes Oxygen 2 Teichoic acids 3 Single lipid bilayer 4 Thick peptidoglycan layer Hint #1 The Gram stain is positive (dark colored) only if a large amount of peptidoglycans are available to bind. Hint #2 A peptidoglycan layer of varying thickness is generally present in both Gram positive and Gram negative organisms, the latter’s being sandwiched between two lipid bilayers. Hint #3 Teichoic acids are an important part of a Gram positive organism’s cell wall and serve an integral function in the organism’s adherence and penetrative properties. Hint #4 Aerophilic organisms, such as H. pylori, are capable of utilizing oxygen for their survival (contrast to anaerobic). Hint #5 H. pylori demonstrates an ability to utilize oxygen.

182


MCAT Practice Test 1 EXPLAINED.pages

Collagen is the most abundant protein in mammals and forms the main component of connective tissue. Over 28 types of collagen have been identified and described in humans. However, type I collagen constitutes over 90% of collagen. Type I collagen is found in bone, tendon, ligament, and skin. This protein is composed of three polypeptide chains, which wind around one another into a triple-helix stabilized by covalent cross-linkages. Collagen is distinctive in that its subunits are composed of a regular arrangement of Gly-Pro-X, where X can be any amino acid. Each amino acid has a precise role, as the glycine side chain is the only one that is small enough to fit into the center of the triple helix. The angle of the proline residue enables the polypeptide chains to fold into the triple helical structure. Osteogenesis imperfecta, which affects 6 to 7 per 100,000 people worldwide, typically results from genetic mutations in the collagen genes. The most common form of osteogenesis imperfecta, type I, arises from a dominant mutation in the COL1A1 gene, which results in a deficiency in collagen production. Other types of osteogenesis imperfecta, including types II, III, and IV are caused by dominant mutations in the COL1A1 or COL1A2 genes that alter the structure of type I collagen. In all cases, people with osteogenesis imperfecta suffer from bones that fracture easily. 11. Where does the mutation that causes osteogenesis imperfecta first occur? Please choose from one of the following options. 1 *Collagen gene 2 Collagen triple-helix 3 Collagen polypeptide 4 Collagen transcript Hint #1 The collagen gene is a sequence of DNA that gets transcribed into collagen mRNA. Hint #2 Collagen mRNA is translated into a collagen polypeptide. Hint #3

183


MCAT Practice Test 1 EXPLAINED.pages

All collagen polypeptides are translated from the genetic code of the collagen gene. Hint #4 The mutation that causes osteogenesis imperfecta first occurs in the collagen gene. 12. If collagen were treated with Protease X, which cleaves at the carboxyl terminal of glycine residues, and Protease Y, which cleaves at the amino terminal of proline, then what would be the length of the majority of the resultant fragments? Please choose from one of the following options. 1 1 residue 2 2 residue 3 *3 residue 4 4 residue Proteases cleave peptide bonds between amino acid residues in a protein. Hint #2 Collagen is made up of repeating segments of three amino acids, glycineproline-X, where X is any amino acid. Hint #3 A peptide bond is formed between two amino acids when the carboxyl group of one amino acid reacts with the amino group of the other amino acid. Hint #4 The length of the resultant fragments would be 3 residues. The three peptide sequence glycine-proline-X repeats, so the collagen sequence prior to the protease treatment will be: ...-glycine-proline-X-glycine-proline-X--glycine-proline-X-... Protease X cleaves at glycine's C terminal, thus cleaving the peptide bonds between the glycines and prolines. Protease Y cleaves at the N terminal of proline, thus it also cleaves the peptide bonds between the glycines and prolines. This leaves the 3 residue long fragments made up of proline-X-glycine. 13. Purified collagen protein samples were collected from the tissue of four different subjects. Which cell isolate would correlate with i) a

184


MCAT Practice Test 1 EXPLAINED.pages

patient with osteogenesis imperfecta type I and ii) a patient with osteogenesis imperfecta type II?

Please choose from one of the following options. 1 *i) Patient B ii) Patient C 2 i) Patient D ii) Patient A 3 i) Patient C ii) Patient B 4 i) Patient D ii) Patient B Hint #1 Compare each patient sample to the control. Hint #2 Osteogenesis imperfecta type I is characterized by a decrease in the amount of collagen, while osteogenesis imperfecta type II is caused by collagen with abnormal structure. Hint #3 Patient B is the only sample where the absolute copies of collagen are less than the control and therefore correlates with osteogenesis imperfecta type I. Any of the other patients could have osteogenesis imperfecta type II, as the absolute number of copies of collagen will not differ from the control.

185


MCAT Practice Test 1 EXPLAINED.pages

14. If a patient comes in with osteogenesis imperfecta type I, which of the following statements is most likely true? Please choose from one of the following options. 1 Both alleles of COL1A1 have the osteogenesis imperfecta type I mutation. 2 Both parents are recessive for the COL1A1 osteogenesis imperfecta type I mutation. 3 *One of the parents has osteogenesis imperfecta type I. 4 One of the COL1A1 alleles has a silent mutation. Hint #1 Osteogenesis imperfecta type I is caused by a dominant mutation in the COL1A1 gene. Hint #2 A dominant mutation means that only one copy of the COL1A1 gene needs to be mutated for osteogenesis imperfecta to occur. Hint #3 Silent mutations are mutations that do not change the amino acid sequence. Hint #4 A patient with osteogenesis imperfecta type I, likely inherited the dominant mutation from one parent. 15. If one allele of COL1A1 was mutated, so that one motif was now Trp-Pro-X, where X can be any amino acid, what disease type would this patient most likely display? Please choose from one of the following options. 1 *Osteogenesis imperfecta type IV 2 No disease state would be displayed 3 This mutation would be lethal 4 Osteogenesis imperfecta type I Hint #1 Osteogenesis imperfecta type I arises from a mutation that causes a deficiency in overall type I collagen production. Osteogenesis imperfecta type IV arises from a mutation that alters the structure of type I collagen. Hint #2

186


MCAT Practice Test 1 EXPLAINED.pages

The fact that the glycine side chain is very small is important for forming the triple-helix structure of collagen. Hint #3 The tryptophan side chain is much bulkier, than the glycine side chain. Hint #4 A tryptophan substitution would disrupt the structure of the collagen triple-helix, resulting in osteogenesis imperfecta type IV. 16. Which of the following amino acids has a net negative charge at physiologic pH (~7.4)? Please choose from one of the following options. 1 Lysine 2 Glutamic Acid 3 Asparagine 4 Histidine Hint #1 Amino acids with acidic side groups carry a net negative charge at physiologic pH. Hint #2 Glutamate (glutamic acid) has a side group containing a carboxylic acid. Hint #3 Glutamate has a net negative charge at physiologic pH. 17. The lagging strand of a DNA molecule undergoing replication reads 3’-CGCATGTAGCGA-5’. What is the code of the DNA that is the template for the complementary leading strand of this segment? Please choose from one of the following options. 1 5’-GCGTACATCGCT-3’ 2 5’-CGCATGTAGCGA-3’ 3 3’-CGCATGTAGCGA-5’ 4 3’-GCGTACATCGCT-5’ Hint #1 The leading strand is complementary to the lagging strand, i.e. 5’GCGTACATCGCT-3’. Hint #2 The newly synthesized strands of DNA (the leading and lagging strands) are complementary to their template strands.

187


MCAT Practice Test 1 EXPLAINED.pages

Hint #3 The template strand for the leading strand is complementary to the leading strand. The newly synthesized leading strand is also complementary to the lagging strand. Hint #4 As the template leading strand and the newly synthesized lagging strand are both complementary to the newly synthesized leading strand, the template leading strand and the newly synthesized lagging strand will have the same sequence. As the newly synthesized lagging strand's sequence was given as 3’-CGCATGTAGCGA-5’, this is the sequence of the template leading strand, and the correct answer. 18. In sickle-cell disease, a glutamate -> valine substitution results in formation of HbS molecules, which: Please choose from one of the following options. 1 *Aggregate abnormally and cannot adequately carry O2 2 Have abnormally high-affinity binding for O2 3 Stabilize the wall of the red blood cell against oxidative damage 4 Cause experience high levels of repulsion between neighboring HbS molecules HbS has diminished, not enhanced, binding capacity for O2. Hint #2 Rather than stabilizing the wall of the cell, HbS causes a cell that tends to collapse and sickle, hence the disease’s name. Hint #3 As mentioned in the tutorial, HbS aggregates in an abnormal fashion, causing red cell sickling, and cannot normally carry O2. 19. ATP hydrolysis under standard biochemical conditions has ∆G < 0, implying which of the following about the reaction under these conditions? Please choose from one of the following options. 1 Cleavage of the phosphoanhydride bond that occurs during hydrolysis does not require the input of energy 2 The hydrolysis reaction decreases entropy

188


MCAT Practice Test 1 EXPLAINED.pages

3 The hydrolysis reaction is exothermic 4 *The hydrolysis reaction is spontaneous Cleaving a covalent bond always requires an input of energy. Hint #2 Exergonic reactions are not necessarily exothermic: entropy matters as well. Hint #3 As a rule of thumb, if a reaction has more products than reactants, entropy is usually positive. Hint #4 The hydrolysis reaction is spontaneous. 20. Two hours after being administered a high dose of glucose patient A blood glucose concentration read 7.6 mmol/L, then after 6 hours his glucose dropped to 7.0 mmol/L. On the other hand patient B numbers were 8.3 mmol/L, and 8.1 mmol/L after 2 and 6 hours, respectively, for the same dose of glucose. What can be said about the hormonal changes and sugar metabolic pathway of each patient? Please choose from one of the following options. 1 *Patient A experienced high secretion of insulin leading to glycogenesis 2 Patient B experienced high secretion of glucagon leading to glycolysis 3 Patient B experienced low secretion of glucagon leading to glycogenolysis 4 Patient A experienced low secretion of insulin leading to lipogenesis Hint #1 Glucagon is secreted in response to hypoglycemia, these patients just received a dose of glucose. Hint #2 Glycogenolysis would increase blood sugar levels Hint #3 Insulin removes excess glucose from the blood. Low insulin secretion would not clear the glucose in patient A. Hint #4 After an increase in his blood sugar levels, patient A experienced high secretion of insulin leading to glycogenesis

189


MCAT Practice Test 1 EXPLAINED.pages

Dietary branched-chain amino acids (BCAA) are marketed to athletes as a supplement that stimulates protein formation in muscle and reduces muscle breakdown. Branched-chain amino acids are those amino acids that have a side chain consisting of an aliphatic group with a branch point, including leucine, isoleucine, and valine (Figure 1). They account for 35% of the essential amino acids found in muscle proteins. Figure 1. Branched-chain amino acid properties Amino Acid Valine' ' Isoleucine' ' Leucine' '

R-group CH(CH3)2' ' CH(CH3)CH2CH3 CH2CH(CH3)2

pKa amino group 9.74'' 9.76'' 9.74''

pKA carboxyl group '

2.30 2.32 2.32

The medication rapamycin has important immunosuppressive effects and its mechanism has been well studied. The mammalian target of rapamycin (mTOR), is a conserved serine/threonine protein kinase that regulates protein metabolism. mTOR integrates intracellular inputs such as growth factors (in particular BCAA amino acids) to promote protein translation. Activated mTOR results in the phosphorylation of a translation initiation protein called eIF-4EBP1. Unphosphorylated eIF-4EBP1 blocks translation by sequestering a different translation initiation protein called eIF4E, which is responsible for binding the mRNA cap and bringing the mRNA transcript to the ribosome. Phosphorylation of eIF-4EBP1 by mTOR causes eIF-4EBP1 to release eIF4E, and results in the initiation of translation. Recently, studies in animal models have suggested that certain BCAAs are more effective than others in activating mTOR (Figure 2), suggesting that nutritional supplements could be more effectively targeted toward building muscle through this pathway.

190


MCAT Practice Test 1 EXPLAINED.pages

Figure 2. Percentage phosphorylated eIF-4EBP1

You may use a calculator for this question. 21. What is the pI, or isoelectric point, of I) isoleucine, II) leucine, and III) valine? Please choose from one of the following options. 1 I) 6.03 II)6.04 III)6.02 2 *I) 6.04 II)6.03 III)6.02 3 I) 7.41 II)7.44 III)7.45 4 I) 7.44 II)7.41 III)7.45 The pI, or isoelectric point, is the pH at which an amino acid carries no net electrical charge. Hint #2

191


MCAT Practice Test 1 EXPLAINED.pages

A functional group exists in protonated form at any pH below its pKa and in deprotonated form at any pH above its pKa. Hint #3 The pH at which you will find the pI of an amino acid is the average of the pKas of the carboxyl and amino groups.

22. Which of the following amino acid substitutions would likely be the least disruptive to protein functionality? Please choose from one of the following options. 1 leucine → arginine 2 isoleucine → lysine 3 *valine → alanine 4 glutamic acid → glutamine Hint #1 A protein can be functional only if it is folded into its proper configuration, which depends on the properties of the individual amino acids. Hint #2 Amino acid substitutions are least disruptive to protein structure when the R group maintains the previous amino acid's non-polar or polar properties. Hint #3 Both valine and alanine are non-polar, therefore this substitution would be the least disruptive to the protein's three-dimensional configuration, and in turn, its functionality. 23. In Figure 2, which mouse has a higher level of mTOR basal activity? Please choose from one of the following options. 1 Mouse 1 2 Mouse 2 3 Both mice have the same mTOR basal activity levels 4 Neither of the mice have any mTOR basal activity Hint #1 In Figure 2, mTOR activity is measured by the phosphorylation levels of eIF-4EBP1. Hint #2

192


MCAT Practice Test 1 EXPLAINED.pages

In Figure 2, the basal activity of mTOR is measured by the baseline levels of eIF-4EBP1 phosphorylation. Hint #3 The baseline levels of eIF-4EBP1 phosphorylation are measured by the control sample. Mouse 2 has higher levels of eIF-4EBP1 phosphorylation.

24. In Figure 2, which BCAA increases the activity of mTOR? Please choose from one of the following options. 1 Valine 2 Isoleucine 3 *Leucine 4 None of the BCAAs The the activity of mTOR is measured by the phosphorylation levels of eIF-4EBP1. Hint #2 The increase or decrease of mTOR activity can be determined by comparing the phosphorylation levels of eIF-4EBP1 in the control with the samples with added amino acids. Hint #3 Leucine is the only amino acid that increases the phosphorylation levels of eIF-4EBP1 within the margin of error. 25. If nutritional companies use the data presented in Figure 2 as a reason to take supplements, which of the following assumptions are they making? I. Any increase in eIF-4EBP1 phosphorylation is due to the mTOR pathway II. The BCAAs are only activating the mTOR pathway III. Increases in eIF-4EBP1 phosphorylation leads to increased protein levels Please choose from one of the following options. 1 I and II 2 II and III

193


MCAT Practice Test 1 EXPLAINED.pages

3 *I and III 4 I, II, and III Figure 2 is based on the assumption that increases in mTOR activity result in increased muscle mass. Hint #2 As long as the BCAA activation of the mTOR pathway leads to eIF-4EBP1 phosphorylation in this experiment, it doesn't matter what else the BCAAs act on. Hint #3 The nutritional companies are assuming that any increase in eIF-4EBP1 phosphorylation levels is due to the activation of the mTOR pathway by BCAAs, and that this in turn leads to increased protein levels.

194


MCAT Practice Test 1 EXPLAINED.pages

Transition state analogues are molecules that closely resemble a subtrate’s transition state structure in a reaction catalyzed by enzymes. The study of transition state analogues is one of the few techniques of rational drug design used to develop new pharmaceuticals. One of the more well known drugs discovered through this technique is Oseltamivir, which is a potent therapy against the infectious influenza virus. Oseltamivir is a competitive inhibitor for the enzyme neuraminidase and prevents neuraminidase from acting on its intended substrate, sialic acid. Neuraminidase is normally found on the influenza virus protein coat and hydrolyzes sialic acid molecules found on the exterior of host cell membranes to allow newly made viral particles to exit the original host cell, and begin infecting other host cells. Through the tight binding of Oseltamivir to neuraminidase’s active site, this process is inhibited, and the infectivity of the influenza virus is reduced. Due to the fact that Oseltamivir is a transition state analogue of sialic acid, their chemical structures are quite similar, as shown below:

195


MCAT Practice Test 1 EXPLAINED.pages

26. Which reactant, aside from sialic acid, would be necessary for neuraminidase to catalyze its reaction? Please choose from one of the following options. 1 Oseltamivir 2 An electron source 3 A co-factor or co-enzyme 4 *Water What kind of reaction does neuraminidase catalyze? Hint #2 Aside from the major substrate, what other substrate do hydrolases need in order to properly catalyze their reaction. Hint #3 Hydrolases catalyze reactions between a specific substrate and water in order to break a covalent bond.

196


MCAT Practice Test 1 EXPLAINED.pages

27. The rate of the uncatalyzed hydrolysis of sialic acid is 107 times slower than the rate of the same reaction with the neuraminidase catalyst. What accounts for this difference? Please choose from one of the following options. 1 Neuraminidase increases the activation energy of the reaction, thereby making the reaction more kinetically favorable. 2 Neuraminidase increases the activation energy of the reaction, thereby making the reaction more thermodynamically favorable. 3 *Neuraminidase decreases the activation energy of the reaction, thereby making the reaction more kinetically favorable. 4 Neuraminidase decreases the activation energy of the reaction, thereby making the reaction more thermodynamically favorable. Enzymes do not change any thermodynamic properties of a reaction. Hint #2 Reactions with lower activation energies have faster rates. Hint #3 By decreasing the activation energy of the reaction, neuraminidase makes the reaction more kinetically favorable.

28. At what point during the course of the neuraminidase catalyzed reaction with sialic acid, does the substrate most closely resemble the Oseltamivir analogue? Please choose from one of the following options. 1 When the energy of the enzyme-substrate complex is at its highest point. 2 When the energy of the enzyme-substrate complex most closely resembles the energy level of the original reactants. 3 When the energy of the enzyme-substrate complex is at its lowest point. 4 When the energy of the enzyme-substrate complex most closely resembles the energy level of the resulting products. Hint #1 Oseltamivir is an analogue of sialic acid’s transition state structure. Hint #2

197


MCAT Practice Test 1 EXPLAINED.pages

Transition states are the most energetically unfavorable state during a reaction. Hint #3 Sialic acid will most closely resemble Oseltamivir when the energy of the enzyme-substrate complex is at its highest point – at the time of the reaction’s transition state. 29. If a 5 mL sample of 0.1 mM neuraminidase is added to a flask containing equal parts Oseltamivir and sialic acid, which of the two substrates is more likely to be bound to the neuraminidase? Please choose from one of the following options. 1 Sialic acid, as it has a greater affinity for neuraminidase as it can more readily bind the enzyme’s transition state structure. 2 *Oseltamivir, as it has a greater affinity for neuraminidase, as it can more readily bind the enzyme’s transition state structure. 3 Sialic acid, as it has a greater affinity for neuraminidase, as it can more readily bind the enzyme’s resting structure. 4 Oseltamivir, as it has a greater affinity for neuraminidase, as it can more readily bind the enzyme’s resting structure. Hint #1 Enzymes and substrates bind most tightly at the corresponding reaction’s transition state. Hint #2 Oseltamivir’s structure most closely resembles the structure of sialic acid when at the transition state of the neuraminidase catalyzed reaction. Hint #3 Oseltamivir will have a greater affinity for neuraminidase as transition state analogues are often the most tightly bound substrates for any enzyme. 30. Which fundamental principle surrounding enzymes explains why transition state analogues are able to act as potent inhibitors of naturally occurring enzymes? Please choose from one of the following options. 1 Allosteric Inhibition 2 Enzyme Specificity 3 Proximity and Orientation Effects

198


MCAT Practice Test 1 EXPLAINED.pages

4

*The Induced Fit Theory

Enzymes and substrates do not fit together like puzzle pieces immediately upon binding. Hint #2 Enzymes and substrates will change their structure slightly upon binding to increase the affinity between the two during the reaction’s transition state. Hint #3 The induced fit theory states that binding between enzymes and substrates is strongest at the corresponding reaction’s transition state.

An overriding trend in the medical profession is the movement toward increasingly patient-specific treatment protocols, or so called ‘personalized medicine’. In accordance with this transition away from generalized applications of medical knowledge, clinical and pharmacological research has taken a particular interest in evaluating the outcomes of patients with increasingly specific risk factors and co-morbidities. Figure 1: The renin-angiotensin-aldosterone system and the site of ACE Inhibitor action

199


MCAT Practice Test 1 EXPLAINED.pages

Angiotensin-converting enzyme (ACE) inhibitors are a first line pharmacological therapy in the management of hypertension (high blood pressure) and congestive heart failure. The primary mechanism by which these drugs achieve lower blood pressures in hypertensive patients is due to the reduction in activity of ACE and thus the inhibition of the conversion of angiotensin I to angiotensin II. In addition, ACE Inhibitors have been shown to inhibit the degradation of the vasodilating peptide, bradykinin, by ACE. Further details of this mechanism are shown in Figure 1. Due to the multifactorial nature of the renin-angiotensin-aldosterone system (RAAS), several potential targets for pharmacological treatment are readily identified. Indeed, in addition to ACE Inhibitors, angiotensin receptor blockers (ARBs), as well as direct renin inhibitors are routinely used. Although all of these drugs are effective in the management of hypertension, each one does so by means of a different mechanism, oftentimes accompanied by unique secondary effects. The efficacies of each of these drugs are continually evaluated through clinical practice, and as new research and potential new applications come to light, treatment algorithms are regularly adjusted. A 1999 study, noting that a strong majority of hypertensive individuals also demonstrate insulin resistance and decreased peripheral glucose dispersal

200


MCAT Practice Test 1 EXPLAINED.pages

as well as hyperinsulinemia, examined the effects of ACE Inhibitors and ARBs on glucose transport in insulin resistant muscle. Obese rats were injected with water, Captopril (an ACE Inhibitor), bradykinin, or eprosartan (an ARB). After treatment, the rats were anesthetized, and both epitrochlearis muscles removed, and incubated in a solution of 8mM glucose, 32mM mannitol, and 0.1% bovine serum albumin (BSA). One muscle from each rat was incubated with the addition of a 2mMUnits/ mLinsulin medium. After twenty minutes of incubation, the muscles were dissolved and used to determine glucose transport activity. The results of the experiment are shown in Figure 2.

Figure 2: Effects of acute treatment with Captopril, bradykinin, or Eprosartan on glucose transport activity. Controls (vehicle-treated) are demonstrated for each treatment group, and the glucose uptake in the absence (-) and presence (+) of insulin reported with the net increase above basal (∆) caused by insulin. A “*” denotes data that is statistically significant.

201


MCAT Practice Test 1 EXPLAINED.pages

To further elucidate causal relationships and eliminate potential statistical confounding factors, the experiment is run again, this time including a treatment group which received a dose of “Drug X�. Drug X is a well documented agent that is known to produce significant anti-hypertensive effects in patients and experimental models. The mechanism by which this is achieved is through the direct inhibition of renin. Passage adapted from: Henriksen, E. J., Jacob, S., Kinnick, T. R., Youngblood, E. B., Schmit, M. B., & Dietze, G. J. ACE inhibition and glucose transport in insulin resistant muscle: roles of bradykinin and nitric oxide. American Journal of Physiology - Regulatory, Integrative and Comparative Physiology, 277.

31. What is the effect of Aldosterone on blood pressure, where is it synthesized, and where is its major site of action? Please choose from one of the following options. 1 Increase; hypothalamus; distal nephron 2 Decrease; adrenal glands; loop of Henle 3 Decrease; hypothalamus; loop of Henle 4 *Increase; adrenal glands; distal nephron

202


MCAT Practice Test 1 EXPLAINED.pages

Aldosterone increases salt reabsorption. Salt is a major osmole, and where salt goes, water tends to follow. The effect is an increased body water volume. Hint #2 Aldosterone is a steroid hormone. Hint #3 Aldosterone acts at the distal tubule and collecting ducts. Through a chemical cascade the distal nephron adds sodium channels to the tubular membrane, and upregulates Na/K ATPase activity on the interstitial (basolateral) membrane. The effect is increased reabsorption of salt and water and an increase in blood volume, thus blood pressure. Hint #4 Aldosterone produces an increase in blood pressure, is synthesized in the adrenal glands, and its major site of action is the distal nephron. 32. Based on this passage, which of the treatment groups from the initial experiment would the results of the new treatment group with “Drug X� most likely resemble and why? Please choose from one of the following options. 1 The Captopril group, because the actions of Drug X and Captopril both result in a net decrease in renin 2 The Eprosartan group, because the inhibition of renin would mimic the blood pressure effect of blocking AT receptors 3 *The Eprosartan group, because neither Drug X nor Eprosartan would inhibit the degradation of bradykinin 4 The Captopril group, because the actions of Drug X and Captopril both occur higher in the RAAS pathway than Eprosartan Hint #1 Inhibition of renin would produce a decreased formation of Angiotensin and other downstream products and effects. ACE is not a product of the RAA system. Hint #2 The passage does not indicate that renin is involved in the degradation of bradykinin. Hint #3

203


MCAT Practice Test 1 EXPLAINED.pages

Captopril inhibits ACE which would block downstream RAAS activation, as well as bradykinin degradation. Hint #4 Inhibition of renin would produce many of the same effects as Eprosartan, which blocks necessary receptors for the RAAS to take effect, and the results of Drug X would most likely resemble those of Eprosartan. Hint #5 Drug X would resemble the Eprosartan group, because neither Drug X nor Eprosartan would inhibit the degradation of bradykinin. 33. Which of the following activate the renin-angiotensin-aldosterone system? Please choose from one of the following options. 1 Increased blood volume 2 Decreased renal sympathetic activity 3 Increased early distal tubule osmolarity 4 *Decreased blood pressure ‘Increased blood volume’ is almost always synonymous with ‘increased blood pressure’. Hint #2 Increased blood pressure→ increased renal artery pressure→ decreased renal sympathetic nerve firing. Hint #3 A drop in blood pressure/ volume would result in an obligate decrease in renal blood flow and thus GFR. A low GFR means less filtrate, and therefore a fall in tubular Na+, K+, and Cl- (decreased tubular osmolarity). This is sensed by the macula densa in the early distal tubule, which signals increased renal sympathetic nerve firing, which signals the production of renin. Hint #4 Decreased blood pressure would activate the RAAS. 34. Based on the passage, what is the association between treatment with Captopril and insulin-resistant muscles? Please choose from one of the following options. 1 *Increases in the response to insulin in insulin-resistant muscles 2 Decreases in the amount of available insulin in insulin-resistant muscles

204


MCAT Practice Test 1 EXPLAINED.pages

3

Increases in the amount of available insulin in insulin-resistant muscles 4 Decreases in the response to insulin in insulin-resistant muscles The amount of Insulin available is a constant in this experiment (2mU/mL) Hint #2 Increased intracellular glucose is indicative of increased glucose transport, which is mediated by insulin. Hint #3 Considering hints 1 and 2, the only conclusion that can be drawn is that treatment with Captopril increases the response of these muscles to insulin. Observe the change from baseline (pre-insulin treatment) in the Captopril group compared with that of the control. Hint #4 Captopril is associated with increase in the the response to insulin in insulin-resistant muscles. 35. With respect to stroke volume and cardiac output, what physiologic changes in the heart would be expected after treatment with Captopril? Please choose from one of the following options. 1 Decreased cardiac output, increased stroke volume 2 Increased cardiac output, decreased stroke volume 3 Increased cardiac output, increased stroke volume 4 *Decreased cardiac output, decreased stroke volume Blocking the RAA system promotes a decrease in blood volume, and therefore blood pressure. Hint #2 Decreased blood pressure is achieved by 1. decreased salt and water reabsorption mediated by aldosterone and 2. decreased vasoconstriction by angiotensin II (a net vasodilation occurs). Hint #3 Cardiac output = stroke volume x heart rate; stroke volume = end diastolic volume - end systolic volume. Decreased blood volume leads to decreased venous return to the heart, which is congruent with a decrease in preload (end diastolic volume), thus a decrease in stroke volume. Hint #4

205


MCAT Practice Test 1 EXPLAINED.pages

The decreased vasoconstriction due to the absence of angiotensin II also produces a decrease in afterload but not in a magnitude significant enough to exceed the decrease in preload. Therefore, the reduced stroke volume produces a decreased cardiac output. Hint #5 Treatment with Captopril would induce a decrease in cardiac output and a decrease in stroke volume. 36. Which intermolecular process primarily drives the formation of a bilayer when phospholipids are added to water? Please choose from one of the following options. 1 Phospholipids self-assemble into a bilayer due to the strong affinity they have for each other. 2 A bilayer arrangement maximizes the strength of Van der Waals forces among phospholipids. 3 *Lipids cause water to arrange in an ordered, unfavorable cage-like structure. Forcing lipids into a bilayer reduces this effect. 4 The ordered arrangement of a bilayer is more favorable than the disordered state of individual free-floating phospholipids. Several intermolecular forces contribute to (or subtract from) the favorability of a phospholipid bilayer. These include stronger Van der Waals forces between hydrophobic phospholipid tails, increased entropy of water molecules, decreased entropy of phospholipid molecules, and stabilization by hydrogen bonds. Hint #2 Systems tend towards increased entropy. A disordered state is more favorable than an ordered state. Hint #3 Notice that two answers are essentially the same: “Phospholipids selfassemble into a bilayer due to the strong affinity they have for each other” and “A bilayer arrangement maximizes the strength of Van der Waals forces among phospholipids.” Typically, if two answers are the same, they can both be eliminated. Van der Waals forces among phospholipids are indeed much stronger in a bilayer, but another process is the primary driver of bilayer formation. Hint #4

206


MCAT Practice Test 1 EXPLAINED.pages

The formation of a phospholipid bilayer is primarily driven by an increase in entropy of surrounding water molecules. Lipids cause water to arrange in an ordered, unfavorable cage-like structure (called a Clathrate cage). Forcing lipids into a bilayer reduces this effect, thus increasing entropy. 37. Which of the following transcriptional control features are found in both prokaryotes and eukaryotes? Please choose from one of the following options. 1 Introns 2 *Promoter repressors 3 5’ methyl-G capping 4 Differential splicing Introns are bits of DNA that are cut out by molecules found in the nucleus of cells. Hint #2 Differential splicing refers to the ideas that different combinations of introns can be cut out to form variations on a protein. Hint #3 The splicing out of introns, as well as the addition of the 5’ methyl-G cap are actions that occurs before an RNA molecule can leave the nucleus. Hint #4 Both prokaryotes (without a nucleus) and eukaryotes have repressor molecules that can bind DNA at promoter sites and regulate transcription. 38. A scientist is investigating a specimen in a laboratory. She is attempting to determine whether it is a virus or not. Which of the following would allow her to conclude that it is not a virus? Please choose from one of the following options. 1 The specimen is extremely small 2 The specimen has a protein coat 3 *The specimen contains DNA and RNA 4 The specimen has no organelles Viruses are extremely small, so this would not allow her to eliminate the possibility. Hint #2

207


MCAT Practice Test 1 EXPLAINED.pages

Viruses are not living cells, and as such do not have organelles. They rely entirely on host cell machinery. Hint #3 Viral nucleic acid is either single-stranded DNA, single-stranded RNA, double-stranded DNA, or double-stranded RNA. It would not have both DNA and RNA, as its size would not allow them to carry that much genetic information. 39. Oogenesis, the process that creates female gametes, is halted at prophase I until puberty. Which of the following describes the DNA content of a female’s gametes during her childhood? Please choose from one of the following options. 1 *46 chromosomes, 92 chromatids 2 23 chromosomes, 23 chromatids 3 46 chromosomes, 46 chromatids 4 23 chromosomes, 46 chromatids Prophase I is the first step of the first division of meiosis. Hint #2 A chromatid is a single copy of the genetic information, while chromosomes are counted by the number of centromeres. Hint #3 In prophase I the cell has undergone DNA replication, but has yet to divide, leaving it with 46 chromosomes and 92 chromatids. 40. A previously unknown cell type is discovered in the central nervous system of a laboratory strain of mice. In cell-culture experiments, researchers find that the cell type can self-renew, but can only differentiate into one other cell type. Which of the following best characterizes this cell type? Please choose from one of the following options. 1 Multipotent embryonic stem cell 2 *Unipotent somatic stem cell 3 Multipotent somatic stem cell 4 Unipotent embryonic stem cell All stem cells must have the ability to reproduce copies of themselves as well as differentiate into other cell types. 208


MCAT Practice Test 1 EXPLAINED.pages

Hint #2 Some stem cells, such as the ones that replenish the skin, are unipotent, meaning that they are capable of differentiating into only one cell type. Hint #3 The unknown cell type that the researchers discovered is best characterized as a unipotent somatic stem cell type The existence of leptin, a 167 amino acid polypeptide hormone that functions to control appetite and feeding behavior, was hypothesized by D.L. Coleman in 1978, almost 20 years before advances in molecular biology allowed for the isolation of the hormone and the identification of its associated gene. Coleman derived his hypothesis from the interpretation of experiments he performed with mice harboring a genetic mutation called db. Mice with this mutation, called db/db mice, are obese, diabetic, and hyperphagic (exhibit uncontrolled eating). When Coleman connected the circulatory system of a lean mouse with the circulatory system of a db/db mouse (referred to as “parabiosis�), the lean mouse eventually died of starvation, while the db/db mouse lived on, unaffected. In order to explain this result, Coleman hypothesized that the db/db mice produce a circulating molecule that, when introduced to the lean mouse via parabiosis, causes the lean mouse to cease eating, eventually leading to death by starvation. Subsequent research confirmed Coleman’s hypothesis: db/db mice produce abnormally large quantities of leptin, and the introduction of these large quantities of hormone to the circulatory system of lean mice precipitates their starvation and eventual death. In addition to his studies with the db mutation, Coleman also studied mice with a different genetic mutation, occurring on a different chromosome, called ob. Mice with this mutation, called ob/ob mice, are obese, diabetic, and hyperphagic, just like db/db mice. The following table contrasts the results of parabiosis experiments involving db/db mice, lean mice, and ob/ ob mice.

209


MCAT Practice Test 1 EXPLAINED.pages

Table 1. Results of three parabiosis experiments: db/db with lean, db/db with ob/ob, and lean with ob/ob.

Experim ent one

db/db: no change observed

Lean: hypophagia, hypoinsulinemia, and hypoglycemia. Death by starvation.

Experim ent two

db/db: no change observed

ob/ob: hypophagia, hypoinsulinemia, hypoglycemia, and reduction in adipose tissue mass. Death by starvation

Experim ent three

Lean: no change observed

ob/ob: normalization of eating behavior, blood glucose levels, and circulating insulin levels. Reduction in adipose tissue mass.

41. How many nucleotides long is the gene that codes for the hormone leptin? Please choose from one of the following options. 1 Exactly (167 x 3) base pairs 2 *Greater than (167 x 3) + 3 base pairs 3 Exactly (167 x 3) + 6 base pairs 4 Exactly (167 x 3) + 3 base pairs Leptin is a 167 amino acid protein. Hint #2 According to the central dogma of molecular biology, each amino acid in a polypeptide is coded for by a three nucleotide sequence in the gene that codes for the polypeptide. Hint #3 Eukaryotic genes contain a start codon, which codes for an amino acid (methionine), and a stop codon, which does not. Hint #4 Eukaryotic genes contain introns, sequences of nucleotides within genes that do not correspond to amino acids in the functional protein. Hint #5 The leptin gene is greater than [(167*3)+3] nucleotides long. 210


MCAT Practice Test 1 EXPLAINED.pages

42. Which of the following statements represents the best explanation for why db/db mice produce abnormally high levels of leptin? Please choose from one of the following options. 1 The db gene codes for a transcription factor bound by leptin in the nucleus of adipocytes, and this transcription factor is constitutively active in db/db mice, resulting in leptin resistance in leptin-sensitive cells and hypersecretion of leptin by leptin-producing cells. 2 The db gene codes for a transmembrane receptor on the surface of leptin sensitive cells, and this transmembrane receptor is constitutively active in db/db mice, resulting in leptin resistance in leptin-sensitive cells and hypersecretion of leptin by leptin-producing cells. 3 *The db gene codes for a transmembrane receptor on the surface of leptin sensitive cells, and this transmembrane receptor is rendered inactive in db/db mice, resulting in leptin resistance in leptin-sensitive cells and hypersecretion of leptin by leptin-producing cells. 4 The db gene codes for a transcription factor bound by leptin in the nucleus of adipocytes, and this transcription factor is inactivated in db/db mice, resulting in leptin resistance in leptin-sensitive cells and hypersecretion of leptin by leptin-producing cells. Leptin is a polypeptide hormone. Hint #2 Protein hormones function via interaction with transmembrane receptors. Hint #3 Molecules secreted by leptin-sensitive cells participate in a negative feedback loop that modulates the production of leptin. Hint #4 The db gene codes for a transmembrane receptor on the surface of leptin sensitive cells. This transmembrane receptor is rendered inactive in db/db mutants, resulting in leptin resistance in leptinsensitive cells and hypersecretion of leptin by leptin-producing cells. 43. A hypothetical mouse mutation is identified that results in the absence of any circulating leptin in mice harboring the mutation: which of the following statements describe a cellular mechanism that

211


MCAT Practice Test 1 EXPLAINED.pages

could explain this lack of circulating leptin? Assume that the mutation is a loss of function mutation. I. The mutation occurs in the leptin gene. II. The mutation occurs in the gene that codes for a chaperone protein involved in post-translational modification of leptin. III. The mutation occurs in the gene that codes for a transcription factor that acts as a co-repressor of the leptin gene. Please choose from one of the following options. 1 III only 2 *I and II only 3 I and III only 4 I, II, and III A mutation in a gene coding for a co-repressor of another gene could render the co-repressor inactive, which could reasonably be assumed to result in increased production of the latter gene’s protein product. Hint #2 Problems in post-translational modification of a protein could result in a failure by cells to produce the protein. Hint #3 The mutation could occur in the leptin gene, or in a gene that codes for a chaperon protein involved in the post-translational modification of leptin.

44. Which of the following compensatory changes should one expect to see as a result of the increased concentration of plasma glucose in hyperglycemic ob/ob mice? Please choose from one of the following options. 1 Increased secretion of glucagon by pancreatic α-cells. 2 *Increased secretion of insulin by pancreatic β-cells. 3 Increased secretion of growth hormone by somatotropic cells of the anterior pituitary. 4 Increased secretion of cortisol by cells in the zone fasiculata of the adrenal cortex.

212


MCAT Practice Test 1 EXPLAINED.pages

Cortisol is a glucocorticoid hormone. One of its biological effects is to increase plasma glucose. Hint #2 One of the biological effects of growth hormone is to increase plasma glucose. Hint #3 Glucagon is secreted by pancreatic alpha cells in response to decreased plasma glucose levels. Hint #4 Increased secretion of insulin by pancreatic beta cells would be a compensatory change in response to hyperglycemia of ob/ob mice. 45. An experiment is designed in which synthetic leptin is administered intravenously to db/db mice and ob/ob mice: which strain would most likely experience a cessation of the symptoms associated with their respective genetic mutations? Please choose from one of the following options. 1 Db/db mice only. 2 *Ob/ob mice only. 3 Both strains. 4 Neither strain. Experiment three of table one shows that, when parabiosed with a lean mice with normal levels of circulating leptin, ob/ob mice experience a cessation of symptoms associated with their mutation. This suggests that the ob mutation prevents ob/ob mice from producing leptin. Hint #2 Experiment one of table one shows that, when parabiosed with lean mice with normal levels of circulating leptin, no change occurs in db/db mice, while the lean mice experience symptoms associated with overexposure to leptin. This suggests that db/db mice produce large amounts of leptin, but are unable to respond properly to the presence of the hormone in the circulatory system. Hint #3 Only ob/ob mice would be likely to experience a cessation of the symptoms associated with their genetic mutation upon intravenous administration of synthetic leptin.

213


MCAT Practice Test 1 EXPLAINED.pages

Despite their hard, persistent nature and relatively static proportions, human bones are an extraordinarily dynamic tissue that is continuously resorbed, remodeled, and renewed. The maintenance of these processes requires precisely regulated homeostatic mechanisms in order to preserve adequate bone strength, structure, and density. Bone resorption is triggered by parathyroid hormone (PTH) under conditions of low calcium. PTH binds to receptors on osteoblasts which upregulate the expression of the signalling protein RANKL. RANKL binds to the receptor RANK on osteoclast precursors, activating the transcription factor Nf-kB, which signals them to differentiate and activate into functioning osteoclasts. OPG, produced in osteoblasts, competitively blocks RANKL, thereby preventing signaling. Because of this, net bone formation or resorption can be said to be a function of the ratio of OPG to RANKL. Osteoblasts synthesize and arrange bone matrix proteins, their chief role being to deposit collagen and other proteins to form new bone. These important interactions are shown in Figure 1.

214


MCAT Practice Test 1 EXPLAINED.pages

Figure 1: Key cells and signaling molecules in bone remodeling. Osteoblasts drive bone formation while osteoclasts drive bone resorption. Osteoclast differentiation and activation involves signals from PTH, receptor-ligand interaction with RANK and Nf-kB activation.

Several diseases are known in which some part of this process has been disturbed, and thus proceeds out of balance. Osteoporosis is a disease characterized by decreased bone density and a change in bone structure wherein the bones become increasingly porous. Osteoporosis affects over half of the population of the United States over 50 years old and is overwhelmingly more common among postmenopausal women, largely due to the marked decrease in estrogen levels. Estrogen plays an important role in regulating levels of RANKL and OPG. Autosomal recessive osteopetrosis (ARO) on the other hand, often referred to as “marble bone disease�, is characterized by excessively thick and dense, albeit brittle, bones. ARO is a congenital condition, affecting 215


MCAT Practice Test 1 EXPLAINED.pages

individuals from birth due to a number of genetic anomalies, typically resulting in death by the age of ten. One variant results from a mutation in the gene coding for RANKL; others from defects in OPG. A hypothetical study hopes to determine the epidemiology of osteoporosis and ARO in the United States, and attaches a survey to an annual political election ballots routinely over ten years, with a yearly sample size of nearly 10 million. The survey asks simple yes/no questions regarding if the individual (the voter) has ever had either of the two conditions. The results are shown in Table 1. Table 1: Epidemiology of bone remodeling disorders. The survey responses for three selected years of the ten year sampling period are shown. Percentages represent the frequency of the indicated response out of all potential respondents (all voters), reported to the nearly tenth of a percent. Blank surveys, illegible surveys, and responses indicating a response of “Unsure� account for the remaining percentage of potential respondents, and are not shown in the Table.

216


MCAT Practice Test 1 EXPLAINED.pages

46. Assuming each disease is due to a single defect in cells involved in bone turnover, what are the most likely defects in osteoporosis and osteopetrosis, respectively? Please choose from one of the following options. 1 Osteoporosis is due to the increased function of osteoblasts, osteopetrosis is due to the decreased function of osteoblasts 2 Both osteoporosis and osteopetrosis are due to the increased function of osteoblasts 3 Both osteoporosis and osteopetrosis are due to the decreased function of osteoclasts 4 *Osteoporosis is due to the increased function of osteoclasts, osteopetrosis is due to the decreased function of osteoclasts Osteoporosis is characterized by decreased bone density. Hint #2 Decreased bone density would be due to disproportionate bone resorption. Hint #3 Osteopetrosis is characterized by excessively dense bones. Intuitively this could mean either decreased resorption or increased formation. Hint #4 Osteopetrosis is associated with mutations in proteins important for osteoclast function. Hint #5 Osteoporosis is due to the increased function of osteoclasts, osteopetrosis is due to the decreased function of osteoclasts. 47. From what primary, embryological germ layer is bone derived? Please choose from one of the following options. 1 *Mesoderm 2 Ectoderm 3 Neural crest 4 Endoderm The neural crest is derived from ectoderm. Hint #2 Endoderm forms visceral organs. Hint #3 Bone is derived from mesoderm.

217


MCAT Practice Test 1 EXPLAINED.pages

48. Are the results of the epidemiology study valid, why/why not, and what can be concluded about the incidence of ARO in the United States from this study? Please choose from one of the following options. 1 Yes, the results are valid. The sample size is sufficient, the study is blinded, and the extensive timeframe of the study minimizes the effect of bias. It can be concluded that the number of new cases of ARO each year did not significantly change across ten years. 2 No, the results are not valid. Although the sample size is large, it is affected by non-response bias. The only conclusion that can be drawn is that there were fewer cases of ARO in the United States in 2000 than in 1990. 3 Yes, the results are valid. The sample size is large, randomized, and likely to represent the study population as a whole. It can be concluded that there have been no new cases of ARO each year. 4 *No, the results are not valid. Although the sample size is large, it is affected by considerable bias. No conclusions about osteopetrosis can be made. “Incidence” refers to new cases per unit of time, typically per year. Do not confuse it with ‘prevalence’, which refers to the total number of cases in a population. Hint #2 By attaching the survey to political surveys, all respondents must be of voting age. Hint #3 Osteopetrosis is typically a congenital condition. New cases would only appear in infants or young people. Hint #4 The results are not valid. The sample only surveys people of voting age who participate, and many groups are underrepresented. No conclusions about osteopetrosis can be made.

218


MCAT Practice Test 1 EXPLAINED.pages

49. ARO is associated with mutations in RANKL and OPG—what structural or functional changes in RANKL and OPG would most likely be found in ARO? Please choose from one of the following options. 1 *Decreased affinity of RANKL for its receptor and increased production of functional OPG would each be found in ARO 2 Decreased affinity of RANKL for its receptor and decreased production of functional OPG would each be found in ARO 3 Increased affinity of RANKL for its receptor and decreased production of functional OPG would each be found in ARO 4 Increased affinity of RANKL for its receptor and increased production of functional OPG would each be found in ARO

Functional OPG serves to block RANKL from binding RANK. Hint #2 A higher concentration of OPG would correspond to a greater degree of OPG:RANKL complexes, and a decreased ability for RANKL to bind RANK. Hint #3 Decreased RANKL binding to RANK would lead to decreased numbers of functional osteoclasts. Hint #4 Fewer osteoclasts means less resorption relative to deposition (by osteoblasts). Hint #5 Decreased affinity of RANKL for its receptor and increased production of functional OPG would each be found in ARO 50. What would be a likely effect of increased PTH secretion? Please choose from one of the following options. 1 Increased osteoclast inhibition 2 Increased OPG expression 3 Increased serum phosphate 4 *Increased free calcium PTH signaling to osteoblasts induces osteoclast differentiation and activity. Hint #2

219


MCAT Practice Test 1 EXPLAINED.pages

The bones are the principal site for calcium storage. Hint #3 Calcium and phosphate levels are typically inversely related under PTH control. Hint #4 Increased PTH secretion would lead to increased resorption and increased free calcium in the blood. For decades one of world’s deadliest infectious diseases has been AIDS, brought about by HIV. According to the World Health Organization, in 2012 there were an estimated 35 million people living with HIV, and about 1.6 million deaths due to AIDS. It is found on every inhabited continent, and while treatment has improved drastically, there is no known cure. HIV is a cytopathic retrovirus – it carries its genetic information in RNA enclosed in a protein envelope surrounded with a heavy layer of glycosylation. Reverse transcriptase transcribes the RNA into DNA, which is then inserted into host cells. The main target of the virus are CD4 T cells, also known as T helper cells. HIV enters cells by binding receptor proteins on the host’s outer membrane. HIV proteins gp120 and gp41 bind the T cell’s CD4 receptor and a coreceptor. Most often, this coreceptor is chemokine receptor 5 (CKR-5). This co-receptor is coded for by the CCR-5 gene. Both receptors are required for the virus to enter and infect the cell.

220


MCAT Practice Test 1 EXPLAINED.pages

Figure 1. Basic structure of HIV virus. RNA is contained within a capsid, surrounded by a lipid membrane. This envelope contains docking proteins such as gp120 and gp41. Image from wikimedia courtesy of the US National Institute of Health Individuals homozygous or heterozygous for a mutation known as CKR-5Δ32 have shown resistance to certain strains of the virus. However, this mutation is only found in a small portion of the population.

Figure 2. Nucleotide sequence of CKR-5 wildtype (WT) and CKR-5Δ32 The only case of being cured of HIV is a man known as the Berlin Patient. In 2006, about 10 years after discovering he had HIV, Timothy Ray Brown was diagnosed with acute leukemia while living in Berlin. As part of his cancer therapy, Mr. Brown received two stem cell transplants from a donor who was homozygous for the CKR-5Δ32 mutation. At this time, he also stopped taking his antiretroviral therapy (ART) treatment. Within a year of the transplants, HIV was undetectable and the patient’s white blood count

221


MCAT Practice Test 1 EXPLAINED.pages

had drastically increased (Figure 3). Mr. Brown remains HIV-free to this day.

Figure 3. Genotype of CCR5 variants. The Berlin Patient’s CCR5 expression before and after stem cell transplant (SCT). Data adapted from: Hutter, G. (2009). Long-Term Control Of HIV ByDelta32/Delta32 Stem-Cell Transplantation. New England Journal of Medicine, 692-698. Liu R, Paxton WA, Choe S, Ceradini D, Martin SR, Horuk R, et al. (1996). Homozygous defect in HIV-1 coreceptor accounts for resistance of some multiply-exposed individuals to HIV-1 infection. Cell, 86: 367-77. WHO Media Centre. (2014). HIV Fact Sheet World Health Organization

222


MCAT Practice Test 1 EXPLAINED.pages

51. What sort of mutation is CKR-5 Δ32? Please choose from one of the following options. 1 *Deletion 2 Missense 3 Insertion 4 Nonsense A nonsense mutation is a point mutation – a change in a single base pair – which creates a premature stop codon. Hint #2 In Figure 2, the sequences line up until a certain point. What happens after that point? Hint #3 CKR-5 Δ32 is a deletion of 32 bp, as seen below: CKR-5 WT TCT CAT TTT CCA TAC A

GT CAG TAT CAA TTC TGG AAG AAT TTC CAG ACA \red{\text{GT CAG TAT CAA TTC TGG AAG AAT TTC CAG ACA}} GT CAG TAT CAA TTC TGG AAG AAT TTC CAG ACA start color red, G, T, space, C, A, G, space, T, A, T, space, C, A, A, space, T, T, C, space, T, G, G, space, A, A, G, space, A, A, T, space, T, T, C, space, C, A, G, space, A, C, A, end color red TTA AAG ATA CKR-5Δ32 TCT CAT TTT CCA TAC ATT AAA GAT AGT CAT CTT GGG ATG ACG CAC TGC TGC ATC AAC 52. Which of the following features of HIV does NOT hinder the immune response? Please choose from one of the following options. 1 *The viral RNA genome 2 The immune system target 3 The glycosylated outer envelope 4 The error-prone reverse transcriptase The outer envelope protects the protein capsid from being lysed by the immune system. Hint #2

223


MCAT Practice Test 1 EXPLAINED.pages

The error-prone reverse transcriptase means that HIV has a very high mutation rate. This makes it difficult for the immune system to fight every version of it. Hint #3 HIV directly targets immune cells (specifically CD4 T cells), hindering the immune response. Hint #4 The immune system has evolved defenses against both RNA and DNA viruses. Thus, out of the four options, only the virus RNA genome is a feature that would not hinder the immune response. 53. How did the Berlin Patient’s cells transform after treatment? Please choose from one of the following options. 1 CD4+ , CCR5+/+ -> CD4+, CCR5-/2 *CD4+ , CCR5+/- -> CD4+, CCR5-/3 CD4+ , CCR5+/+ -> CD4+, CCR5+/4 CD4+ , CCR5+/ - -> CD4+, CCR5+/+ Figure 3 shows that before the transplant, the Berlin Patient was heterozygous for CCR5. Hint #2 After the transplant, the Berlin Patient was resistant to HIV. This means the virus could no longer gain access to his white blood cells. Hint #3 Mr. Brown went from heterozygous for CCR5 to homozygous negative. 54. If you were to design a vaccine for HIV, which of the following would be the best target? Please choose from one of the following options. 1 *Gp120, because its active site remains relatively constant among HIV mutations 2 The protein capsid, because it would destroy the structural integrity of the virus 3 CCR5, because without it the HIV cannot get into the host cell 4 Gp41, because without it the gp120 would not be attached to the virus

224


MCAT Practice Test 1 EXPLAINED.pages

Vaccines mimic antigens – distinct parts of the disease – to allow the immune system to ready a response in case of later infection. Hint #2 HIV’s high mutation and proliferation rates mean that large numbers of highly variable forms of the disease attack the body. Hint #3 Due to its relative structural stability and exterior location, gp120 is an excellent candidate for vaccine trials. 55. How did the stem cell transplant affect the immune system of the Berlin Patient? Please choose from one of the following options. 1 The transplant gave the Berlin Patient enough HIV resistant white blood cells to fight both the cancer and the virus 2 *The transplant caused the Berlin Patient to begin to make HIV resistant white blood cells 3 The transplant allowed the Berlin Patient to manufacture white blood cells, which had been halted by the HIV 4 The transplant replaced the HIV target cells with the resistant white blood cells from the donor A stem cell transplant injects new progenitor cells (often bone marrow) into the patient. Hint #2 HIV destroys white blood cells, but it does not destroy the host’s ability to produce more. Hint #3 A stem cell transplant can change a host’s genotype – in the case of the Berlin Patient, after his SCT he began to produce white blood cells resistant to HIV. 56. Which of the following structures is a part of the rhombencephalon? Please choose from one of the following options. 1 Temporal lobe 2 Thalamus 3 Substantia nigra 4 *Medulla

225


MCAT Practice Test 1 EXPLAINED.pages

Hint #1 The rhombencephalon is also known as the hindbrain. Hint #2 The substantia nigra is a midbrain structure. Hint #3 The temporal lobe is a forebrain structure. Hint #4 The hindbrain contains three structures- the pons, medulla, and cerebellum. 57. What is the correct path through the circulatory system which describes the passage of a blood clot originating in the left leg? Please choose from one of the following options. 1 *Vena cava → right atrium → right ventricle → lungs → left atrium → left ventricle → aorta 2 Vena cava → left atrium → right atrium → lungs → left ventricle → right ventricle → aorta 3 Vena cava → left atrium → left ventricle → lungs → right atrium → right ventricle → aorta 4 Vena cava → right atrium → left atrium → lungs → right ventricle → left ventricle → aorta Blood from the vena cava enters the right atrium. Blood from the atria flow into their respective ventricles. Hint #2 A blood clot in the left leg would first enter the venous system to drain back towards the heart, first encountering the vena cava. Hint #3 A clot would flow from the vena cava to the right atrium, to the right ventricle, to the lungs, left atrium, left ventricle and finally the aorta. 58. People with albinism have a defect in tyrosinase, an oxidase that helps to control skin pigment production. In what epidermal layer is tyrosinase active? Please choose from one of the following options. 1 *Stratum basale 2 Stratum spinosum 3 Stratum granulosum

226


MCAT Practice Test 1 EXPLAINED.pages

4 Stratum lucidum Tyrosinase controls melanin production by melanocytes. Hint #2 Melanocytes are found in the deepest layer of the epidermis. Hint #3 Tyrosinase in the stratum basale, which contains the melanocytes that produce melanin - skin pigment that is absent in those with albinism. 59. In humans, which meiotic phase has the longest duration? Please choose from one of the following options. 1 Prophase I in females 2 Anaphase I in females 3 Metaphase II in females 4 Prophase II in males Hint #1 In males meiosis starts at puberty and it never stops until the end of life. Hint #2 Meiosis in males takes 74 days Hint #3 Female gamete formation starts at birth, but the gametes remain arrested in meiosis I until ovulation occurs. Hint #4 From birth to beginning of ovulation at puberty eggs are locked in prophase I

Part IV of IV Psychological, Social and Biological Foundation of Behavior 59 Questions 95 Minutes

227


MCAT Practice Test 1 EXPLAINED.pages

A recent study found that obesity tends to spread like a “contagion” through a social network. In other words, when a person experiences weight gain, close friends in the same networks tend to gain weight as well. The investigators conducted a detailed analysis of a mass network of 12,067 people who had been closely followed over 32 years, from 1971 to 2003. In the study, 5,124people were used as key subjects, or “egos”, whose behavior was analyzed. Any persons linked to the egos serve as “alters”— those who may influence ego behavior. The researchers examined several aspects of obesity spread, such as clustering of obese persons within the network, association of weight gain among an individual’s social contacts, degree of dependence of association of social ties, and influence of gender or geographical distance. The researchers found that there were discernible clusters of obese persons (BMI ≥ 30) in the network at all time points. Figure 1 shows some results from the study. The extent of interpersonal association in obesity was evaluated with regression analysis. Homophily was taken into account by including a measurement of the alter’s obesity. The researchers evaluated the possible role of unobserved contemporaneous events by separately analyzing models of subgroups of the data involving ego-alter pairings. In particular, three types of “directional” friendships are defined: 1) an “ego-perceived friend” in which the ego identifies the alter as a friend;2) an “alter-perceived friend” in which the alter identifies the ego as a friend; 3) a “mutual friend” in which the identification is reciprocal. Familial ties (parents, siblings) and marital ties (spouses) are treated as reciprocal. “Immediate neighbor” denotes the geographical distance between an alter and an ego. For example, from the results, we can see that if an ego stated that an alter was a friend, the ego’s chances of becoming obese appears to increase by 57% (“risk of obesity”).

228


MCAT Practice Test 1 EXPLAINED.pages

Figure 1: Percentage increase in obesity risk for an ego based upon his/her relationship with an alter. The dependent variable in each model is the obesity of the ego. Independent variables include a timelagged measurement of the ego’s obesity, the obesity of the alter, a time-lagged measurement of the alter’s obesity, the ego’s sex, age, and education. Mean effect sizes (solid black dot) and 95% confidence intervals (line) are shown.

In later studies on the relationship between social networks and health behaviors, one of the researchers further found that existing social ties (especially close friendships) are more likely to dissolve between people who have health traits that are dissimilar, including health traits that are immutable such as height and personality, and traits that are mutable such as BMI, blood pressure, etc. In particular, those with similar BMIs are less likely to dissolve existing ties and more likely to form ties. Another study demonstrated that food choices also were made in accordance to social 229


MCAT Practice Test 1 EXPLAINED.pages

networks. In particular, spouses showed the strongest influence in food consumption behaviors, controlling for social contextual factors. Across all peers (spouses, siblings, friends), eating patterns that were most likely to be shared were “alcohol and snacks�. Source: Adapted from Christakis, N. A., & Fowler, J.H. (2007). The Spread of Obesity in a Large Social Network over 32 Years. The New England Journal of Medicine. 357(4), 370-379. O'Malley, J., & Christakis, N. A. (2011). Longitudinal analysis of large social networks: Estimating the effect of health traits on changes in friendship ties. Statistics in Medicine. M. A. 30(9), 950-964. Pachucki, M.A., Jacques, P.F., & Christakis, N.A. (2011). Social Network Concordance in Food Choice Among Spouses, Friends, and Siblings.American Journal of Public Health, 101(11), 2170-2177.

1. Which conclusion is best supported by the findings in Figure 1? Please choose from one of the following options. 1 Friends of opposite genders only marginally increased the likelihood of obesity for the ego. 2 Obese persons do not seem to selectively form social ties only with other obese persons. 3 *There is almost no effect on the ego when someone in the same geographic proximity gained weight. 4 If a mutual friend living far away gained weight, the ego would not be more likely to gain weight. Mutual friends, ego-perceived friends, and same-sex friends increase the risk of obesity the most for egos. Hint #2 Opposite-sex friends decrease the risk of obesity for egos. Hint #3 Immediate neighbors, or social ties within the same geographic proximity, show approximately a 0 percent increase in risk of obesity for egos. 2. Which of the following is NOT a plausible policy implication of the peer effects findings in the above passage? Please choose from one of the following options.

230


MCAT Practice Test 1 EXPLAINED.pages

1

2

3

4

It may be possible to exploit variations in people’s social network position to target interventions where they may be more effective in generating benefits for the group, such as key nodes who exert stronger influence on others. Group-level interventions such as Alcoholics Anonymous or other support groups that can serve as a set of artificial social network ties may be more successful than individual-level interventions. *Shared external sources contribute to obesity, so interventions that take common environmental factors into consideration can effectively target individual health behavior. If we spend $500 to get a person to quit smoking, this person’s quitting may in turn result in his or her social contacts quitting, increasing the cost-effectiveness of an intervention.

The studies all show that group peer effects identified through social networks influence risks of obesity. Hint #2 The studies show that there are peer effects on various sorts of health behavior such as smoking or food consumption. Hint #3 Since the studies emphasize peer effects through social groups, contacts, and networks in influencing individual health behavior, a plausible policy implication would not be the examination of external environmental factors, but rather one that focuses on utilizing social ties. 3. If the studies were combined to investigate the effects of social networks on food selection and how this food selection is associated with the likelihood of obesity, how would this change the design of the first study (social network effects on obesity)? Please choose from one of the following options. 1 A new dependent variable would be added. 2 *A new mediating variable would be added. 3 A new independent variable would be added. 4 The variables would not change. Hint #1

231


MCAT Practice Test 1 EXPLAINED.pages

When examining the effect of social networks on obesity, social networks would be the independent variable and obesity levels would be the dependent variable. Hint #2 When taking food selection habits into account, the model becomes: social networks-->food selection-->obesity level. Hint #3 A mediating variable is a variable that attempts to assess the mechanisms underlying in the relationship between an independent and dependent variable. Therefore, food selection habits as a mediating variable would help clarify the relationship between social networks and obesity. 4.Which of the following statement best describes the concept of homophily? Please choose from one of the following options. 1 Smokers are more likely to experience cognitive dissonance with other smokers, compared to non-smokers. 2 Obese people are more likely to change their unhealthy behaviors around non-obese people. 3 *Individuals with similar health risks such as high blood pressure are less likely to dissolve social ties with one another. 4 Individuals who are in the same geographical proximity are more likely to conform to the same health behaviors. Hint #1 Homophily is defined as the tendency for people to choose relationships with other people who have similar attributes. Hint #2 Homophily in social networks also implies that individuals with similar traits are more likely to form social ties with one another, which also often impacts their actions. Hint #3 Individuals that share similar health risks such as high BP are more likely to form stronger social ties with one another, or put another way are less likely to dissolve social ties with one another.

232


MCAT Practice Test 1 EXPLAINED.pages

5. Which independent variable is most relevant for a study that solely investigates the impact of primary groups on health behaviors? Please choose from one of the following options. 1 Religious Affiliation 2 *Sibling 3 Occupational Affiliation 4 Neighbor Hint #1 Primary groups are often understood in contrast to “secondary groups”. Hint #2 Primary groups are typically closer and longer-lasting ties, while secondary groups are more impersonal and temporary. Hint #3 Siblings and other family members are considered primary groups, so the effect of siblings on health behavior is the most relevant for assessing the effect of primary groups. Religious affiliations can incorporate both primary and secondary groups so it would not be the most relevant. Significant dietary and nutritional differences can be found among racial and socioeconomic groups in the United States. These nutritional behavioral differences are the foci of many studies since they contribute to racial disparities in the incidence and prevalence of chronic disease and premature morbidity. Using survey data of over 60,000 participants collected by the US Department of Agriculture, Study 1 compared the dietary trends among 32,406 Black and White nonpregnant adults (18 years or older) of varying socioeconomic status (SES). The primary outcome was the score (0-16) on the Diet Quality Index (DQI), a composite of eight food-and-nutrient-based recommendations from the National Academy of Sciences, including areas such as “eating 5 or more servings daily of vegetables and fruits” and “limit total daily intake of sodium to 2,400 mg or less”. For each recommendation a person could score 0-2 according to specified intake amounts. A total score of 4 or less was considered to indicate a more healthy diet, and a value of 10 or more indicates a relatively less healthy diet.

233


MCAT Practice Test 1 EXPLAINED.pages

Categories of SES were based on education and income. Respondents with over 12 years of education and an income level over 350% of poverty level were categorized as “high SES”, and those with less than a highschool education and income less than 185% were classified as “low SES”. Three time periods were taken into account: 1965(I), 1977-1978(II), and 1989-1991( III ), to observe trends over time. Table 1 displays some results.

More recently Study 2 adjusts for SES, and directly explores the association between race and nutrition. Using data from the 1993-1999 California Dietary Practices Survey, a researcher examines the differences between the nutritional behavior of Blacks and Whites (n=3,350). The researcher attempts to observe whether Blacks differ significantly from Whites in terms of health-related nutritional behaviors that have established associations to the development of heart disease, cancer, stroke, and diabetes. The results indicate that, even within the same SES group, Whites are more likely, on average, to exhibit healthy nutritional behaviors 234


MCAT Practice Test 1 EXPLAINED.pages

such as the consumption of at least five fruits, dairy products, high fiber cereals, lowfat dairy products, and avoiding the consumption of deep-fried foods and snacks (statistically significant). However, for categories such as “consuming wholegrain products” or “consuming beans” no significant differences were found among the two racial groups in the same SES groups. Sources: Adapted from Popkin, B.M., Siega-Riz, A.M. & Haines, P.S. . A Comparison of Dietary Trends among Racial and Socioeconomic Groups in the United States. New England Journal of Medicine,. Race and nutrition: an investigation of Black-White differences in health-related nutritional behaviors. Sociology of Health and Illness,

6. According to Table 1 which of the following is true? Please choose from one of the following options. 1 On average among Blacks, those in the low SES group had less healthy dietary behavior compared to those in the medium SES group across all time periods. 2 Differences in the mean DQI score according to race and SES have widened over time. 3 *Dietary quality improved over time for both Blacks and Whites across all socioeconomic status. 4 Comparing Blacks and Whites in low and medium SES groups, the proportion of Blacks with poor diet quality exceeded that of Whites in each time period. The lower the DQI score, the better the diet. Hint #2 For each SES group, the average DQI score decreased over time. Hint #3 Across both Blacks and Whites of each SES group in each time period, the DQI score decreased, indicating an increase in dietary quality. 7. Consider the results from both Study 1 and Study 2. Assuming statistical significance for all findings presented, which of the following conclusion is best supported?

235


MCAT Practice Test 1 EXPLAINED.pages

Please choose from one of the following options. 1 In either study, no health-related racial disparities were found. 2 In Study 2 , the researcher finds that poor Whites have better diets than rich Whites. 3 In Study 2 the researcher shows that Blacks are more likely to have heart disease compared to Whites due to the lack of positive dietary behaviors. 4 *In Study 1, results indicate that on average, poor Whites have better dietary behaviors than rich Whites. In Study2 , the researcher does not examine actual disease outcomes, only nutritional behaviors believed to be associated with diseases. Hint #2 There is not enough information in Study2 to conclude that there are differences among rich and poor Whites in health behaviors. Hint #3 In Study 1, the mean DQI score for Whites in the low SES group is lower than the mean DQI score for Whites in the high SES group, indicating a higher-quality diet. 8. If another researcher finds that strong social support among Whites explains positive health behavior, which of the following is correct? Please choose from one of the following options. 1 Race is the independent variable, social support is the dependent variable. 2 Health behavior is the independent variable, race is the dependent variable. 3 Social support is the independent variable, race is the dependent variable. 4 *Social support is the independent variable, health behavior is the dependent variable. An independent variable is the explanatory variable. Hint #2 Social support is an independent variable since it explains varying health outcomes. Hint #3

236


MCAT Practice Test 1 EXPLAINED.pages

Since social support explains health behavior, health behavior is the outcome or dependent variable. 9. Socioeconomic gradients in health can be best explained by which of the following basic concept Please choose from one of the following options. 1 Intersectionality 2 Social exclusion 3 *Social stratification 4 Class consciousness Socioeconomic gradients have to do with the distribution of inequalities in the social system. Hint #2 Social gradients in health refer to how inequalities in population health are related to inequalities in social status. Hint #3 Socioeconomic gradients in health are an aspect of social stratification (how people are categorized into rankings of socioeconomic tiers). 10. To further understand how SES influences health behavior and outcomes, we must think about the processes in which health outcomes occur. Which of the following is NOT a causal mechanism through which SES may affect health outcomes? Please choose from one of the following options. 1 A person with higher SES has more resources to buffer health threats. 2 *A person with lower SES is more likely to have mental health issues. 3 A person with lower SES is less likely to have social support or capital that would protect against mental health diseases. 4 A person with higher SES is less likely to be exposed to bad environmental conditions that contribute to diseases. A causal mechanism in sociology refers to the processes or pathways through which an outcome is brought into being (why or how it happened). Hint #2 There is a causal mechanism leading from x (SES) to y (health outcomes). Hint #3

237


MCAT Practice Test 1 EXPLAINED.pages

SES leading to mental health issues (health outcome) is not a causal mechanism because it does not provide an explanation of the process or path.

Epidemiologically speaking, it is crucial to understand the contact structure through which disease travels. In studies of sexually transmitted diseases (STDs) in particular, the structure of sexual networks is critical for understanding STD diffusion. Generally, the network structures of disease diffusion and infection can be categorized into four types: the core infection model, the inverse core infection model, the bridging model, and the spanning tree model (see below). Figure 1. The network structure of four models of infection. For Panels A, B, and C, black circles denote actors that are high-activity or high-risk, and white circles indicate low-activity or low-risk actors. Panel D circles represent randomly selected individuals. Lines represent the relationship between two actors. The dotted lines in Panel A and B delineate the “core�.

238


MCAT Practice Test 1 EXPLAINED.pages

Disease diffusion is widespread among adolescent populations. A study mapped the romantic and sexual relationships of an entire high school population of over 800 adolescents in a midsized town in the Midwestern US for a period of 18 months. Students were asked to identify their sexual (not dating) and romantic partners (dating) in the past 18 months from a roster of other students attending their school. The study found that adolescent sexual networks are structured very differently from adult ones. In an adult sexual network, there is normally a core group of very sexually active people that links out to others. This can be envisioned as a transportation hub system where many points are connected to a small number of hubs. At the high school, on the contrary, there was no core group. Instead, the romantic and sexual network at the school created long chains of connections that spread out through the community, with few places where students directly shared the same partners with each other. This can be comparable to rural phone lines that run from a long main line to individual houses. One single component of the network was found to have 288 linked students, in one long chain. This chain featured 52%

239


MCAT Practice Test 1 EXPLAINED.pages

52, percent (288) of the romantically involved students at the high school, but most students had only one partner and they had little idea of their connections to the long network chain. Adapted from Bearman, P.S., Moody, J., & and Stovel, K. (2004). Chains of Affection: The Structure of Adolescent Romantic and Sexual Networks. American Journal of Sociology. 110 (1), 44-91. 11. Which of the four models of disease infection network structure supports the results of the adolescent sexual networks study? Please choose from one of the following options. 1 Panel A 2 Panel B 3 Panel C 4 *Panel D The researchers found that the sexual networks at the high school created long chains of connections that spread out through the community. Hint #2 The researchers found that adolescent sexual networks do not have a core group of people with high sexual activity. Hint #3 Panel D, the spanning tree infection model, is a long chain of interconnections that stretches across a population, and thus is most similar to the network structure observed in the study. 12. A study shows that a central group of infected people such as sex workers transmits disease out to others but does not pass infection directly among themselves. Which of the four models of disease infection network structure demonstrates this finding? Please choose from one of the following options. 1 Panel A 2 *Panel B 3 Panel C 4 Panel D A central group of infected people implies that there is a core group in the disease diffusion network structure. Hint #2

240


MCAT Practice Test 1 EXPLAINED.pages

The core infection model shows linkages among core actors, while the inverse core infection model does not show linkages among core actors. Hint #3 Since the study finds that a central core of infected people do not transmit disease among themselves and only to others, we can conclude that it is the inverse core infection model. 13. Which of the following statements best explains Panel C? Please choose from one of the following options. 1 It is a structure with few cycles, low redundancy, and sparse density. 2 One individual’s past partner is tied through multiple chains to his or her current or future partner. 3 *Two populations of persons engaged in different behaviors are linked by one actor. 4 One actor shows the ability to stop the transmission of STDs between two networks. Panel C describes disease diffusion dynamics as driven by a bridging process. Hint #2 For example, an IV drug user who shares needles with his drug partners and who has sex with non-IV-drug users would be the connecting actor see in the network. Hint #3 A bridging process model posits that two populations with different behaviors are linked by a few individuals who bridge the boundary between each world. 14. For epidemiological studies exploring flu transmission, “random mixing” captures the essential aspects of the diffusion process, for example, you do not choose to sit next to a person with the flu. However, for STD transmission studies, a crucial aspect to consider would be “preferred mixing” where people select partners with similar characteristics. Which of the following concept is similar to “preferred mixing”? Please choose from one of the following options. 1 Labeling 2 Symbolic interactionism

241


MCAT Practice Test 1 EXPLAINED.pages

3 Stigma 4 *Homophily Preferred-mixing indicates how there are high levels of contact among people who share similar attributes. Hint #2 Homophily is defined as the tendency for people to choose relationships with other people who have similar attributes. Hint #3 Based on the homophily principle, people often prefer mixing with those who are similar to themselves. 15. Based on the results of the study, which of the following makes sense as a policy intervention to prevent the spread of STDs in teenage populations? Please choose from one of the following options. 1 We should target high-risk teenagers to help curb the overall spread of STDs. 2 *We should have comprehensive STD education for all adolescents, not just those at highest risk. 3 We should isolate teenagers who already have STDs to prevent increased incidence of new infections. 4 We should target sexually active teenagers who are the main core conduits of STDs. Study results show that there is no particular core group of highly-active teenagers. Hint #2 For core infection models, it matters “which” actors are reached in intervention, but for spanning tree models (chains), it only matters if “some” are reached. Hint #3 A policy intervention geared towards all adolescents, and not just targeted ones, such as comprehensive STD education is what study results suggest. 16. Clinicians will see large amounts of data (labs results, patient symptoms, etc) over the course of their practice, and their brains will subconsciously group that data along certain established principles.

242


MCAT Practice Test 1 EXPLAINED.pages

Awareness of this underlying mechanism will help clinicians identify potential bias and provide better care. Which of the following best explains the Gestalt principles of grouping? Please choose from one of the following options. 1 The whole is more than the sum of its parts 2 Invariance, reification, and multistability are necessary to understanding perception 3 *The mind processes the whole rather than the sum of its parts 4 Reality is reduced to its simplest form While invariance, reification, and multistability are important concepts of perception, listing them does not explain the principle of Gestalt grouping. Hint #2 One of the observations that led to the principles is that we tend to process reality in its simplest form. However, the overarching theory of Gestalt grouping combines this observation with several others to explain how we process the world around us. Hint #3 Gestalt psychology states that the mind processes the whole of a perception rather than the sum of its parts, but makes no value judgement about more or less than. 17. The release of which neurotransmitter is associated with an increase in alertness? Please choose from one of the following options. 1 *Norepinephrine 2 Beta-endorphin 3 GABA 4 Serotonin Beta-endorphins are an opioid neuropeptide that have a role in pain perception, some behavioral patterns, and obesity. Hint #2 GABA is one of the main inhibitory neurotransmitters and has a role in decreasing anxiety. Hint #3 Serotonin is important in mood regulation, memory, and sleep. Hint #4 Norepinephrine

243


MCAT Practice Test 1 EXPLAINED.pages

18. Which of the following disorders from the DSM 5 is often characterized by delayed language development and unusual communication patterns? Please choose from one of the following options. 1 *Autism spectrum disorder 2 Selective mutism 3 Developmental coordination disorder 4 Specific learning disorder, dyslexia Selective mutism is an anxiety disorder characterized by difficulty speaking in social situations, but the individual is developmentally normative in terms of their language and communication ability. Hint #2 The specific learning disorder dyslexia is characterized by problems with accurate or fluent word recognition, poor decoding, and poor spelling abilities. Hint #3 Developmental coordination disorder is characterized by difficulties in acquiring and coordinating motor movements. Hint #4 Autism spectrum disorder is characterized by a variety of issues related to social and communication abilities, and first symptoms of the disorder typically include delayed language development and unusual communication patterns. 19. When a person is surprised by a loud noise, the information from the cochlea travels to the auditory cortex. The startle reflex causes physiological changes which include the neck muscles tensing within two-tenths of a second from the stimulus to the increased tension in the neck muscles. What is the pathway that allows this extremely quick transmission of threat information? Please choose from one of the following options. 1 *Auditory cortex => lateral and basolateral amygdala => midbrain => pons 2 Auditory cortex => midbrain => lateral and basolateral amygdala => pons

244


MCAT Practice Test 1 EXPLAINED.pages

3

Auditory cortex => lateral and basolateral amygdala => pons => motor cortex 4 Auditory cortex => midbrain => pons => lateral and basolateral amygdala The midbrain, which includes the periaqueductal grey (an area stimulated by the amygdala during the startle reflex), is a relay station for the integration of sensory information. Hint #2 The auditory cortex outputs to the amygdala. The lateral and basolateral amygdala receive input from nociception, vision, and audition. These areas control the intensity of the reaction and also send axons to the hypothalamus to control the physiological response (i.e. rise in blood pressure). Hint #3 Output to the pons rather than the motor cortex cause the startle reflex. Damage to the pons causes a loss of the startle reflex. Hint #4 Auditory cortex => lateral and basolateral amygdala => midbrain => pons 20. Which of these questions would an individual ask during the secondary appraisal according to Lazarus and Folkman’s Cognitive appraisal model? Please choose from one of the following options. 1 Is this encounter a threat to myself or a loved one? 2 *Do I have the resources to deal with this? 3 What are the possible benefits to my self-esteem from this encounter? 4 How might I be challenged by this situation? Primary appraisal ask questions regarding the risk or demand associated with a stressor as it relates to the individual. This would include personal threat and challenge assessment. Hint #2 Secondary appraisal evaluates possible outcomes, feelings, and perceptions associated with the stressor. It usually involves an assessment of one’s ability to successfully cope with the stressor. Hint #3

245


MCAT Practice Test 1 EXPLAINED.pages

The question that addresses one’s ability to cope with the stressor is: Do I have the resources to deal with this? In studies of drug usage, individual physical or psychological characteristics have been used to differentiate users from non-users. This approach, common in “deviant behavior” studies, assumes that certain traits of individuals predispose or motivate them to engage in negative behavior. However, some researchers have suggested that drug usage behavior should be instead viewed as interactive social processes, and that the influence of peers should be taken into account. An early study (Study 1) focusing on marijuana users found that socialization is an important aspect in the maintained “pleasurable” usage of the drug (regular users). The researcher conducted participant observation and interviews with 50 marijuana users. Respondents who eventually used marijuana regularly were observed to undergo three stages: 1) they learn to smoke it in a way that produces real effects; 2) they learn to recognize the effects and connect them with drug use; and 3) they learn to enjoy the sensation they perceive. Individuals who do not go through the sequence of changes do not end up using marijuana regularly. In a more recent study (Study 2) illustrating the social aspects of drugs, researchers used data from National Longitudinal Study of Adolescent Health (Add Health) to further examine the peer effects of marijuana usage among adolescents grades 7-12. The investigators looked at close friends and classmates. The measure for close friends is “nominated peers”, which refers to the percentage of friends who used marijuana in the past 30 days and were named by the respondent as friends, and the classmates measure is “grade-level peers”, which refers to the percentage of peers who used marijuana in the past 30 days in the same grade and school as the respondent. The main results are shown below in Table 1.

246


MCAT Practice Test 1 EXPLAINED.pages

Table 1: Increase in likelihood of individual marijuana usage (in%), with every 10% increase in nominated peers and grade-level peers Nominated Peers Grade-Level Peers Individual Usage Increase Observations (n)

2.8%'' 6,733''

'

4.4% 19,335

*The coefficients for both are significant (p-values= 0.000). The two groups are not mutually exclusive. Source: Adapted from Becker, H. S. (1953). Becoming a Marihuana User. American Journal of Sociology, 59(3), 235-242. Ali, M. M., Amialchuk, A., & Dwyer, D. S. (2011). The Social Contagious Effect of Marijuana Use among Adolescents. Plos One. 6(1), e16183 21. According to the above study, which concept is most relevant for explaining sustained marijuana usage? Please choose from one of the following options. 1 Antecedent predisposition 2 Social subjectivity 3 Deviance theory 4 *Symbolic interactionism The study demonstrates that social interaction is key to sustained marijuana usage. Hint #2 Through the process of social interaction, the meaning and interpretation of marijuana usage is transformed. Hint #3 The symbolic meaning of marijuana usage, developed through social interactions, explains why individuals become sustained marijuana users. 22. Which conclusion is best supported by the findings in Table 1?

247


MCAT Practice Test 1 EXPLAINED.pages

Please choose from one of the following options. 1 Classmates influence the likelihood of nominated peers using marijuana by 4.4% 2 A 4.4% increase in grade-level peers leads to more than 10% increase in the likelihood of individual marijuana usage. 3 Individual marijuana usage leads to a 2.8% increase in close friends’ marijuana usage. 4 *A 10% increase in close friends who use marijuana leads to a more than 2% increase in the likelihood of individual marijuana usage. The dependent variable is individual marijuana usage, so it is the variable that would increase and decrease with every unit increase in percentage of close friends or classmates. Hint #2 In the study, the measures for close friends and classmates are designed to be10%increments. Hint #3 With a 10% increase in close friends (nominated-peers), the likelihood of individual marijuana usage increases by more than 2%. 23. Which of the following conclusion can be drawn from Study 2 Please choose from one of the following options. 1 Peer groups do not significantly affect individual marijuana usage. 2 *Interventions targeting all students in a school may be more costeffective than targeting individual marijuana users. 3 Identifying individual marijuana users is an effective way to decrease drug consumption rates at school. 4 If a student expresses deviant behavior, the likelihood of individual marijuana usage will increase. In Study 2 , peer networks are found to influence the likelihood of individual marijuana usage. Hint #2 The effect of peer networks on individuals indicate that drug-related interventions should target larger networks. Hint #3 Interventions that utilize peer effects and target the entire network of students may be more cost-effective than singling out individual users. 248


MCAT Practice Test 1 EXPLAINED.pages

24. If we study heroin usage and addiction using a similar social process model similar to Study 1, which of the following statements is most likely to be a reasonable finding? Please choose from one of the following options. 1 Cognitive expectancies due to the rewarding drug effects of heroin significantly influence an individual’s drug usage behavior. 2 Heroin usage is learned but self-interpretation of usage is the key indicator of prolonged abuse. 3 Individuals who express deviant behavior in general are more likely to become heroin addicts. 4 *The interpretations of bodily effects and self-perceptions of heroin usage may be socially organized. A social process model emphasizes social interaction and influence in shaping behavior or actions. Hint #2 From social interactions with others, individuals develop novel interpretations or conceptual reorganizations of drug usage, different from their original self-understandings of a drug. Hint #3 Employing a social process model in examining heroin addiction, we would likely find that self-perceptions and interpretations are organized according to a user’s social interactions. 25. The above study emphasizes the effects of social interaction on drug usage. One further aspect to consider is how an individual’s drug-related behavior may be further reinforced by the way others judge or classify him/her. A guiding concept for this aspect would likely be: Please choose from one of the following options. 1 *Labeling 2 Discrimination 3 Negative reinforcement 4 Prejudice

249


MCAT Practice Test 1 EXPLAINED.pages

This concept holds that terms used to negatively identify or classify a drug user are considered to also negatively influence the person’s self and social identity. Hint #2 This concept indicates that deviant or negative behavior is not inherent; behavior is only deviant when society or other individuals refer to it as deviant. Hint #3 Labeling theory as a concept refers to how deviant individual behavior becomes even more deviant when a person is negatively labeled or classified as such. There are 6 universal emotions: happiness, sadness, anger, surprise, fear, and disgust; each can be identified by universally produced facial muscle movements. Culturally linked emotional expressions also exist, such as winking or raising one eyebrow. In addition, the triggers for the display of emotion are often linked to cultural factors, such as Western cultures’ propensity to display emotion more openly than Eastern cultures. Darwin believed facial muscles signaled emotion and since the configuration of the facial muscles is universal, facial expressions enhanced communication. This, in turn, led to an increased chance of survival. In 1971, Ekman and Friesen published a study which supported the Darwinian viewpoint. They visited a Neolithic, preliterate culture in New Guinea called the Fore people, which had been isolated from Western contact until 12 years earlier. Only subjects that met very specific criteria were recruited, amounting to less than 3% of the total Fore population. A judgment task was given through a translator, who was instructed that there were no correct answers for the task. The translator told a well-rehearsed story which is shown in Table 1. After the story, subjects were presented 3 pictures, each displaying a different emotion (1 correct, 2 distractors). These pictures had to be correctly identified by at least 70% of the literate participants of Western or Eastern ancestry in a previous study. The subject was asked to point to the picture presented, which displayed the emotion that the subject believed was being described. Table 1 displays universal emotions along with the corresponding story that was told to study subjects.

250


MCAT Practice Test 1 EXPLAINED.pages

Table 1 Universal emotion

Story told to subject

Happiness

Friends have come to visit and he or she is happy.

Sadness

A family member (such as a child) has died.

Anger

The subject is angry and about to fight.

Disgust

The subject is looking at something that smells bad.

251


MCAT Practice Test 1 EXPLAINED.pages

Surprise

The subject is seeing something new or unexpected.

Fear

The village is empty, there are no weapons, and the subject is attacked.

The null hypothesis was that there are no differences between the subjects representing the Fore tribe and the subjects representing Western or Eastern culture. No statistically significant differences were found in identification of emotions between groups except in the discrimination of fear from surprise and sadness, when the emotion described in the story was fear. Often, the subject could not tell the gender of the person but was still able to correctly identify the picture displaying the correct facial expression. Table 2 displays the results of the Fore Tribe responses in identifying the correct photograph corresponding with the correct emotion in the story. Table 2 Emotion describe d in story

Emotion shown in correct photograp h

Emotion shown in the first distractor photograph

Emotion shown in the second distractor photograph

Happine ss

Happiness

Fear

Anger

86

Sadnes s

Sadness

Anger

Disgust

69

% choosing correct photogra ph

252


MCAT Practice Test 1 EXPLAINED.pages

Anger

Anger

Surprise

Sadness

81

Disgust

Disgust

Surprise

Sadness

77

Surprise

Surprise

Happiness

Anger

65

Fear

Fear

Surprise

Sadness

28

Adapted from: Ekman, P., & Friesen, W. V. (1971). Constants Across Cultures in the Face and Emotion. Journal of Personality and Social Psychology, 11, 124-129.

26. What bias between the Fore people and Westerners could produce confounding results with the usage of the fear story (Table 1)? Please choose from one of the following options. 1 The Fore people could not interpret which emotion was being elicited due to a lack of common language, while Western people have a common language, English. 2 The Fore people have many emotions that are very similar in their expression, while individual Westerners display widely differing emotions. 3 The Fore people had difficulty understanding fear, surprise, and sadness because they had not been able to learn these facial expressions through imitation. Westerners, however, have observed these facial expressions constantly. 4 *The Fore people have different social constructs, taboos, and may interpret a situation differently when compared to Westerners. While many emotions have similar expressions, such as fear and surprise, sadness is less likely to elicit a physiologically similar response to fear and surprise (fight or flight), but was highly correlated with anger. Hint #2 253


MCAT Practice Test 1 EXPLAINED.pages

The Fore people were able to understand the questions being posed to them. The stories were translated into an intermediate language, which could then be translated to the Fore people’s native language. Hint #3 While facial expressions associated with emotion are universal, emotions can be evoked and shaped by many factors. The Fore people have different social constructs, taboos, and may interpret a situation differently when compared to Westerners. Likely, most situations which would cause a member of the Fore tribe to feel fear would also cause surprise or sadness, such as the sudden arrival of a warrior from another tribe or an unexpected encounter with a spirit. 27. What is meant by “No statistically significant differences were found in identification of emotions between groups except in the discrimination of fear from surprise and sadness?” Please choose from one of the following options. 1 The null hypothesis was confirmed. There are no significant differences between Western and Eastern groups of people in the expression of emotions, except between fear, surprise, and sadness. 2 The alternative hypothesis was confirmed. There were likely biological differences in facial structure which caused the problems with the interpretation between fear, surprise, and sadness. 3 The alternative hypothesis was confirmed. With a 1 in 3 chance, the subject had a better chance of choosing the correct photograph with happiness than with fear as the correct photograph. 4 *The null hypothesis was confirmed. There are likely cultural differences in the interpretation of fear which contributed to the difficulty with the distractors surprise and sadness. There is no significant difference in the facial structure or facial expressions between different cultures. Emotions differ culturally in relation to how and when it is appropriate to express a certain emotion. Hint #2 The null hypothesis in this case describes no difference between groups (literate participants of Western or Eastern ancestry compared to Fore tribe members). There is little statistical difference between the interpretation of

254


MCAT Practice Test 1 EXPLAINED.pages

facial expressions by the Fore tribe when compared to the Western and Eastern interpretation. Hint #3 The null hypothesis was confirmed. There are likely cultural differences in the interpretation of fear which contributed to the difficulty with the distractors surprise and sadness. 28. During the 1950’s and 1960’s, the Fore people suffered from Kuru, a spongiform encephalopathy. Suppose that during the examination of brain tissue samples from tribe members who had died from natural causes, many samples had bilateral damage to the amygdala which was not related to Kuru. Which designation (Table 2) would most likely be affected by this discovery? Please choose from one of the following options. 1 The correct photograph was sadness, while the distractors were anger and disgust. 2 The correct photograph was surprise, while the distractors were happiness and anger. 3 The correct photograph was disgust, while the distractors were surprise and sadness. 4 *The correct photograph was fear, while the distractors were surprise and sadness. The orbitofrontal cortex is associated with the processing of both positively and negatively balanced emotions. When activity is lowered in the right hemisphere, euphoria is experienced. Conversely, when activity is lowered in the left hemisphere, depression is reported. Hint #2 The recognition of facial expressions associated with sadness have been linked to the subcallosal cingulate. Hint #3 The insula and basal ganglia are most often associated with disgust. Hint #4 Anger has been associated with the left superior temporal sulcus. Hint #5 The correct photograph was fear, while the distractors were surprise and sadness. The amygdala is most consistently associated with fear.

255


MCAT Practice Test 1 EXPLAINED.pages

Damage to the amygdala has been linked to difficulty attending to facial expressions that would normally signal fear. 29. Which of these theories accurately describes the process through which emotion is perceived by the Fore people during Ekman and Friesen’s experiment? Please choose from one of the following options. 1 *Lazarus theory 2 James-Lange theory 3 Cannon-Bard theory 4 Schachter-Singer theory The Schachter-Singer theory of emotion describes an event (story describing fear) followed by a physiological response (fight or flight) which is interpreted as fear and fear is perceived. Hint #2 Cannon-Bard theory describes an event (story describing fear) which elicits simultaneous physiological response (fight or flight) and perception of an emotion (fear). Hint #3 The James-Lange theory describes an event (story describing fear) which elicits physiological response (fight or flight). Hint #4 Lazarus theory requires that interpretation must happen before arousal or emotion, which happen simultaneously. Since the event (story describing fear) was interpreted as being non-threatening before a physiological response (fight or flight) and emotion (fear) were to be perceived simultaneously, neither the physiological response (fight or flight) nor the emotion (fear) were elicited. Hint #5 Since there is no evidence that any of the subjects experienced a physiological response to any of the stories, the Lazarus theory is the correct response.

256


MCAT Practice Test 1 EXPLAINED.pages

30. The task given to the Fore people was originally developed by John Frederick Dashiell in 1927 for use with young children. What advantage(s) does this task have which allows it to be applied to the Fore people? Please choose from one of the following options. 1 A perfect translation is necessary to ensure that the emotion described in the story is understood. 2 The translator does not have to speak the language of the Fore people. 3 *This task does not require literacy or a verbal response from the subject. 4 The subject must remember what emotion is being described. A perfect translation was not possible, since an English language story was being translated into a regional language, Pidgin. The story was then translated again by a more Westernized tribe member to the tribe’s native language. One of the reasons for the choice of this task was that rehearsal by the translator was extensive, thus controlling for elaboration and story changes. Hint #2 The subject was not required to know or remember any emotions or words representing emotions for this experiment. Hint #3 By requiring the subject to point to the picture which best described the emotion from the story, this task did not require literacy or a verbal response from the subject. Jeff has two cats, Whiskers and Tiger. Jeff wants to play with both of them using a laser pointer, but the cats respond very differently to playtime. Whiskers is an active cat and loves to chase the red dot, but he sometimes gets so excited by playtime that he becomes overly agitated and attacks Jeff. Tiger is very lazy - he loves to look out the window from his bed and lounge in the sun; he is almost never interested in playing with the laser pointer. Jeff wants both of his cats to be equally active and healthy, so he decides to use principles from behaviorism to teach his cats better play behavior. He has two goals - to increase Tiger’s playtime, and to decrease Whisker’s aggressive playtime.

257


MCAT Practice Test 1 EXPLAINED.pages

He uses a variety of different strategies with his cats: he tries giving each cats treats when they play nicely, taking away Tiger’s bed to increase playtime, putting the cats in a carrier when they do not play nicely, and loudly yelling “No!” if Whiskers becomes aggressive. Jeff notices that the cats respond well to the treats, so he decides to vary when and how Whiskers and Tiger receive their treats. Table 1 outlines the different schedules of reinforcement that Jeff tries with Whiskers and Tiger. Table 1 Schedule

Cat

Strategy

A

Tiger

1 Treat for every 3 minutes of play

B

Tiger

1 Treat after a random amount of time of play

C

Whiskers

1 Treat every time play is nonaggressive

D

Whiskers

1 Treat after a random number of nonaggressive plays

31. One of Jeff’s friends suggests giving Whiskers a treat when he is aggressive, as a way to distract him. Could this be an effective way to teach Whiskers to behave properly during playtime? Please choose from one of the following options. 1 No, Whiskers would be negatively reinforced to act out more often 2 *No, Whiskers would be positively reinforced to act out more often 3 Yes, Whiskers would be negatively reinforced to behave properly 4 Yes, Whiskers would be positively reinforced to behave properly Negative reinforcement would encourage desired behavior, but requires the removal of a stimulus. Hint #2 Positive reinforcement is useful in encouraging desired behavior. Hint #3

258


MCAT Practice Test 1 EXPLAINED.pages

Jeff would be encouraging Whiskers to continue acting out if he gave him treats. 32. What type of reinforcement schedule does Schedule C describe? Please choose from one of the following options. 1 Variable interval 2 Fixed interval 3 Variable ratio 4 *Fixed ratio Variable ratio is a reward after a randomized number of responses. Hint #2 Variable interval would be a reward after a random amount of time. Hint #3 Fixed interval is a reward after a set period of time. Hint #4 Fixed ratio would be a reward after a set number of responses. 33. Which of the following describes why rewarding Tiger with a treat for every 3 minutes of play could be more effective in changing his behavior than taking away his bed? Please choose from one of the following options. 1 *More immediate feedback following desired behavior makes for a stronger consequence 2 Negative reinforcement is extinguished more quickly than positive reinforcement 3 Punishment is not the best way to shape an animal's behavior 4 Variable reinforcement is learned more quickly than fixed reinforcement The schedule described is not a variable schedule; it is fixed. Hint #2 There is no evidence that behavior reinforced positively is extinguished more or less quickly than behavior reinforced negatively. Hint #3 Jeff takes away the bed to increase Tiger's play behavior, not decrease it. Thus, taking away the bed is not punishment, it is reinforcement. Hint #4

259


MCAT Practice Test 1 EXPLAINED.pages

The immediacy of feedback is an important factor in influencing behavior. 34. When Jeff put the cats in a carrier when they didn't play nicely, what kind of operant conditioning strategy was he using? Please choose from one of the following options. 1 Positive reinforcement 2 Negative reinforcement 3 *Positive punishment 4 Negative punishment Positive refers to adding a stimulus. Hint #2 Negative refers to removing a stimulus. Hint #3 Punishment refers to the decrease of a behavior. Hint #4 Reinforcement refers to the increase of a behavior. Hint #5 When Jeff put the cats in a carrier when they didn't play nicely, he was seeking to decrease a behavior (not playing nicely), an example of punishment. Jeff added a new stimulus, placing the cats in the carrier, an example of positive operant conditioning. Hint #6 Thus the operant conditioning strategy Jeff used is best described as positive punishment. 35. Jeff has been successfully increasing the number of times Whiskers has to play nicely before he receives a treat, until finally he decides he can stop giving treats during playtime all together. Whiskers suddenly becomes ultra-aggressive - he bites, hisses, and scratches at an increased rate when playing with the laser pointer. Which of the following best describes Whisker's behavior? Please choose from one of the following options. 1 Whiskers only responds to continuous reinforcement. 2 Whiskers' behavior has been shaped to be aggressive. 3 Whisker's aggressive behavior has become generalized. 4 *Whiskers is experiencing an extinction burst.

260


MCAT Practice Test 1 EXPLAINED.pages

For Whiskers' behavior to be generalized, he would need to react aggressively during other types of play - maybe with a mouse or a feather toy. Hint #2 Whiskers' behavior has been shaped to be non-aggressive, not aggressive. Hint #3 Continuous reinforcement occurs on a 1:1 ratio - this means that for each behavior, there is a reward. Whiskers has responded well to larger ratios of reinforcement, which means that this statement is inaccurate. Hint #4 When an animal no longer receives regular reinforcement, its original behavior will sometimes spike (meaning increase dramatically) - this is known as an extinction burst. 36. Which of the following hormones is a hunger stimulant? Please choose from one of the following options. 1 *Ghrelin 2 Insulin 3 Leptin 4 Dopamine Insulin is associated with satiety, or feelings of fullness.. Hint #2 Dopamine may control motivation for food, but does not directly stimulate or inhibit hunger. Hint #3 Leptin is also associated with satiety, or feelings of fullness. Hint #4 Increases in ghrelin, the “hunger hormone,� produce increases in appetite. 37. Josephine is an American college student who spends her sophomore year studying abroad in France. Her appreciation of French culture is heavily influenced by her experiences with French cuisine. She feels that the French believe it is important for families to take pride in the quality of their food and how it is prepared. She believes that the culture benefits from families eating and enjoying their food together. She ultimately decides French culture is superior to American culture. What term describes Josephine’s beliefs?

261


MCAT Practice Test 1 EXPLAINED.pages

Please choose from one of the following options. 1 Ethnocentrism 2 *Xenocentrism 3 Cultural relativism 4 Cultural imperialism Cultural imperialism is the deliberate imposition of one’s own cultural values on another culture. Hint #2 Cultural relativism is the practice of assessing a culture by its own standards rather than viewing it through the lens of one’s own culture. Hint #3 Ethnocentrism is judging one’s own culture as superior to another culture. Hint #4 Xenocentrism is judging another culture as superior to one’s own culture. 38. Dr. Walsh is interested in seeing whether symptoms of depression can be manipulated using principles of classical conditioning. For several weeks of an experiment, he gives a group of patients a sweetened soda that has a mood-enhancing drug in it and notices that symptoms of depression improve significantly. Then, he removes the drug from the beverage and notices that the symptoms are still improved when the patients consume the soda. Which of the following is the conditioned stimulus in Dr. Walsh’s experiment? Please choose from one of the following options. 1 *The sweetened soda 2 The improved symptoms in response to the soda 3 The improved symptoms in response to the drug 4 The mood-enhancing drug The unconditioned stimulus is a stimulus that elicits an unconditioned response. Hint #2 The conditioned stimulus is a stimulus that has been paired with an unconditioned stimulus. Hint #3 The pairing of the conditioned and unconditioned stimulus produces the conditioned response.

262


MCAT Practice Test 1 EXPLAINED.pages

Hint #4 Improvement in symptoms is the response, not the stimulus. Hint #5 The soda and drug are paired together to form the conditioned response. Hint #6 The conditioned stimulus is the soda. 39. Research shows that our behavior is affected by intentions. In addition to intentions, however, it is also found that whether we have the resources or capacity to carry out these intentions also influence our behavior. This additional factor can be referred to as: Please choose from one of the following options. 1 Subjective norms 2 Planned behavior 3 *Perceived behavior control 4 Attitudes This demonstrates a person’s perception of the ease or difficulty in performing a behavior. Hint #2 Whether a person perceives that he or she has the resources or ability to carry out certain intentions influences their behavior. Hint #3 Perceived behavior control refers to a person’s ability to carry out intentions to perform a certain behavior. 40. Deviations from homeostasis can create an internal state of biological and psychological tension. What is this best term for this state? Please choose from one of the following options. 1 *Drive 2 Regulation 3 Equilibrium 4 Dissonance Equilibrium is another term for homeostasis, and is not related to tension. Hint #2

263


MCAT Practice Test 1 EXPLAINED.pages

Dissonance is a broad term for tension, but does not specifically describe the internal state of biological and psychological tension. Hint #3 Drives are internal states of bodily tension. Prosopagnosia is a neurological disorder characterized by the inability to recognize familiar people based on facial information alone. Prosopagnosia, often called facial blindness, can be acquired through lesion, stroke, head trauma, or manifested without any discernible cause. Other patients with prosopagnosia may have developmental prosopagnosia (DP), which is characterized by a lifelong deficit in facial recognition and cannot be traced to acquired brain damage. A patient with DP may not be aware of this deficit because they would form the ability to recognize people by other distinguishing features that are unrelated to their facial characteristics. For patients with DP, the ability to recognize objects can be completely unaffected or only slightly impaired, but they may struggle to recognize close family members, friends, or themselves. At one time, DP was believed to be an extremely rare disorder, with only 9 case studies conducted between 1947 and 2001. In recent years this belief has been challenged as increased numbers of patients are diagnosed with DP, and researchers now believe that as many as 1 in 50 people may have some form of prosopagnosia. A researcher interested in prosopagnosia conducts an Internet survey in which respondents click a link, sign into a Social Media site, and are connected to a test battery that shows them a series of photos of famous faces. Each face is isolated from any other identifying features and is presented with eyes forward, as shown in Figure 1, which shows the isolated face of Barack Obama, adapted from President Obama’s 2008 Official Presidential Portrait. As each photo appears, the subject is asked to provide the name they associate with the presented face. If the subject is not able to remember a name, they are told to provide a description of the famous person's work or experiences. Once the subject submits this response, the famous person's name is revealed and the subject is asked to self-report whether their response was correct or incorrect. If the name presented is unfamiliar, the

264


MCAT Practice Test 1 EXPLAINED.pages

subject selects a radio button labeled, “I do not know this person� and their response is not counted against their accuracy score. Afterwards, the researcher calculates the percentage of correct answers, excluding the pictures described as unfamiliar. The percentage of faces recognized by each respondent is presented in Figure 2 , and mean accuracy across all respondents is 82%. Figure 1

Figure 2

265


MCAT Practice Test 1 EXPLAINED.pages

41. Based on the information in the passage, which of these would describe the most accurate top-down processing that a prosopagnosic might use to recognize faces? Please choose from one of the following options. 1 The use of monocular cues such as lighting and shading to recognize the person. 2 The use of eye color, skin color, and height to recognize the person. 3 *The interpretation of gait, tone of voice, and context to recognize the person. 4 The use of facial cues to recognize the person. While skin color and height may be enough to recognize a person when that person is expected, a prosopagnosic often does not gain any useful information from eye color. In fact, in most studies there is no significant difference between the recognition of faces with or without eyes. Hint #2 While monocular cues may aid in the recognition of the person, they may also be misinterpreted and cause a misidentification. Hint #3 The interpretation of gait, tone of voice, and context to recognize a person describes the most accurate top-down process to recognize a face for a prosopagnosic.

266


MCAT Practice Test 1 EXPLAINED.pages

42. Which of the following brain areas could produce symptoms of prosopagnosia if damaged? Please choose from one of the following options. 1 Frontal lobe 2 *Fusiform gyrus 3 Cerebellum 4 Wernicke's area Damage to Wernicke's area is associated with language losses. Hint #2 Damage to the cerebellum is associated with balance and coordination dysfunction. Hint #3 Damage to the frontal lobe is associated with changes in personality and cognitive processing. Hint #4 The fusiform gyrus is a part of the visual system in the brain, and plays a role in high level visual processing and recognition. Damage to this part of the brain could result in prosopagnosia. 43. Which of these people are likely to be diagnosed with DP? Please choose from one of the following options. 1 A 45-year-old man who may have had head trauma early in life and has always had trouble with faces. 2 A 12-year-old boy who has a lesion on the occipital lobe and can no longer recognize shapes, objects, or people. 3 A 65-year-old woman who recently had a stroke and is having trouble naming faces. 4 *A 31-year-old woman who has always had trouble with faces. Developmental Prosopagnosia occurs from early in life, possibly from birth. Hint #2 Developmental Prosopagnosia does not occur as a result of trauma. Hint #3 A

3 1 31 267


MCAT Practice Test 1 EXPLAINED.pages

31 31 -year-old woman who has always had trouble with faces is most likely to be diagnosed with DP. 44. Which of these could be a possible reason for the extremely low score in the 6th respondent? Please choose from one of the following options. 1 The researcher failed to check whether the subject was colorblind. 2 The researcher chose faces that most people did not know. 3 The respondent did not know most of the faces presented. 4 *The respondent purposefully responded incorrectly. Since most people seemed to be able to identify the ‘famous’ faces presented, it is unlikely that the choice of faces presented affected respondent

6 6 6 6 ’s results. Hint #2 By giving the respondent, the option of choosing a radio button labeled “I do not know this person” and removing the response from the results if the name was not recognized, the researcher controlled for construct validity. Hint #3 Colorblindness should not affect one’s ability to identify faces. Hint #4 The respondent purposefully responded incorrectly. 45. The researcher decides to examine the results item by item, and finds that one famous face has been mis-identified by every participant - they all name her as a different actress! He realizes that the photo he chose makes the actress look like a different woman. In terms of psychometric properties, what does this item have? Please choose from one of the following options. 1 *Reliability, but not validity. 2 Validity and reliability. 268


MCAT Practice Test 1 EXPLAINED.pages

3 4

Neither validity nor reliability. Validity but not reliability. Items that are high in validity accurately address the construct. Hint #2 In this example, the participants are unable to correctly name the actress pictured - not because they can't think of a name (would indicate some level of prosopagnosia), but because the photo isn't an accurate representation of her. This means that the item is probably not a valid test for prosopagnosia. Hint #3 However, because every respondent gave the wrong answer, the item is reliable - the answers are consistent. Autism has very diverse symptoms, and comprises many similar neurological disorders, which are classified as autism spectrum disorders (ASD). Temple Grandin is an autistic woman who is best known for her research on animal behavior, pressure therapy, and the design of the ‘squeeze box’ (shown in Figure 1), a pressure device she designed after observing the calming reaction in cattle during immunization when they were confined in a squeeze chute. As a toddler Grandin would rock, spin, or become fixated on objects for hours. Grandin suffered from hypersensitivity to touch, sound, and many other types of stimuli. Her reaction was often an intense, spontaneous, and uncontrollable panic attack. A noticeable change in her anxiety and sensitivity level occurs after approximately 5 to 15 minutes in the squeeze box, with diminishing results after 45 minutes to an hour. The perceptions of pain and hypersensitivity are psychophysical. Although there is a physiological factor, this does not fully describe the experienced pain or hypersensitivity. An autism researcher conducted a study on the efficacy of Grandin’s squeeze box on children displaying signs of ASD. The child was shown how to operate the squeeze box; a trusted adult entered it and operated it, thereby demonstrating that the device was safe. The child was then asked to enter, was given control, and was asked to operate the machine for 15 minutes at different pressures throughout. Every 3 minutes the child’s stimulation was assessed. The data is shown in Figure 2 below.

269


MCAT Practice Test 1 EXPLAINED.pages

Figure 1: Temple Grandin’s squeeze box, front view. A female subject is inside with her head and hands protruding. Deep touch pressure is applied by the squeeze box in response to the subject’s manipulation of the joystick (increasing and decreasing pressure consistently across the entire body).

Figure 2: Average Likert scores for all subjects. Likert scale ranging from 1 to 5 ( 1 being calm, 5 being extremely stimulated). Each child was treated with the squeeze box for 15 minutes.

270


MCAT Practice Test 1 EXPLAINED.pages

Adapted from: Grandin, T. (1992). Calming effects of deep touch pressure in patients with autistic disorder, college students, and animals. Journal of Child and Adolescent Psychopharmacology, 2(1), 63-72. 46. Which mechanoreceptors would likely fire when the squeeze box is first engaged and touching the child’s body and when pressure is released? Please choose from one of the following options. 1 Meissner corpuscle and Merkel receptor 2 Merkel receptor and Pacinian corpuscle 3 Merkel receptor and Ruffini cylinder 4 *Meissner corpuscle and Pacinian corpuscle The Ruffini cylinder fires to constant pressure. Hint #2 The Merkel receptor also fires to constant pressure. Hint #3 Both the Meissner corpuscle and the Pacinian corpuscle fire when pressure is first applied and again when pressure is released.

271


MCAT Practice Test 1 EXPLAINED.pages

47. How does sensory adaptation impact the efficacy of the squeeze box? Please choose from one of the following options. 1 Sensory adaptation quiets the nerves allowing the subject to adjust to the constant pressure. 2 *Sensory adaptation has little impact on hypersensitivity with the use of the squeeze box. 3 Sensory adaptation has no impact on hypersensitivity with use of the squeeze box since there is constant pressure. 4 Sensory adaptation allows the subject to become adjusted to the pressure, thereby lowering hypersensitivity. The children were instructed to operate the squeeze box at different pressures. Hint #2 Sensory adaptation requires a constant strength of stimulus. Hint #3 Given that the pressure was changed throughout, sensory adaptation has little chance to develop. Sensory adaptation has little impact on hypersensitivity with the use of the squeeze box. 48. During the test, some children reported feeling like they were floating or were unaware of the position of their body in space. Which of these senses controls the symptoms that these children experienced? Please choose from one of the following options. 1 Nociception 2 Mechanoception 3 Kinesthesia 4 *Proprioception Nociception is the perception of pain. Hint #2 Kinesthesia is the sense of movement. Hint #3 Proprioception describes the sense of position and balance.

49. What conclusion can be drawn from the graph in Figure 2?

272


MCAT Practice Test 1 EXPLAINED.pages

Please choose from one of the following options. 1 Adaptation is the main cause of the increase in perceived comfort over time. 2 The length of time spent in the squeeze box is not related to the perceived comfort over time. 3 The refractory period is a major influence on the amount of perceived comfort over time. 4 *The refractory period and adaptation have little influence. Psychological comforting is perceived as time increases. The refractory period is measured in seconds rather than minutes. Hint #2 Adaptation requires a constant stimulus; the amount of pressure was varied by the child throughout the time spent in the squeeze box. Hint #3 The refractory period and adaptation have little influence. Psychological comforting is a much more likely reason for perceived comfort over time. 50. How does the average child in this study react to treatment in the squeezebox? Please choose from one of the following options. 1 The average child was unaffected by the treatment. 2 *The average child became increasingly more calm as the treatment progressed. 3 The average child did not respond for the first 6 minutes and became only slightly calmer by the end of treatment. 4 The average child became more stimulated throughout the treatment. The ‘box’ of a box and whisker plot shows the inter-quartile range. Hint #2 The average child was affected by the treatment. Hint #3 Both the average Likert score and the inter-quartile range are decreasing over the length of the treatment. Hint #4 The average child became increasingly more calm as the treatment progressed.

273


MCAT Practice Test 1 EXPLAINED.pages

Pavlov’s dogs are perhaps the most famous case of classical conditioning. In his experiments Pavlov paired the ringing of a bell with presenting his dogs with food, and then he measured the amount of saliva produced by ringing the bell alone. By taking these measurements before and after the pairing, it was shown how a stimulus that normally has no response could be paired with a stimulus that does. The principles of classical conditioning extend far beyond canines and saliva; they can even be used to condition our immune system as a means of treating autoimmune diseases. Lupus is one such autoimmune disease, and treatment requires the suppression of a person’s immune system to protect their tissues and organs from being targeted by their own white blood cells. Cyclophosphamide is a chemotherapeutic drug taken orally or intravenously that is considered to be the standard treatment for lupus; however, like many chemotherapies, the side-effects of the drug can be quite severe. In 1992 a team of researchers showed that the human immune system can be classically conditioned, such that an 11 year old girl suffering from lupus was able to have significant reduction of her symptoms without any need of immunosuppressant drugs. This provided an opportunity to treat their patient’s disease while avoiding the deleterious side effects of cyclophosphamide. For their experiment, the researchers utilized a “Compound CS” which was a liquid that tasted of cod liver oil and had the smell of a rose. They paired Compound CS with the cyclophosphamide treatments on 6 occasions for a year. Every other month of treatment, the researchers did not administer the cyclophosphamide, and simply gave their patient compound CS instead. Despite administering the immunosuppressant drug at half the normal dose, the 11 year old patient still evidenced immunosuppression and continued to do well after a 5 year followup.

274


MCAT Practice Test 1 EXPLAINED.pages

*Citation: Giang DW, Goodman AD, Schiffer RB, Mattson DH, Petrie M, Cohen N, Ader R. J Conditioning of cyclophosphamide-induced leukopenia in humans. * Neuropsychiatry Clin Neurosci. 1996 Spring;8(2):194-201. Cohen N, Ader R. Immunomodulation by classical conditioning. Adv Biochem Psychopharmacol. 1988;44:199-202. 51. If cyclophosphamide is a standard drug treatment for lupus, why should a physician consider using Compound CS and classically conditioning his or her patients? Please choose from one of the following options. 1 Classical conditioning could be used in cases where the drug is ineffective in treating lupus 2 *Compound CS could be helpful when a drug’s side-effects are not tolerable 3 Compound CS could be helpful in ending the use of cyclophosphamide to treat lupus 4 Classical conditioning could be used when a patient is not compliant with treatment The conditioning relies on the fact that the drug is efficacious, if it did not produce immunosuppression then a patient could not be conditioned with it to treat lupus. Hint #2 The research specifically only replaced half of the dosage of cyclophosphamide with Compound CS, taking cyclophosphamide is still an integral part of treatment. Hint #3 If a patient is noncompliant treatment, one would have to identify why a patient isn’t taking their medication before attempting a different means of treatment. Hint #4 Compound CS functions as an alternative treatment for when cyclophosphamide’s side effects are too severe.

275


MCAT Practice Test 1 EXPLAINED.pages

52. In Pavlov’s experiment and in the 1992 experiment involving autoimmunity, the bell and Compound CS both represent which of the following aspects of Classical Conditioning? Please choose from one of the following options. 1 Unconditioned stimulus (US) 2 Conditioned response (CR) 3 *Conditioned stimulus (CS) 4 Unconditioned response (UR) A conditioned response is the resultant behavior after a stimulus is presented. Hint #2 A stimulus is the event which prompts some behavior in the subject. Hint #3 The conditioned stimulus would be the initially neutral stimulus that is introduced by the researchers in both experiments.

53. Why did the researchers choose Compound CS to be used as the conditioned stimulus? Please choose from one of the following options. 1 Compound CS could be used to help the patient have a positive treatment experience 2 *Cod liver oil and rose perfume would not normally interfere with the immune system 3 Cod liver oil and rose perfume as a placebo would mimic the same methods of treatment as cyclophosphamide 4 Compound CS could lessen side-effects when taken in conjunction with cyclophosphamide Compound CS would be taken in liquid form, while cyclophosphamide is administered either as a pill or through IV therapy. Hint #2 The experiment was set up such that Compound CS is taken to replace cyclophosphamide, rather than being taken concomitantly. Hint #3

276


MCAT Practice Test 1 EXPLAINED.pages

Though it may help in overcoming a patient’s aversion to treatment, his or her enjoyment of Compound CS wouldn’t be necessary for the purposes of the experiment. Hint #4 Compound CS wouldn’t be useful as a placebo if it normally interefered with a patient’s immune system. 54. The use of Compound CS carries many ethical considerations because its use involves the replacement of a standard drug treatment with what is essentially a placebo. Which of the following would NOT support the use of Compound CS for a patient study? Please choose from one of the following options. 1 *Administering Compound CS would be no different from administering placebos in other experimental studies 2 Obtaining consent from a patient before undergoing treatment with Compound CS would better justify their participation in the study 3 Giving Compound CS to a patient would not cause additional harm and therefore there would be no risk in replacing cyclophosphamide 4 Enrolling a patient in the study would be better justified if they were unable to tolerate the side effects of cyclophosphamide The severity of side effects is the main reason classical conditioning is being considered to treat the disease. Hint #2 Patients must always have informed consent, especially in experimental research. Hint #3 If a patient is unable to tolerate a drug, then finding a safe alternative is ideal. Hint #4 In the case of Compound CS, it is being used differently than most placebos. In most studies, placebos are being used with the intent of comparing treatment (the drug) versus no treatment (the placebo), in the case of Compound CS, researchers were specifically trying to use a placebo as the treatment itself.

277


MCAT Practice Test 1 EXPLAINED.pages

55. If a dog was classically conditioned to salivate upon hearing a bell ring, which of the following terms would describe the salivation? Please choose from one of the following options. 1 *Conditioned response (CR) 2 Conditioned stimulus (CS) 3 Unconditioned stimulus (US) 4 Unconditioned response (UR) A stimulus in the case of Pavlov’s dogs would be either the bell ringing or the food. Hint #2 Whether or not salivating occurs alongside a stimulus determines whether it is unconditioned or conditioned response. Hint #3 The conditioned response would be the result of a paired stimulus causing a response that it normally would not, i.e. the dog salivating because of a bell ringing. 56. Which of the following statements accurately describes the terms “sex” and “gender?” Please choose from one of the following options. 1 *Sex refers to biological identity, whereas gender refers to psychological or social identity. 2 Sex and gender are interchangeable terms; both describe biological and psychological or social identity. 3 Sex and gender are interchangeable terms; both describe choices regarding sexuality and romantic partnerships. 4 Sex refers to psychological or social identity, whereas gender refers to biological identity. Sex and gender are not interchangeable terms. Hint #2 Sex is a biological construct and refers to physiological aspects of a person’s identity. Hint #3 Gender is a social or psychological construct. 57. Which of these would describe a research question viewed from a microsociology perspective?

278


MCAT Practice Test 1 EXPLAINED.pages

Please choose from one of the following options. 1 How does the conflict between Republicans and Democrats in Congress affect the U.S. role in the Middle East? 2 How does the patenting of seeds for corn, soybeans and other crops by large businesses such as Monsanto cause a conflict between local farmers and agribusiness? 3 What impacts do cultural views of medicine have on the expectations of care provided? 4 *What role do people who self-identify as locavores play in their community? A macrosociological perspective would describe the big picture interactions between social structures. Hint #2 A microsociological approach would investigate the interactions between individuals or small groups of individuals. Hint #3 Functionalism and conflict theory are normally viewed as macrosociological perspectives, while symbolic interactionism is often viewed as a microsociological perspective. Hint #4 What role do people who self-identify as locavores play in their community? 58. Structural theories of stratification predict that groups with low positions in social hierarchies experience high rates of mental health problems. The “double jeopardy” or “triple jeopardy” hypotheses postulate that groups that are subordinate in multiple stratification systems such as gender, race, and class are especially high risk. These hypotheses best demonstrate which of the following concepts? Please choose from one of the following options. 1 *Intersectionality 2 Cultural bias 3 Symbolic interactionism 4 Social constructionism There are many types of discrimination, such as discrimination based on gender, race, or class. Hint #2

279


MCAT Practice Test 1 EXPLAINED.pages

Some people experience overlapping categories of discrimination, such as discrimination in gender and race (e.g., female, Hispanic). Hint #3 Higher mental health risks for individuals who are discriminated in two or three stratification categories can be referred to the “double jeopardy” or “triple jeopardy” hypotheses. Hint #4 The theory of intersectionality proposes that we need to understand how all these discriminations (double or triple jeopardy) can simultaneously exist. 59. The “American Dream” is the idea that if a person believes in their ability to apply themselves and work towards their goals, they can achieve success and happiness. Which of the following qualities does the American Dream describe? Please choose from one of the following options. 1 *Self efficacy 2 Self concept 3 Self esteem 4 Self awareness Self awareness is the extent to which a person fixes their attention on their own self-concept. Hint #2 Self concept is a person’s beliefs or thoughts about themselves. Hint #3 Self esteem is a person’s judgements or appraisals about themselves, both positive and negative. Hint #4 Self efficacy is the sense a person has about what they can reasonably accomplish.

280


Turn static files into dynamic content formats.

Create a flipbook
Issuu converts static files into: digital portfolios, online yearbooks, online catalogs, digital photo albums and more. Sign up and create your flipbook.